Naked Science Forum

Non Life Sciences => Physics, Astronomy & Cosmology => Topic started by: hamdani yusuf on 14/10/2019 08:29:19

Title: why do a lot of people confuse between interference and diffraction?
Post by: hamdani yusuf on 14/10/2019 08:29:19
I googled "what is the difference between diffraction and interference?" and get following result on top:
Quote
``Interference'' is the more general concept: it refers to the phenemenon of waves interacting. Waves will add constructively or destructively according to their phase difference. ``Diffraction'' usually refers to the spreading wave pattern from a finite-width aperture.

If I only show pictures category, they give different answers.
(https://i.ytimg.com/vi/wtPPx0J-4JU/maxresdefault.jpg)
(https://www.thenakedscientists.com/forum/proxy.php?request=http%3A%2F%2Fcdn.differencebetween.net%2Fwp-content%2Fuploads%2F2018%2F02%2FDiffraction-VERSUS-Interference.jpg&hash=7640c78dd343c6aaaeefc044d3ca6ce3)
(https://www.thenakedscientists.com/forum/proxy.php?request=http%3A%2F%2Flatestcontents.com%2Fwp-content%2Fuploads%2F2014%2F06%2F157.jpg&hash=551b53f6eea53362dc730fb397f8a067)
(https://www.brainkart.com/media/extra/GdNAEru.jpg)
(https://www.jagranjosh.com/imported/images/E/Articles/4Effectofslitwidth.png)
(https://www.differencebetween.com/wp-content/uploads/2012/08/Difference-Between-Diffraction-and-Interference-Tabular-Form.jpg)
(https://image.slidesharecdn.com/ph-101-2-160527092812/95/ph-1012-4-638.jpg?cb=1464341312)
(https://www.thenakedscientists.com/forum/proxy.php?request=http%3A%2F%2Fimages.myshared.ru%2F17%2F1174440%2Fslide_5.jpg&hash=d9dc908ed13f970bec61f928bdddf59c)

Here are some answers from quora:
Quote
Two separate wave fronts originating from two coherent sources produce interference. Secondary wavelets originating from different parts of the same wave front constitute diffraction.
The region of minimum intensity is perfectly dark in interference. In diffraction they are not perfectly dark.
Width of the fringes is equal in interference. In diffraction they are never equal.
The intensity of all positions of maxima are of the same intensity in interference. In diffraction they do vary.
When we have two infinitely narrow slits separated by a distance apart near the source, we get interference. But when we have a single slit of finite width or rather an aperture near the source, we get diffraction.
Quote
The name interference itself gives it away. Two waves interfere or meet each other. When they do they cause an energy distribution which makes the famous bright and dark bands.

Diffraction refers to the fact that waves can bend around things. Diffraction of sound is so common, that you must have never thought about it. But since light is a wave, it diffracts too.

If you look at the two definitions, it would make one wonder, what’s the meaning of this question. The two phenomenon are so different and so nicely defined, why is there a confusion? Forget differences, what is even the similarity between the two?

Well the confusion is caused by a lot of teachers and books. It’s because of captions given to the images like these
(https://qph.fs.quoracdn.net/main-qimg-be785d243ed73a28c9af6f1007ad62ca.webp)
(https://qph.fs.quoracdn.net/main-qimg-ea002cde3ecf6a8aa1d99b9da090818c.webp)
(https://qph.fs.quoracdn.net/main-qimg-246fbf882072a76823e1dd361cf2e546.webp)

Some are called as interference patterns, and some others are called as diffraction patterns. Which is absolutely non sense. So here is what is really going on.

In each picture, the light is made to pass through slit/s. When they do, the small opening makes the light diffract (bend around and spread out). So there is diffraction happening in all of these pictures. These waves, eventually meet each other and redistribute their energies, ergo, interfere with each other, which we can see as alternate dark and bright regions.
So all patterns (bright and dark regions) are caused by interference of light (by the very definition of it), and in all experiments when slits are used there is diffraction happening at each slit.

So it’s wrong to call something as interference or diffraction pattern. All are interference pattern itself.

Then we have something called the diffraction grating. It’s called so, because it has lots of slits, and so light undergoes lots of diffraction, but again, eventually the pattern is caused by interference of light.
(https://qph.fs.quoracdn.net/main-qimg-e8cf0e4b58acde5db9a53fca9d63895e.webp)
Here is a CD. If someone asks what causes the colouration? If you say diffraction that’s wrong. The pits in the CD act like slits. So in that sense, its called a diffraction grating. But the coloration are caused by interference itself. Where you see Blue, it’s because the blue light is undergoing strong construction and rest, not so much. It’s not so different than the thin film interference, you see in oil spills and soap bubbles

(https://qph.fs.quoracdn.net/main-qimg-2ddbb7bb6a40dcb9ed62f3ef4a3dafe6.webp)
So to summarise, all the energy distribution business, which causes coloration (in white light) or alternate dark and bright bands (in mono chromatic light) are called interference. Whenever waves spread out, or bend around a corner, it’s called diffraction. The two are worlds apart.


And some from stackexchange:
Quote
Two separate wave fronts originating from two coherent sources produce interference. Secondary wavelets originating from different parts of the same wave front constitute diffraction. Thus the two are entirely different in nature.
The region of minimum intensity is perfectly dark in interference. In diffraction they are not perfectly dark.
Width of the fringes is equal in interference. In diffraction they are never equal.
The intensity of all positions of maxima are of the same intensity in interference. In diffraction they do vary.
Quote
Feynman has come from heaven to answer your question! Listen to him:

No one has ever been able to define the difference between interference and diffraction satisfactorily. It is just a quest of usage, and there is no specific, important physical difference between them. The best we can do is, roughly speaking, is to say that when there are only a few sources, say two interference sources, then the result is usually called interference, but if there is a large number of them, it seems that the word diffraction is more often used.1

To be more explicit read this passage from Ajoy Ghatak:

We should point out that there is not much of a difference between the phenomenon of interference and diffraction, indeed, interference corresponds to the situation when we consider the superposition of waves coming out from a number of point sources and diffraction corresponds to the situation when we consider waves coming out from an area sources like a circular or rectangular aperture or even a large number of rectangular apertures (like the diffraction grating). 2
Credits: 1 Feynman Lectures on Physics 2Optics-Ajoy Ghatak.

If this question shows up in a physics exam, what answer should be written by students?
Title: Re: why do a lot of people confuse between interference and diffraction?
Post by: Colin2B on 14/10/2019 09:01:47
The first definition you give is correct:
“Interference'' is the more general concept: it refers to the phenemenon of waves interacting. Waves will add constructively or destructively according to their phase difference. ``Diffraction'' usually refers to the spreading wave pattern from a finite-width aperture.” although I would add that diffraction also occurs at a sharp edge.
The confusion occurs because some of the definitions are referring to diffraction patterns, which are interference patterns from a diffracted wave.
You might like to read this http://www.math.ubc.ca/~cass/courses/m309-03a/m309-projects/krzak/
Title: Re: why do a lot of people confuse between interference and diffraction?
Post by: hamdani yusuf on 14/10/2019 10:04:58
The first definition you give is correct:
“Interference'' is the more general concept: it refers to the phenemenon of waves interacting. Waves will add constructively or destructively according to their phase difference. ``Diffraction'' usually refers to the spreading wave pattern from a finite-width aperture.” although I would add that diffraction also occurs at a sharp edge.
The confusion occurs because some of the definitions are referring to diffraction patterns, which are interference patterns from a diffracted wave.
You might like to read this http://www.math.ubc.ca/~cass/courses/m309-03a/m309-projects/krzak/
I don't think that diffraction is a subset of interference. We can produce diffraction without interference pattern.
(https://www.qsl.net/g0isw/knifeedgeeffect.gif)
(https://www.4gon.co.uk/solutions/images/fresnel_diffraction.jpg)

We can also produce interference pattern without diffraction.
(https://files.askiitians.com/cdn1/images/20171213-12219301-957-surface-of-a-denser-medium.gif)
(https://sciencedemonstrations.fas.harvard.edu/files/science-demonstrations/files/thinfilminterference-freeze-bottom-3.jpg?m=1446824435)

I think they are both results of superposition principle. Don't get it wrong. Superposition doesn't necesarily produce interference pattern. If we combine vertically polarized light and horizontally polarized light with the same frequency and amplitude, generally we'll get elliptically polarized light. The eccentricity depends on their phase difference, which can give circularly polarized light as well as linearly polarized light in diagonal orientation.

Lens and mirrors (flat, concave, convex) all work based on superposition principle. Yet they are rarely called interference phenomena.
Title: Re: why do a lot of people confuse between interference and diffraction?
Post by: alancalverd on 14/10/2019 10:24:36
Correct. Diffraction = bending, interference = superposition.

The confusion arises when you model diffraction by the superposition of wavelets, which always struck me as an unnecessary complication.
 
Title: Re: why do a lot of people confuse between interference and diffraction?
Post by: hamdani yusuf on 14/10/2019 10:38:02
Correct. Diffraction = bending, interference = superposition.

The confusion arises when you model diffraction by the superposition of wavelets, which always struck me as an unnecessary complication.
 
I added into my post after yours. I just want to show that interference is a subset of superposition.  We get interference pattern if the superposition produces darker and lighter area.
I think a contributing factor that adds into the confusion is overreaching analogy of electromagnetic wave using mechanical waves such as sound  and wave on water surface.
Title: Re: why do a lot of people confuse between interference and diffraction?
Post by: alancalverd on 14/10/2019 10:58:46
Your microwave transmitter diagram is important in radio navigation. Some systems use extremely accurate VHF (VOR) and microwave (DME) transmission, where the diffracted signal outside the line of sight is very weak but the range is limited at low altitudes. Older systems (ADF) use medium-wave transmission where the range is excellent right down to sea level but directional accuracy is compromised by all sorts of coastal, terrain and atmospheric "bending".

Not that it matters. Having studied all this and taken the tedious theory and practical exams as a living tribute to the pioneers of navigation, you get your licence, switch on the GPS, and fly or sail direct to your destination - until the time comes to renew your licence!
Title: Re: why do a lot of people confuse between interference and diffraction?
Post by: Colin2B on 14/10/2019 13:00:39
I don't think that diffraction is a subset of interference. We can produce diffraction without interference pattern.
That’s right. As Alan says diffraction is bending, and as your first quote puts it ‘spreading’.
Title: Re: why do a lot of people confuse between interference and diffraction?
Post by: hamdani yusuf on 15/10/2019 08:34:27
I think that a significant portion of the confusion comes from the assumption that single slit diffraction is the simplest case of diffraction, thus missing the explanation for single edge diffraction.

Let's examine each of the answers available online to identify which answers are the correct ones.

(https://i.ytimg.com/vi/wtPPx0J-4JU/maxresdefault.jpg)
The first point of the answer above is incorrect due to counter example of thin film interference.

The second point is refuted by following counter example.
(https://www.universetoday.com/wp-content/uploads/2011/02/Double-Slit-Experiment.jpg)


Title: Re: why do a lot of people confuse between interference and diffraction?
Post by: hamdani yusuf on 15/10/2019 09:49:27
For the next answer, it seems that they have the label swapped. The blue/left side should be for interference while the red/right side for diffraction.
(https://www.thenakedscientists.com/forum/600x318xproxy.php,qrequest=http,P3A,P2F,P2Fcdn.differencebetween.net,P2Fwp-content,P2Fuploads,P2F2018,P2F02,P2FDiffraction-VERSUS-Interference.jpg,ahash=7640c78dd343c6aaaeefc044d3ca6ce3.pagespeed.ic.5v9qTKoGC4.webp)
After the label swap, the top left point is correct.
The top right point has issue with definition of crack. It excludes knife edge/single edge diffraction.
Quote
a line on the surface of something along which it has split without breaking apart.
"a hairline crack down the middle of the glass"

The middle and bottom points don't really show the difference between interference and diffraction. But the bottom right point is demonstrably false.
Single slit experiment can produce interference pattern even when the slit width is a thousand times the wavelength, e.g. 0.5 mm slit and 500 nm laser pointer.
In usual single slit experiment, following phenomena occur: diffraction, reflection, interference. Interference pattern is produced by superposition of reflected and diffracted light beams by both edges of the slit.



Title: Re: why do a lot of people confuse between interference and diffraction?
Post by: hamdani yusuf on 15/10/2019 10:15:34
(https://www.thenakedscientists.com/forum/xproxy.php,qrequest=http,P3A,P2F,P2Flatestcontents.com,P2Fwp-content,P2Fuploads,P2F2014,P2F06,P2F157.jpg,ahash=551b53f6eea53362dc730fb397f8a067.pagespeed.ic.syqYdtNeE6.webp)
These answers seems to be describing difference between multiple slits (left side) and single slit (right side) experiment instead.

The same thing happens for the answers below
(https://www.brainkart.com/media/extra/GdNAEru.jpg)
(https://www.jagranjosh.com/imported/images/E/Articles/4Effectofslitwidth.png)
(https://image.slidesharecdn.com/ph-101-2-160527092812/95/ph-1012-4-638.jpg?cb=1464341312)

It seems that those confusions are affected by this answer:
(https://www.thenakedscientists.com/forum/600x450xproxy.php,qrequest=http,P3A,P2F,P2Fimages.myshared.ru,P2F17,P2F1174440,P2Fslide_5.jpg,ahash=d9dc908ed13f970bec61f928bdddf59c.pagespeed.ic.pcAYioIKXj.webp)
Even this answer is not consistent with the result of multiple slit experiment
(https://encrypted-tbn0.gstatic.com/images?q=tbn:ANd9GcQocA39-7N4wOh9YX97TExFqYcMtVArBQYn6M9QbnKUA_5xyVH3)
Title: Re: why do a lot of people confuse between interference and diffraction?
Post by: alancalverd on 15/10/2019 12:29:57
Alas, Feynman is wrong! Taking the MW radio navigation case as an example, there is only one source and no interference - the signal strength is always the inverse square of distance to the transmitter with no interference maxima and minima - but substantial angular deflection is possible from terrain and atmosphere.

But the confusion is no more important than speed/velocity. The context reveals all, and a pedantic editor will point out any significant ambiguity.
Title: Re: why do a lot of people confuse between interference and diffraction?
Post by: hamdani yusuf on 16/10/2019 04:07:22
(https://www.differencebetween.com/wp-content/uploads/2012/08/Difference-Between-Diffraction-and-Interference-Tabular-Form.jpg)
The definition of diffraction above is grossly imprecise. For interference, it seems only specific to mechanical waves while also still imprecise.
The principle is almost correct, although the edge doesn't have to be sharp.
The obstacle part is correct. Interference can happen with our without obstacle, e.g. from two independent sources having same frequency.
Title: Re: why do a lot of people confuse between interference and diffraction?
Post by: Colin2B on 16/10/2019 08:23:21
The principle is almost correct, although the edge doesn't have to be sharp.
The definition is very imprecise. Sharp is relative to wavelength, what is sharp for a light wave is not going to diffract a radio wave or sound wave
Title: Re: why do a lot of people confuse between interference and diffraction?
Post by: hamdani yusuf on 16/10/2019 10:14:59
The name interference itself gives it away. Two waves interfere or meet each other. When they do they cause an energy distribution which makes the famous bright and dark bands.
The number of waves is not limited to two in order to produce interference pattern.
Title: Re: why do a lot of people confuse between interference and diffraction?
Post by: hamdani yusuf on 16/10/2019 10:19:08
Quote
Definition of diffraction
: a modification which light undergoes especially in passing by the edges of opaque bodies or through narrow openings and in which the rays appear to be deflected
https://www.merriam-webster.com/dictionary/diffraction

The obstacle doesn't have to be opaque to produce diffraction. Transparent materials will do, as I've shown in my experiment.
Title: Re: why do a lot of people confuse between interference and diffraction?
Post by: hamdani yusuf on 02/12/2019 09:22:06
Another source of confusion I found is inconsistency among sources.
https://en.wikipedia.org/wiki/Diffraction
Quote
In classical physics, the diffraction phenomenon is described by the Huygens–Fresnel principle that treats each point in a propagating wave-front as a collection of individual spherical wavelets.[3] The characteristic banding pattern is most pronounced when a wave from a coherent source (such as a laser) encounters a slit/aperture that is comparable in size to its wavelength, as shown in the inserted image. This is due to the addition, or interference, of different points on the wave-front (or, equivalently, each wavelet) that travel by paths of different lengths to the registering surface. However, if there are multiple, closely spaced openings, a complex pattern of varying intensity can result.
(https://upload.wikimedia.org/wikipedia/commons/thumb/0/0d/Single_Slit_Diffraction.svg/330px-Single_Slit_Diffraction.svg.png)

Quote
The solution provided by the integral theorem for a monochromatic source is:

(https://wikimedia.org/api/rest_v1/media/math/render/svg/7e67edbeca05606b66de2ee168a36cd22599e697)
where U is the complex amplitude of the disturbance at the surface, k is the wavenumber, and s is the distance from P to the surface.

The assumptions made are:

U and ∂U/∂n are discontinuous at the boundaries of the aperture,
the distance to the point source and the dimension of opening S are much greater than λ.
https://en.wikipedia.org/wiki/Kirchhoff%27s_diffraction_formula#Derivation_of_Kirchhoff's_diffraction_formula
Title: Re: why do a lot of people confuse between interference and diffraction?
Post by: hamdani yusuf on 30/12/2020 09:26:12
I answered the same question in quora. Feel free to comment here to improve accuracy and clarity.
https://www.quora.com/What-is-the-difference-between-interference-and-diffraction/answer/Hamdani-Yusuf

I’ve seen many confusions on these concept which I think need to rectify. We need to clarify what are those concepts, and what are not. Some commonly observed phenomena are combinations of those two. I’ll focus the answer on electromagnetic waves, including light.

DIFFRACTION is the bending of light as it passes around the edge of an object with higher opacity than the medium, towards the area/volume behind that object. The amount of bending angle and intensity of bent light depends on the opacity and shape of the object. Opacity and penetration depth depends on the frequency of light as well as the material of the object.

Non-diffractive obstacle shows that diffraction of light is produced by the obstacle, not the medium. It’s corroborated by vertically tilted diffraction slits that produce conic shaped light beam, which makes the pattern on the screen elliptic or hyperbolic.
(https://qph.fs.quoracdn.net/main-qimg-3cd68f9b8b29f60592c1469e9e3a250b)

In single-slit diffraction, the two sides of the slit act as two sources. If the slit is smaller than the wavelength of light, it effectively becomes a single point source, and no observable interference pattern occurs.
(https://qph.fs.quoracdn.net/main-qimg-0b89c1a7d4f8b0a3a3cbbecdb48b8d83)

Not diffraction: Bending of light as it passes from one transparent substance into another. It’s called refraction.
(https://qph.fs.quoracdn.net/main-qimg-72b5e7159cc05c02cc21c1a2f5b70db9)
(https://qph.fs.quoracdn.net/main-qimg-ada54a1c122998a0e6e8f0fc183e2750)

Not diffraction: Bending of light at an interface between two different media so that it returns into the medium from which it originated. It’s called reflection.
(https://qph.fs.quoracdn.net/main-qimg-8441acdbf62bb4d73c6f3ea5ff325087.webp)

INTERFERENCE is a phenomenon in which more than one waves combine to form a resultant wave of greater or lower amplitude which are resolved spatially. It’s because different places receive the original waves in different phases. To produce observable interference pattern, the original light waves need to have the same frequencies and similar polarizations.

Interference is a subset of a more general concept, i.e. superposition, which is simply the combination of more than one wave. Below are some examples of superposition which are not generally called interference.

When the amplitude difference is resolved temporally, it is called BEAT. It is caused by slight difference in frequencies of original waves.
(https://www.animations.physics.unsw.edu.au/jw/images/beats_files/beats.gif)

When the frequency is significantly different, what we’ll get is color mixing/addition. The variable amplitude is unnoticeable.
(https://qph.fs.quoracdn.net/main-qimg-0994f2e51b33ce5beb10a5d502a341c2)

When the polarization axes are different, we don’t get visible destructive interference, although they do superpose.

Single slit diffraction is a combination of diffraction, reflection, and interference between diffracted and reflected light wave by the edge of the slit material.

Thin film interference is a combination of refraction, reflection, and interference. No diffraction is involved. Light dispersion occurs in the same experiment when incoming light waves have more than one frequencies.
(https://qph.fs.quoracdn.net/main-qimg-b3953a4614272da6f7973119bb4bf8aa)

Another example of interference without diffraction is Lloyd’s mirror. The interference pattern is produced by superposition of reflected light with original lightbeam.
(https://qph.fs.quoracdn.net/main-qimg-cd601f711daa362353db56d958b08b82)

Beam splitter can also produce interference without involving diffraction.
(https://qph.fs.quoracdn.net/main-qimg-0fb7dfb77b22a3d3da3475f39b5c269e)

Transmission diffraction grating may involve diffraction when it is made of opaque material. If it’s just a transparent plate with wavy surface, then the phenomena involved are refraction and interference.
(https://qph.fs.quoracdn.net/main-qimg-9717d133e52ba6112de3e9a8faf3734a)

Reflection diffraction gratings don’t involve diffraction. Contrary to the name, they involve reflection and interference instead.
(https://qph.fs.quoracdn.net/main-qimg-dd63ec77d5a7d01501da08cb80db2cc4)
Title: Re: why do a lot of people confuse between interference and diffraction?
Post by: hamdani yusuf on 31/12/2020 03:13:06
Here is another answer I found arguing that Interference and diffraction are really the same phenomenon.
https://www.quora.com/Why-does-interference-happen-during-diffraction
Quote
Tipper Rumpf, PhD Optics, University of Central Florida (2006)
Answered April 7, 2018
Interference and diffraction are really the same phenomenon so the question is a little strange. They have slightly different connotations. Diffraction usually describes the spreading or splitting of a wave. Interference describes when there are two or move waves passing through the same location and the waves add or subtract (i.e. constructive or destructive interference).

Perhaps an answer to your question is that diffraction produces waves at different angles and when these overlap you get interference.

Here is another one,
Quote
Bill Otto, studied Physics & Chemistry at The University of Alabama in Huntsville (1976)
Answered April 13, 2018
Diffraction and interference are the same phenomena, and there is no clear distinction.

Interference is usually used to characterize the effects of a very small number of slits or beams, while diffraction is used to characterize a large number of slits (such as a grating) or a large area such as the diffraction from from a telescope mirror.

That said, there is no clear distinction between the two, and a discussion of why one is present with the other is moot, unless perhaps in your textbook the terms have been defined differently from the usual definitions.

From The Feyman Lectures[1]

the name has been changed from Interference to Diffraction. No one has ever been able to define the difference between interference and diffraction satisfactorily. It is just a question of usage, and there is no specific, important physical difference between them. The best we can do, roughly speaking, is to say that when there are only a few sources, say two, interfering, then the result is usually called interference, but if there is a large number of them, it seems that the word diffraction is more often used. So, we shall not worry about whether it is interference or diffraction, but continue directly from where we left off in the middle of the subject in the last chapter.
https://www.feynmanlectures.caltech.edu/I_30.html
Title: Re: why do a lot of people confuse between interference and diffraction?
Post by: hamdani yusuf on 31/12/2020 03:50:52
Quote
Jess H. Brewer, Physics professor since 1977.
Answered May 11, 2018
They are pretty much the same thing. The way we calculate single-slit diffraction is by breaking the slit up into an infinite number of infinitely small slits right next to each other and calculating the resultant interference pattern.

468 viewsView Upvoters · Answer requested by Bashir Ahme
Quote
Vaibhav Sharma, Physics PhD Student at Cornell University
Answered February 3, 2014
Originally Answered: what is the diffrence between interference and diffraction ?
In reality, there is no difference. According to Huygen's principle, each point is a source of a secondary wavelet. In both the phenomena, secondary wavelets interact and build up a pattern of bright and dark fringes. In interference, we consider the case where there are 2 or multiple slits and where the size or the aperture of the slit is ignored and only the distance between the two slits is taken into account. In diffraction, there is only one slit and thus it's size or aperture is taken into account.

For interference, we consider mainly the interaction among wavelets coming from the two different slits by ignoring the size of the slit and assuming that only one wavelet is emitted from each slit. For diffraction, we consider the interaction among the different wavelets coming from the different portions of the same slit by accepting the fact that the slit is not just a point source but it has a finite aperture with many points. In reality, both diffraction and interference take place simultaneously.

It seems that the confusion has been widely spread even among academics. It may happened because experiments most frequently introduced to demonstrate those phenomena involve both of them simultaneously, such as single slit and double slit experiments. Showing experiments which demonstrate each phenomenon independently will help a lot in recognizing the difference.

Title: Re: why do a lot of people confuse between interference and diffraction?
Post by: hamdani yusuf on 25/01/2021 04:35:11
A answered another question in Quora, Why are there dark fringes in a single slit experiment?
https://www.quora.com/Why-are-there-dark-fringes-in-a-single-slit-experiment

Because in a single slit apperture, there are two edges. They can act as light sources which then interfere constructively and destructively to create dark and bright fringes on a screen. Double slit apperture has 4 edges, hence there are more light sources which can interfere with each other.

(https://qph.fs.quoracdn.net/main-qimg-dc85396cec36db5b1b5a9d84e6debb80)
Top picture is the pattern produced by the same double slit aperture as the bottom picture, but one slit is covered.

According to Babinet principle, single slit apperture and a thin wire produce similar interference patterns.
(https://qph.fs.quoracdn.net/main-qimg-e16e1e47ce8a76e113029ddfea2730cc)(https://qph.fs.quoracdn.net/main-qimg-e5ac27bc205544b408d1966f240c1d80)(https://qph.fs.quoracdn.net/main-qimg-e926c372f4fb3f34a6abedb106c91346)
If you think that single slit interference pattern is produced by the space between edges of the slit which act as wave sources, you’ll be in trouble explaining the same pattern produced by a thin wire.
Title: Re: why do a lot of people confuse between interference and diffraction?
Post by: hamdani yusuf on 05/07/2021 11:09:12
I've compiled the important points to distinguish diffraction from interference in a Youtube video. It is un-shortened version from the one I posted previously to participate in #VeritasiumContest. Without time constraint, I hope it's more bearable to watch and learn from.
Title: Re: why do a lot of people confuse between interference and diffraction?
Post by: Eternal Student on 05/07/2021 13:56:12
Hi Hamdani.  I hope you are well.

I saw the original short video you put in the Just chat thread and made comments there.
   I will watch this longer video and comment again but the situation or the nature of this thread is getting a bit marginal now.  To state the situation directly:  If you get enough views then you get paid for that video.  This is a grey area, since a line has to be drawn to stop people advertising their own products on this forum.
   I am assuming that you are seeking some feedback and general discussion.

1.    As before, the narrator is a digitised voice and is hard to listen to.   Actually, I'd prefer to just read the information, the narrator is adding nothing, except possibly increasing accessibility for people who are partially sighted.  However, you could just spend the time to fill out the subtitles properly and then they can use their own software to read this out in a voice of their own choosing.

2.   The video is a little easier to follow and doesn't leave the viewer feeling quite as uncomfortable and panicked as before.  This is because the narrator is at least reading all the text and not just snippets of it.  This is definitely an improvement.

3.   There is no use of animation, which is OK since it's not always needed.  However, there is nothing in this video that actually required  animation or audio.   There's hardly enough time to even study the diagrams and some of them weren't required anyway.  You could have produced the same information and diagrams in a book or static web page.  As mentioned earlier, I think this would actually be better since the viewer can then set their own pace rather than be rushed along to the next section before they were ready or bored with something they already knew.  My teacher training course would have said this (the video production) was an inappropriate and inefficient use of media.

4.  The video is non-interactive and does not engage the audience at all.  We're being given a talking to by a boring voice.  A better video will slow down and ask question - possibly giving the audience a moment to think about what they might expect to see as the result of an experiment  etc.  In general, the audience should be actively engaged if you want them learn better.

5.   Overall there is a lot of improvement here over your short version.  However, you may have demonstrated that this topic cannot be presented as an interesting video in just 1 minute, you can't even fit it into 2 minutes.  If you're still going after that competition then perhaps you have to consider another topic is required and the limitation of having only 1 minute must feature heavily in your choice of topic.

6.    There are some factual errors in your information, although they are minor and probably the result of rushing the delivery of information.   For example, you said something like "interference requires waves with the same frequency".
Title: Re: why do a lot of people confuse between interference and diffraction?
Post by: hamdani yusuf on 05/07/2021 14:26:50
Thanks for you feed back.
I'll reply the points specifically about the competition in the other thread.

6.    There are some factual errors in your information, although they are minor and probably the result of rushing the delivery of information.   For example, you said something like "interference requires waves with the same frequency".
Can you show me interference from waves with different frequency?
Title: Re: why do a lot of people confuse between interference and diffraction?
Post by: Eternal Student on 05/07/2021 14:49:25
Beats.

Sound waves with almost the same frequency combine to produce a wave with a discernible amplitude modulation at fixed spatial location.

https://en.wikipedia.org/wiki/Beat_(acoustics)

You mentioned beats for 10 seconds in the video so you should know about them.

Any two sound waves (let's say travelling in opposing directions) will interfere, it doesn't matter how different their frequencies are.   It's just that we only notice stable effects easily when the frequencies are the same or very similar.
Title: Re: why do a lot of people confuse between interference and diffraction?
Post by: hamdani yusuf on 05/07/2021 22:42:45
Beats.

Sound waves with almost the same frequency combine to produce a wave with a discernible amplitude modulation at fixed spatial location.

https://en.wikipedia.org/wiki/Beat_(acoustics)

You mentioned beats for 10 seconds in the video so you should know about them.

Any two sound waves (let's say travelling in opposing directions) will interfere, it doesn't matter how different their frequencies are.   It's just that we only notice stable effects easily when the frequencies are the same or very similar.
I mentioned in the video that beat is a form of superposition which is not called interference. Otherwise, the names would be redundant. 

Even diffraction, reflection, and refraction are effects of superposition.  None of them are called interference by themselves.
Title: Re: why do a lot of people confuse between interference and diffraction?
Post by: Eternal Student on 05/07/2021 23:41:22
Hi hamdani,

   I know what you're saying but sadly these are called "interference effects" by many Physicists and we can't just change the terminology:

Examples of definitions and usage of the phrase "interference":
Wave interference is the phenomenon that occurs when two waves meet while traveling along the same medium.
   Taken from  https://www.physicsclassroom.com/class/sound/Lesson-3/Interference-and-Beats

In acoustics, a beat is an interference pattern between two sounds of slightly different frequencies, perceived as a periodic variation in volume whose rate is the difference of the two frequencies.
     Taken from  https://en.wikipedia.org/wiki/Beat_(acoustics)

  ... and many more exist...

Also, while we're here discussing the issue:  It is possible to explain diffraction around sharp edges using Huygens wavelet's Principle   -  and then it is just like interference.   See, for example   https://en.wikipedia.org/wiki/Huygens%E2%80%93Fresnel_principle.

Perhaps, if we return to the original title of your thread - people confuse diffraction and interference because they are actually all a consequence of superposition under certain assumptions and models.
Title: Re: why do a lot of people confuse between interference and diffraction?
Post by: Eternal Student on 06/07/2021 00:08:09
Hi again.

I think this might be a better way to phrase something:

At school level, there can be a clear distinction between diffraction and interference.   You can find references like the ones you (Hamdani) have posted already that will compare the differences.

At university level, the distinction becomes blurred.

What this means is that your video, which aims to show the two are completely different things, may be helping some people to understand initially  but  causing them problems and becoming something they need to un-learn later if they study physics at a higher level.   This isn't necessarily too much to worry about, that's what school physics teaching is often about.  Sometimes your teacher will admit that they are only showing students the simpler models and giving them definitions that are stipulated on the school examination syllabus.
Title: -
Post by: VanessaOweDa on 06/07/2021 03:19:10
One thing I dont understand is that speed is defined as the absolute value of velocity,
but speed distance /time and velocity displacement / time have different terms in the numerator, havent they?

Please could someone clarify. Thanks.
Title: Re: why do a lot of people confuse between interference and diffraction?
Post by: hamdani yusuf on 06/07/2021 06:07:36
I know what you're saying but sadly these are called "interference effects" by many Physicists and we can't just change the terminology:
At least we can suggest or persuade them to be more consistent and stop using confusing terminology.
Perhaps, if we return to the original title of your thread - people confuse diffraction and interference because they are actually all a consequence of superposition under certain assumptions and models.
Agreed.
Title: Re: -
Post by: Eternal Student on 06/07/2021 10:19:52
One thing I dont understand is that speed is defined as the absolute value of velocity,
but speed distance /time and velocity displacement / time have different terms in the numerator, havent they?

Please could someone clarify. Thanks.

Hi Vanessa,

    You are displaying an email address or ICQ contact.  This worries me a bit.  Are you sure you want to do this?
Your question can certainly be answered but could I ask you to start a new thread for that question please.  It does not relate to the main topic in this thread unless you are suggesting that Hamdani should make this a topic for his video.  Are you suggesting Hamdani should do that?
      Why am I being a little evasive?   You're quite new here and displaying contact details.  I'm suspicious I could waste an hour of my time making replies while your primary intention was just to advertise or spam the site anyway.
Title: Re: why do a lot of people confuse between interference and diffraction?
Post by: hamdani yusuf on 06/07/2021 22:44:23
There's a reason why spatially resolved superposition aka interference is not commonly mentioned in the case of sound. Naturally, humans can only sense sound at two points at once, using both ears. It's in contrast with eyes with many light sensitive cells in retina, which can detect many points of light at once.
The lack of easily observed example may had motivated educators in the past to look for the closest analogy for interference of light in sound wave. They started to call beat, which is temporally resolved superposition of sound, as an example of interference too.
Title: Re: why do a lot of people confuse between interference and diffraction?
Post by: TommyJ on 30/07/2021 14:17:01
Confusion really comes from different formulation, demonstration examples/experiments (light, optic, sound, sea waves) and practical implementations.
Interference - is  waves superposition, resulting in altering amplitude. These may be different sources or instances of the same source reflections.
Diffraction - is banding of a wave around an obstacle or opening (a wave here is either a 'pure' single source or resulting superposed waves).
Title: Re: why do a lot of people confuse between interference and diffraction?
Post by: alancalverd on 30/07/2021 15:36:53
One thing I dont understand is that speed is defined as the absolute value of velocity,
but speed distance /time and velocity displacement / time have different terms in the numerator, havent they?

Please could someone clarify. Thanks.
The term "absolute value" is misleading and unhelpful.

Velocity is a vector: it has magnitude and direction
Speed is a scalar: it has magnitude only

Acceleration is the rate of change of velocity, so an object can accelerate at constant speed by changing direction.
Title: Re: why do a lot of people confuse between interference and diffraction?
Post by: hamdani yusuf on 24/08/2021 10:14:12
Clickbait is Unreasonably Effective
Quote
The title and thumbnail play a huge role in a video's success or failure.
Maybe I can try his advice to crank up the number of views of my video.
I have several options, such as
"Simple physics concepts many people get confused"
or a more bombastic one
"Simple physics concepts even physics professors got wrong"
I've compiled the important points to distinguish diffraction from interference in a Youtube video. It is un-shortened version from the one I posted previously to participate in #VeritasiumContest. Without time constraint, I hope it's more bearable to watch and learn from.
Title: Re: why do a lot of people confuse between interference and diffraction?
Post by: emelymorris on 26/08/2021 14:55:41
Diffraction is inextricably linked with the phenomenon of interference. Moreover, the very phenomenon of diffraction is often interpreted as a case of interference of waves limited in space (interference of secondary waves). A common property of all diffraction phenomena is the dependence of the degree of its manifestation on the ratio between the wavelength λ and the size of the wavefront width d, or an opaque screen in the path of its propagation, or inhomogeneities in the structure of the wave itself.
Title: Re: why do a lot of people confuse between interference and diffraction?
Post by: Eternal Student on 26/08/2021 18:41:18
Hi.

Diffraction is inextricably linked with the phenomenon of interference. Moreover, the very phenomenon of diffraction is often interpreted as a case of interference of waves limited in space (interference of secondary waves).
    Sounds like Huygens principle.   I think this was mentioned a long time ago - but I know it is impossible to read all the previous posts.   Anyway, emelymorris you seem fairly new here -  welcome and I hope all is well with you.
Title: Re: why do a lot of people confuse between interference and diffraction?
Post by: hamdani yusuf on 27/08/2021 02:44:01
Diffraction is inextricably linked with the phenomenon of interference. Moreover, the very phenomenon of diffraction is often interpreted as a case of interference of waves limited in space (interference of secondary waves). A common property of all diffraction phenomena is the dependence of the degree of its manifestation on the ratio between the wavelength λ and the size of the wavefront width d, or an opaque screen in the path of its propagation, or inhomogeneities in the structure of the wave itself.
It seems like you are one of the victims of misinformation. You can produce diffraction effect without involving interference, and vice versa. You can read this thread from the start, or just simply watch my video in post#20.

I've compiled the important points to distinguish diffraction from interference in a Youtube video. It is un-shortened version from the one I posted previously to participate in #VeritasiumContest. Without time constraint, I hope it's more bearable to watch and learn from.
Title: Re: why do a lot of people confuse between interference and diffraction?
Post by: hamdani yusuf on 27/08/2021 06:12:43
At university level, the distinction becomes blurred.
Why should it? 
Two distinct phenomena shouldn't be confused just because they often come together. As long as they can occur separately, the distinction should be kept clear.
On the other hand, if they always come together, and there's no way to separate them, we can say that they are synonymous.
Title: Re: why do a lot of people confuse between interference and diffraction?
Post by: hamdani yusuf on 27/08/2021 06:39:58
I've also made an experiment on diffraction using microwave.
My conclusion so far is that diffraction can occur if the penetration depth of the light through the obstacle is significantly more than the wavelength. In case of microwave on aluminum plate, the penetration depth is much lower than the wavelength. That's why diffraction effect can't be detected.
Title: Re: why do a lot of people confuse between interference and diffraction?
Post by: alancalverd on 27/08/2021 07:37:46
Very neat experiment!

What do you think is happening at 1:24?

What wavelength are you using?
Title: Re: why do a lot of people confuse between interference and diffraction?
Post by: hamdani yusuf on 27/08/2021 11:49:28
Very neat experiment!

What do you think is happening at 1:24?

What wavelength are you using?
Thanks.
I think it's caused by reflection. It's not counted as diffraction because the microwave doesn't go behind the obstacle in the line of sight from the transmitter.

The transmitter is 10.5 GHz, so the wavelength is around 28.6 mm
Title: Re: why do a lot of people confuse between interference and diffraction?
Post by: hamdani yusuf on 28/08/2021 02:20:22
This is my earlier video showing that diffraction can occur on the edge of transparent materials.
It even occurs when the refractive index is very similar to the medium.
Title: Re: why do a lot of people confuse between interference and diffraction?
Post by: hamdani yusuf on 28/08/2021 03:31:26
My conclusion so far is that diffraction can occur if the penetration depth of the light through the obstacle is significantly more than the wavelength. In case of microwave on aluminum plate, the penetration depth is much lower than the wavelength. That's why diffraction effect can't be detected.
I don't know any material which has penetration depth in visible spectrum much less than 500 nm. So I used another method to produce impenetrable barrier for visible light, using total internal reflection in glass.


As shown in the video, it doesn't produce diffraction effect.
Title: Re: why do a lot of people confuse between interference and diffraction?
Post by: hamdani yusuf on 28/08/2021 03:47:12
With two total internal reflection edges, we can effectively produce a single slit aperture of impenetrable barrier. It produces no interference pattern.
Title: Re: why do a lot of people confuse between interference and diffraction?
Post by: hamdani yusuf on 28/08/2021 04:18:53
I'm preparing a new video investigating diffraction of light by producing one sided interference pattern. One edge of the single slit aperture is made of total internal reflection barrier, while the other edge is made of folded aluminium foil as a normal barrier.
(https://www.thenakedscientists.com/forum/index.php?action=dlattach;topic=77918.0;attach=32300)
Title: Re: why do a lot of people confuse between interference and diffraction?
Post by: hamdani yusuf on 28/08/2021 13:30:34
If my experiments posted in this thread don't convince people that diffraction and interference pattern formed in single slit experiment come from the edges instead of the space between them,  I don't know what will.
Title: Re: why do a lot of people confuse between interference and diffraction?
Post by: alancalverd on 28/08/2021 23:05:08
I think you have misinterpreted your excellent microwave experiments.

Single-edge diffraction produces a single bright peak outside the shadow of the edge - see  https://sciencedemonstrations.fas.harvard.edu/presentations/edge-diffraction - and I think this is what you are seeing at 1:24. There is a secondary, much smaller peak around 1:25.

Things are a bit different with the partial absorber. The diffracted wave interferes with scattered radiation through the plate, and produces a bright peak inside the shadow area

Likewise the  paraffin wax block acts as a single edge and a scatter source. Bringing up the second block creates a second edge so you now have the beginning of a diffraction grating, which will produce bigger maxima and minima according to the grating equation mλ = d sinθ where d is the grating spacing, m is an integer, and in your case θ ≈ 30°. Note how you get a maximum when the edges area about λ/2 apart, and a minimum when they overlap by about λ. 
Title: Re: why do a lot of people confuse between interference and diffraction?
Post by: evan_au on 28/08/2021 23:16:55
Quote from: hamdani yusuf
If my experiments posted in this thread don't convince people that diffraction and interference pattern formed in single slit experiment come from the edges instead of the space between them,  I don't know what will.
The distance between the edges is defined by the space between them.
See: https://en.wikipedia.org/wiki/Negative_space
Title: Re: why do a lot of people confuse between interference and diffraction?
Post by: hamdani yusuf on 29/08/2021 03:13:43
Things are a bit different with the partial absorber. The diffracted wave interferes with scattered radiation through the plate, and produces a bright peak inside the shadow area
The reflector and partial reflector came with the microwave transceiver kit as accessories. They don't absorb much of the microwave. Most of it is either reflected or transmitted.
Title: Re: why do a lot of people confuse between interference and diffraction?
Post by: hamdani yusuf on 29/08/2021 04:58:09
Likewise the  paraffin wax block acts as a single edge and a scatter source. Bringing up the second block creates a second edge so you now have the beginning of a diffraction grating, which will produce bigger maxima and minima according to the grating equation mλ = d sinθ where d is the grating spacing, m is an integer, and in your case θ ≈ 30°. Note how you get a maximum when the edges area about λ/2 apart, and a minimum when they overlap by about λ.
I interpret the second block as additional source of diffracted wave. The optical distance to the receiver is almost equal with the first edge, thus produced constructive interference.
Title: Re: why do a lot of people confuse between interference and diffraction?
Post by: hamdani yusuf on 29/08/2021 05:52:14
Quote from: hamdani yusuf
If my experiments posted in this thread don't convince people that diffraction and interference pattern formed in single slit experiment come from the edges instead of the space between them,  I don't know what will.
The distance between the edges is defined by the space between them.
See: https://en.wikipedia.org/wiki/Negative_space
I have three types of single slit experiments.
1. Penetrable barriers are used to create the edges. Interference pattern shows on both sides. This is the type usually used in physics demonstrations as teaching materials.
2. Impenetrable barriers are used to create the edges. No interference pattern shows on either side.
3. One edge is penetrable barrier, while the other is impenetrable. Only one side shows interference pattern. The side behind the impenetrable barrier doesn't show interference effect.

The width of the slit is almost the  same in all of those cases. But the results are different.
Title: Re: why do a lot of people confuse between interference and diffraction?
Post by: alancalverd on 29/08/2021 11:44:27
And I forgot to mention refraction! Paraffin wax doesn't just absorb and diffract a bit of the radiation, it also bends it.

https://news.mit.edu/2012/new-metamaterial-lens-focuses-radio-waves-1114 describes an interesting microwave lens that exploits the negative refractive index of a conductive metamaterial. This explains why the peaks of your metal-edge diffraction pattern appear on the "wrong" side of the beam central plane.

https://phyweb.physics.nus.edu.sg/~L3000/Level3manuals/Microwave.pdf  discusses a number of microwave optical experiments. Definitely worth remembering that your source is strongly polarised so the effect of an asymmetric slit, lens or whatever will depend on its orientation.
Title: Re: why do a lot of people confuse between interference and diffraction?
Post by: hamdani yusuf on 29/08/2021 13:14:35
And I forgot to mention refraction! Paraffin wax doesn't just absorb and diffract a bit of the radiation, it also bends it.
Where does it bend to on a flat surface?
Title: Re: why do a lot of people confuse between interference and diffraction?
Post by: hamdani yusuf on 29/08/2021 13:20:49
https://news.mit.edu/2012/new-metamaterial-lens-focuses-radio-waves-1114 describes an interesting microwave lens that exploits the negative refractive index of a conductive metamaterial. This explains why the peaks of your metal-edge diffraction pattern appear on the "wrong" side of the beam central plane.
I used a plain aluminium plate. Noone would call it a metamaterial.
Title: Re: why do a lot of people confuse between interference and diffraction?
Post by: alancalverd on 29/08/2021 15:40:31
You can see the bending of light passing through a flat surface with your glass blocks. Obviously there is no displacement if the beam is perpendicular to the surface, but in the case of microwaves, your source was delivering a cone of radiation so the first encounter with the  paraffin wax was at an angle of around 10 - 15 degrees to perpendicular.

Aluminium isn't a metamaterial but it is an excellent conductor, so one edge can display the properties of a metamaterial in a limited geometry.
Title: Re: why do a lot of people confuse between interference and diffraction?
Post by: hamdani yusuf on 31/08/2021 06:15:28
You can see the bending of light passing through a flat surface with your glass blocks. Obviously there is no displacement if the beam is perpendicular to the surface, but in the case of microwaves, your source was delivering a cone of radiation so the first encounter with the  paraffin wax was at an angle of around 10 - 15 degrees to perpendicular.
Refraction by a dielectric plate with parallel surface doesn't change the direction of light at the end of the process. It's only translated, which depends on the thickness of the plate.
(https://upload.wikimedia.org/wikipedia/commons/thumb/8/85/Refraction_photo.png/220px-Refraction_photo.png)
In my experiment, the plates aren't that thick to produce significant effect of refraction.
Title: Re: why do a lot of people confuse between interference and diffraction?
Post by: hamdani yusuf on 31/08/2021 06:23:53
Aluminium isn't a metamaterial but it is an excellent conductor, so one edge can display the properties of a metamaterial in a limited geometry.
Or it's just a simple reflection by a convex surface located at the edge of the aluminum plate.
Title: Re: why do a lot of people confuse between interference and diffraction?
Post by: alancalverd on 31/08/2021 23:10:17
Aluminium isn't a metamaterial but it is an excellent conductor, so one edge can display the properties of a metamaterial in a limited geometry.
Or it's just a simple reflection by a convex surface located at the edge of the aluminum plate.
Unlikely to be significant - the wavelength is around 50 times the radius of any possible convexity! But the phenomenon you demonstrated is exactly what is predicted by a wavelet model of diffraction.
Title: Re: why do a lot of people confuse between interference and diffraction?
Post by: hamdani yusuf on 01/09/2021 09:11:43
Unlikely to be significant - the wavelength is around 50 times the radius of any possible convexity! But the phenomenon you demonstrated is exactly what is predicted by a wavelet model of diffraction.
Why not? A radio transmitter antenna can transmit radio wave with wavelength a thousand times its diameter.
The aluminum plate doesn't produce diffracted microwave, unlike paraffin wax plate.
Title: Re: why do a lot of people confuse between interference and diffraction?
Post by: alancalverd on 01/09/2021 09:37:02
I must repeat what I said in #46 above, with a bit of emphasis

Single-edge diffraction produces a single bright peak outside the shadow of the edge - see  https://sciencedemonstrations.fas.harvard.edu/presentations/edge-diffraction - and I think this is what you are seeing at 1:24. There is a secondary, much smaller peak around 1:25.

Aluminum is a conductor, paraffin wax is a dielectric, so you wouldn't expect them to behave the same.
Title: Re: why do a lot of people confuse between interference and diffraction?
Post by: hamdani yusuf on 01/09/2021 11:01:57
Aluminum is a conductor, paraffin wax is a dielectric, so you wouldn't expect them to behave the same.
Aluminum produces diffraction in visible light spectra, just like paraffin wax.

I'm preparing a new video investigating diffraction of light by producing one sided interference pattern. One edge of the single slit aperture is made of total internal reflection barrier, while the other edge is made of folded aluminium foil as a normal barrier.
(https://www.thenakedscientists.com/forum/index.php?action=dlattach;topic=77918.0;attach=32300)
Title: Re: why do a lot of people confuse between interference and diffraction?
Post by: hamdani yusuf on 26/12/2022 12:09:47
It's been 3 years since I started this thread. But still, the confusion doesn't seem to go away.
Here's what I got from Google search just now.
Quote
Interference is a property originated by waves from two different coherent sources, whereas secondary wavelets that originate from the same wave but occur from different parts of it, produce the phenomenon termed diffraction.
And the next top searches are:
Quote
https://dewwool.com/difference-between-diffraction-and-interference/
Diffraction and interference are phenomenons associated with the wave nature of particles. Diffraction can be plainly defined as the spreading of waves while passing through obstacles. Interference can be defined as the combination of two or more waves. In this article, we discuss 10 differences between diffraction and interference.

Quote
https://collegedunia.com/exams/difference-between-diffraction-and-interference-physics-articleid-1566
Interference is a feature caused by waves from two independent coherent sources, whereas Diffraction is caused by secondary wavelets that originate from the same wave but occur in various areas of it. It's essential to understand the fundamental differences between them by looking at the region of least intensity; in interference, this region is extremely dark, but in diffraction, it's less dark. There are a few other criteria that distinguish diffraction from interference, in addition to these few distinctions which are tabulated below.
Quote
https://www.toppr.com/guides/physics/difference-between/diffraction-and-interference/
Important difference between diffraction and interference

Parameter   |   Diffraction   |   Interference
Occurrence   |   It occurs because of the secondary wavelengths superposition   |   It occurs due to the light waves superposition that is from two sources
Width of fringes   |   Unequal   |   Equal
Intensity of fringe   |   Not same, in case of diffraction, for all fringes   |   Same fringe intensity for all the fringes
Obstacle or slit   |   There is a requirement for it   |   There is not a requirement for it
Fringe spacing   |   Non-uniform in the case of diffraction   |   Uniform in the case of interference
Contrast between maxima and minima   |   The contrast between maxima and minima is poor   |   The contrast between maxima and minima is certainly good
Wave propagation direction   |   It changes after diffraction   |   It does not change after superposition
Quote
https://www.vedantu.com/physics/difference-between-diffraction-and-interference
Interference

Interference is the phenomenon that takes place when the meeting of the two waves takes place as they travel along with the same medium. Besides this, the interference causes the medium to take a particular shape or orientation. Moreover, this shape is due to the whole or the net effect of two individual waves on the medium’s particles.

Diffraction
Diffraction happens due to the spreading out of waves passing through an aperture. In the case of diffraction, the size of the obstacle or aperture is of similar straight dimensions to the incident wave’s wavelength, and its occurrence is significant. Furthermore, it takes place when the traveling wavelength’s part gets obscured or shaded.
Title: Re: why do a lot of people confuse between interference and diffraction?
Post by: alancalverd on 26/12/2022 19:15:44
You have chosen some very bad sources. The English is poor and the physics is awful.
Title: Re: why do a lot of people confuse between interference and diffraction?
Post by: hamdani yusuf on 27/12/2022 02:54:54
You have chosen some very bad sources. The English is poor and the physics is awful.
Nevertheless, they are chosen by Google's algorithm.
Some say that AI is simply statistic in steroid. Whatever repeated most often, will come up as most significant factor in final results. I hope this issue catches the eyes of popular science communicators, so the vicious circle of spreading misconceptions can eventually be stopped.
Title: Re: why do a lot of people confuse between interference and diffraction?
Post by: Eternal Student on 27/12/2022 07:18:58
Hi.
It's been 3 years since I started this thread. But still, the confusion doesn't seem to go away.  Here's what I got from Google search just now.

    Are you really surprised by this?   You might be seriously over-estimating what a thread in this forum can do or should be trying to do.

    Google (and people in general) don't usually check this small science forum website to get a definition of what the terms "interference" and "diffraction" should be or to obtain an explanation for the subtle differences between them.   
   I'm not criticising your thread or your ideas at all, actually it's quite interesting and shows you have spent a lot of time thinking over some stuff.    However, if there was any scientific term I wanted a definition for,  let's say the "Schwarzschild radius", then I wouldn't start by looking at what some members on a small science forum had to say about it.

    None-the-less, if you are trying to change the use of these terms in the wider community then I wish you the best of luck.   
    At some point in the thread you might have told us why you feel the terms need to be kept so separate but I can't find that on scanning through the thread.  Perhaps you could tell us again.   In particular, is there a change to any physics that follows from something or is it just that you dislike two terms being used inconsistently or interchangeably in some situations?

Best Wishes.
Title: Re: why do a lot of people confuse between interference and diffraction?
Post by: alancalverd on 27/12/2022 09:37:29
Nevertheless, they are chosen by Google's algorithm.
There's the problem.
Google rates the most-viewed sites highest. The English idioms in the passages you quoted are distinctly Indian and appear to have originated in a text aimed at schoolkids - possibly an online classroom. So it will have had an enormous number of readers and no real editing . One obvious giveaway was the use of "medium" - electromagnetic waves can interfere in vacuo!

IMHO school science texts are not edited by scientists but by "communicators" who insist on replacing my three old chums A, B, and C with ethnically balanced gender-neutral names and don't give a damn about physics.

Far better to turn to Wikipedia, which is effectively peer-reviewed and refined by people who know what they are talking about.

However you have clearly demonstrated one important problem: beware of any easy consensus based on poorly-stated or poorly-understood science!
Title: Re: why do a lot of people confuse between interference and diffraction?
Post by: alancalverd on 27/12/2022 09:51:59
At some point in the thread you might have told us why you feel the terms need to be kept so separate but I can't find that on scanning through the thread.  Perhaps you could tell us again.

Diffraction occurs at an edge or a point and results in a downstream wavefront with the same wavelength as the original source, appearing to originate from that edge/point.

Interference is the result of superposition of two or more wavefronts producing maxima and minima with a spatial distribution related to the distribution of the sources. 

If you stand in the shadow of a mountain you may hear the radio signal of a station that is just out of your line of sight. The signal strength you receive decreases smoothly as you move away from the station or further "behind" the mountain. Diffraction. This can cause serious radio navigation errors.

If you have two stations transmitting  the same signal, the intensity you receive will vary sinusoidally as you move in any direction  relative to them. Interference. This was used for very precise long range radio navigation systems such as "Knickebeine" and "LORAN".
Title: Re: why do a lot of people confuse between interference and diffraction?
Post by: Eternal Student on 27/12/2022 18:24:54
Hi.

    Yes, OK, it's possible to try and keep diffraction and interference separate.
However,   is it necessary to try and do so    and    is it accidentally concealing the possibility that they are both due to the same underlying phenomena?

    Let's take an example and consider Huygens principle of secondary wavelets.   It's not the best ever explanation of how or why em radiation travels and exhibits certain phenomena,  for example you can always ask why the secondary wavelets only travel forward,  however it is one reasonable model that can be used.
    Let's just assume that something like Huygens principle is happening.   According to many references (e.g. see any reference provided below), the continued propagation of a plane wave or spherical wave can be explained just by using Huygens principle.   Specifically, once a wavefront has travelled away from it's original source (any tiny distance) then it is possible to forget about that source entirely.   The next position of the wavefront at a later time, t+δt, can be determined just by considering every point on the wavefront at time t to act as a new spherical source.   This is an important point so it's worth saying it again in a slightly different way...
    Nature does not need to know or be concerned about what the original source of the wave might have been.   It is not trying to keep the wave progressing or propagating away from that original source.  As far as Nature is concerned there is now a set of sources (one new source at every point on the wavefront) and the new position of the wavefront will be determined just by propagating each of those spherical waves away from those sources.
   A simple version of Huygens principle is provided by reference [2],  where it is stated that the new wavefront would be on the surface which is tangential to all the spherical wavelets.   Other references (e.g. [3] ) make the situation more easily expressed by referring to "the envelope" of the spherical wavelets but this is the same surface.  I think - but I'm not certain - that the original statement by Christiaan Huygens used the concept of an envelope but it probably wasn't in English and the translation is subject to some interpretation.
   Now, most versions of Huygens principle suggest that the secondary sources do behave like genuine sources of a new wave in most respects.   In particular there will be some superposition and interference.   It is very possible that the placement of the new wavefront on the envelope of the set of spherical wavelets is just a simplification of the underlying process that is actually happening.  Examples:
a)    From reference [1]:   ...every point on a wavefront is itself the source of spherical wavelets, and the secondary wavelets emanating from different points mutually interfere. The sum of these spherical wavelets forms a new wavefront.
b)    From reference [4]:   Wave propagation is linear so superposition holds: it should be possible to decompose an impulsive propagating wave front into its constituent points, then consider the impulsive wavelets radiating from each of those points at a future time, and combine those wavelets in a simple direct geometric manner to obtain the progressing wave front at that future time.

    I have included reference [4] just because it's one of the few articles I've seen where a very direct attempt is made to remove the notion of placing the new wave front "on the envelope" and there is an attempt to show that the new wavefront could just be a result of something like conventional superposition and interference.   There is a different type of wave generated by Huygens secondary sources (a Dirac delta function) but with that assumption you no longer need to exclude a backward propagation, there is just an automatic cancellation when a sum of all the waves is computed.  In a similar way, there is no effect often described as a "wake" left behind.   What remains from the superposition is precisely the same as putting the new wavefront right on the envelope (i.e. a lot of the problems and apparently arbitary rules for Huygens principle have been resolved).
   Anyway the key issue is that what is seen as a conventional straight line or spherical propagation of a wave could actually be the result of interference from many secondary wave sources on the wavefront.

Let's take this point mentioned in one of the much earlier posts:
I don't think that diffraction is a subset of interference. We can produce diffraction without interference pattern.

(https://qph.fs.quoracdn.net/main-qimg-3cd68f9b8b29f60592c1469e9e3a250b)
   
   It is quite possible that you are seeing the results of interference.  The propagation of those waves just doesn't look like a typical interference pattern but wherever there is straight line or spherical propagation of a wave then this has been the result of interference from Huygens secondary spherical wavelets.   More-over the "diffraction" effect from the knife edge on the mountain is entirely due to the phenomena of Huygens secondary wavelets.   The radio waves are entering the geometric shadow only because there were no wavelets below them that were able to provide the interference necessary to keep the radio wave progressing in a straight line from the original transmission aerial.

Summary:
   This is NOT an attempt to state that the propagation of an e-m wave has to be this way.  I have provided reference [4] because it's an article in a respected publication ("Nature", see * LATE EDITING) and it simply suggests that Huygens principle and ideas along the lines of conventional superposition and interference could play an important role in all wave propagation phenomena.
   As such what we call "diffraction" could very well be just an "interference" effect.  When light seems to bend around an obstacle, that could just be due to the lack of the interference required to keep it going in a straight line from the original source.  The opening sentence of this post can be re-stated:   Is it necessary to keep the terms "diffraction" and "interference" entirely separate    and    is it accidentally concealing the possibility that they are both due to the same underlying phenomena?


Best Wishes.

References:
    1.    Wikipedia:    https://en.wikipedia.org/wiki/Huygens%E2%80%93Fresnel_principle
    2.   Byjus:    https://byjus.com/physics/the-huygens-principle-and-the-principle-of-a-wave-front/
    3.   University of New Mexico, physics course:   https://physics.unm.edu/Courses/Fields/Phys2310/Lectures/lecture16.pdf
    4.   Nature*:   https://www.nature.com/articles/s41598-021-99049-7

* LATE EDITING:   On closer inspection, reference [4] is properly an article from "Scientific Reports" which is a sub-journal affiliated to "Nature".   It still has a peer review process but the readership and general academic reference value is not the same as the major journal "Nature".
Title: Re: why do a lot of people confuse between interference and diffraction?
Post by: hamdani yusuf on 27/12/2022 21:37:08
is it accidentally concealing the possibility that they are both due to the same underlying phenomena?
they are both due to the same underlying phenomena called superposition.
Title: Re: why do a lot of people confuse between interference and diffraction?
Post by: alancalverd on 27/12/2022 21:44:35
It is important to distinguish between them because they have different causes and different effects, as I pointed out in reply #66.

The diagram from HY's contribution is bizarre as it shows the diffracted signal as having a variable frequency, roughly twice that of the incoming signal! That's a pity because it prevents us seeing interference between the primary and diffracted wavefront which he has actually demonstrated in some of his videos. 

Huygens gives us a mathematical model from which we can predict both phenomena.

Beware of byjus. I have no idea where it originates - the idioms appear to be Indian - but it is generally of poor quality.

The UNM lecture opens with a key statement, the precise details of which are correct and all too frequently overlooked

 
Quote
We are going to talk about a way to visualize wave
propagation that is controversial in its exact physical
interpretation.

.....yes, it is a mathematical model of propagation - nothing more or less. And a very good one, because it also allows us to predict the outcome of interference.

Part of the problem of taking the model too seriously is summarised in the Nature article - you have to invent some physical process for eliminating the backward wave, and this can lead to all sorts of nonsense about virtual dipoles in space, compression of the aether, etc, whereas Maxwell's equations address the physics and only generate the forward vector that actually happens.

Yes, I'm a pedant, but if we don't distinguish between

de-fuse and diffuse (listen to any newsreader)
hypo and hyper
m and M
force, power and energy

we may end up killing someone. And if you can't distinguish diffraction from interference when approaching a coast at night, you can end up killing yourself. 
 


Title: Re: why do a lot of people confuse between interference and diffraction?
Post by: alancalverd on 27/12/2022 21:47:01
they are both due to the same underlying phenomena called superposition.
NO!!! there is no superposition in diffraction, because there is only one wavefront!!!!

That, in a nutshell, is the whole point of the argument!!!
Title: Re: why do a lot of people confuse between interference and diffraction?
Post by: hamdani yusuf on 28/12/2022 02:35:35
At some point in the thread you might have told us why you feel the terms need to be kept so separate but I can't find that on scanning through the thread.  Perhaps you could tell us again.

I've summarized the topic into a short video in another thread.
This is my first video entry. The time limitation is real pain in the process. I did what I can to meet it without losing much of important information.
Let me know what you think.

PS. If someone plays the video in this page, will it count as a Youtube view?
Title: Re: why do a lot of people confuse between interference and diffraction?
Post by: hamdani yusuf on 28/12/2022 02:52:50
Google (and people in general) don't usually check this small science forum website to get a definition of what the terms "interference" and "diffraction" should be or to obtain an explanation for the subtle differences between them.
The differences are conceptual, and they are not by any mean subtle. IMO, the confusion occurred because the most popular experiments to demonstrate those phenomena involve both of them. Phenomena which show them separately don't seem to get much attention, but they are exactly what we need to clear up the confusion.
Here's my unshrink video, I hope you find it easier to follow.
Title: Re: why do a lot of people confuse between interference and diffraction?
Post by: hamdani yusuf on 28/12/2022 07:54:06
they are both due to the same underlying phenomena called superposition.
NO!!! there is no superposition in diffraction, because there is only one wavefront!!!!

That, in a nutshell, is the whole point of the argument!!!
Diffraction does involve superposition, as it requires macroscopic objects which in turn are made of multiple particles. The direction of propagation of resulting wave depends on the position of the particles, as well as the phase of their oscillations. The training video from Royal Canadian Air Force that I've posted earlier shows that clearly.
Title: Re: why do a lot of people confuse between interference and diffraction?
Post by: alancalverd on 28/12/2022 08:14:39
The direction of propagation of resulting wave depends on the position of the particles, as well as the phase of their oscillations.
Poppycock.

The frequency of a radio homing beacon is usually around 400 kHz. How fast do you think the atoms of the white cliffs of Dover are oscillating? Or are you thinking about the blades of grass on top?  Wavelength of 750 m is rather longer than an atom or even a tall stem of wheat. If the phenomenon had anything to do with  interference, the maxima would be 1.5 km apart - utterly useless for navigation.

What you call a "conceptual difference" is what scientists (and parents) call "giving different names to different things".
Title: Re: why do a lot of people confuse between interference and diffraction?
Post by: hamdani yusuf on 28/12/2022 08:53:08
 In particular, is there a change to any physics that follows from something or is it just that you dislike two terms being used inconsistently or interchangeably in some situations?
Inconsistency is the meaning of words may cause miscommunication, and we don't know for sure when will it cause catastrophic events. Miscommunication about measurement units has been recorded in history as the cause of some devastating events.
In educational world, when misconception is widespread among the educators, then students who know the correct concept and give the correct answers in an exam could be jeopardized.
Title: Re: why do a lot of people confuse between interference and diffraction?
Post by: hamdani yusuf on 28/12/2022 08:54:34
The direction of propagation of resulting wave depends on the position of the particles, as well as the phase of their oscillations.
Poppycock.

The frequency of a radio homing beacon is usually around 400 kHz. How fast do you think the atoms of the white cliffs of Dover are oscillating? Or are you thinking about the blades of grass on top?  Wavelength of 750 m is rather longer than an atom or even a tall stem of wheat. If the phenomenon had anything to do with  interference, the maxima would be 1.5 km apart - utterly useless for navigation.

What you call a "conceptual difference" is what scientists (and parents) call "giving different names to different things".
I wrote that interference and diffraction are distinct phenomena, but both are effects of superposition.
Title: Re: why do a lot of people confuse between interference and diffraction?
Post by: evan_au on 28/12/2022 09:16:41
Quote from: hamdani yusuf
(400 kHz homing beacon)...interference and diffraction are distinct phenomena, but both are effects of superposition
The behaviour of radio waves from a homing beacon can be described quite well by Maxwell's equations. This is "classical" physics.
- Superposition derives from quantum theory, and cannot be described by classical physics.
- You don't need superposition to describe radio waves
Title: Re: why do a lot of people confuse between interference and diffraction?
Post by: hamdani yusuf on 28/12/2022 09:50:26
Quote from: hamdani yusuf
(400 kHz homing beacon)...interference and diffraction are distinct phenomena, but both are effects of superposition
The behaviour of radio waves from a homing beacon can be described quite well by Maxwell's equations. This is "classical" physics.
- Superposition derives from quantum theory, and cannot be described by classical physics.
- You don't need superposition to describe radio waves


Superposition principle has been widely used long before quantum theory.
https://en.wikipedia.org/wiki/Superposition_principle
Quote
The superposition principle,[1] also known as superposition property, states that, for all linear systems, the net response caused by two or more stimuli is the sum of the responses that would have been caused by each stimulus individually. So that if input A produces response X and input B produces response Y then input (A + B) produces response (X + Y).

A function F(x) that satisfies the superposition principle is called a linear function. Superposition can be defined by two simpler properties: additivity

(https://wikimedia.org/api/rest_v1/media/math/render/svg/8a5a5e6a5f38e9dea6a9bdc84e03075720332f0f)
and homogeneity
(https://wikimedia.org/api/rest_v1/media/math/render/svg/d21e99513866cbd9a3b233b7d231eb70265d5d42)
for scalar a.
This principle has many applications in physics and engineering because many physical systems can be modeled as linear systems. For example, a beam can be modeled as a linear system where the input stimulus is the load on the beam and the output response is the deflection of the beam. The importance of linear systems is that they are easier to analyze mathematically; there is a large body of mathematical techniques, frequency domain linear transform methods such as Fourier and Laplace transforms, and linear operator theory, that are applicable. Because physical systems are generally only approximately linear, the superposition principle is only an approximation of the true physical behavior.

The superposition principle applies to any linear system, including algebraic equations, linear differential equations, and systems of equations of those forms. The stimuli and responses could be numbers, functions, vectors, vector fields, time-varying signals, or any other object that satisfies certain axioms. Note that when vectors or vector fields are involved, a superposition is interpreted as a vector sum. If the superposition holds, then it automatically also holds for all linear operations applied on these functions (due to definition), such as gradients, differentials or integrals (if they exist).
Title: Re: why do a lot of people confuse between interference and diffraction?
Post by: hamdani yusuf on 28/12/2022 10:08:41
Interestingly, the article also mentions the difference between wave diffraction vs. wave interference
Quote
https://en.wikipedia.org/wiki/Superposition_principle#Wave_diffraction_vs._wave_interference
Wave diffraction vs. wave interference
With regard to wave superposition, Richard Feynman wrote:[2]

No-one has ever been able to define the difference between interference and diffraction satisfactorily. It is just a question of usage, and there is no specific, important physical difference between them. The best we can do, roughly speaking, is to say that when there are only a few sources, say two, interfering, then the result is usually called interference, but if there is a large number of them, it seems that the word diffraction is more often used.

Other authors elaborate:[3]

The difference is one of convenience and convention. If the waves to be superposed originate from a few coherent sources, say, two, the effect is called interference. On the other hand, if the waves to be superposed originate by subdividing a wavefront into infinitesimal coherent wavelets (sources), the effect is called diffraction. That is the difference between the two phenomena is [a matter] of degree only, and basically, they are two limiting cases of superposition effects.

Yet another source concurs:[4]

In as much as the interference fringes observed by Young were the diffraction pattern of the double slit, this chapter [Fraunhofer diffraction] is, therefore, a continuation of Chapter 8 [Interference]. On the other hand, few opticians would regard the Michelson interferometer as an example of diffraction. Some of the important categories of diffraction relate to the interference that accompanies division of the wavefront, so Feynman's observation to some extent reflects the difficulty that we may have in distinguishing division of amplitude and division of wavefront.

The Fyenman's quote above doesn't seem to show his own opinion, but rather his observation on how those words were used by his peers.
Title: Re: why do a lot of people confuse between interference and diffraction?
Post by: hamdani yusuf on 28/12/2022 10:25:49
Interference is the result of superposition of two or more wavefronts producing maxima and minima with a spatial distribution related to the distribution of the sources. 
Yes. The defining characteristics of interference is the occurrence of points in space with cancelling wave, called destructive interference. Wave addition/superposition that doesn't produce "darker spots" isn't usually called interference. In my video, I mentioned color mixing, beat, and superposition of two light waves with perpendicular polarizations as some examples of superposition that doesn't produce interference.
Title: Re: why do a lot of people confuse between interference and diffraction?
Post by: hamdani yusuf on 28/12/2022 10:27:47
What you call a "conceptual difference" is what scientists (and parents) call "giving different names to different things".
Agreed.
Title: Re: why do a lot of people confuse between interference and diffraction?
Post by: alancalverd on 28/12/2022 14:23:09
I wrote that interference and diffraction are distinct phenomena, but both are effects of superposition.
Which is wrong. Superposition requires two sources, by definition. There is only one in diffraction.
Title: Re: why do a lot of people confuse between interference and diffraction?
Post by: alancalverd on 28/12/2022 14:28:45
In my video, I mentioned color mixing, beat, and superposition of two light waves with perpendicular polarizations as some examples of superposition that doesn't produce interference
Color mixing is due to physiology, not physics! If you illuminate a surface with two light beams of different frequencies, the eye may perceive one different color but a spectrometer will show the original frequencies only.

This is different from true frequency modulation or mixing in a nonlinear medium. 

"Beat" is true superposition. When one frequency is much higher than the other, it is called "amplitude modulation".
Title: Re: why do a lot of people confuse between interference and diffraction?
Post by: alancalverd on 28/12/2022 14:35:26
- Superposition derives from quantum theory, and cannot be described by classical physics.
- You don't need superposition to describe radio waves
Beg to differ. The superposition of radio waves can and does cause spatial modulation, entirely calculable from the classical addition of sinusoids. It was the basis of  Knickebeine and LORAN long-range precision navigation. There is no mention of quantum physics in my radio navigation textbooks, though they do talk about relativity in the GPS chapters!
Title: Re: why do a lot of people confuse between interference and diffraction?
Post by: Eternal Student on 28/12/2022 15:49:21
Hi.

   So having a look through all the replies, this is how I'm tempted to summarise the responses:

1.   We do not know exactly what the underlying process for diffraction or interference is.   There are some models.

2.   In those models, especially Huygens principle, it is quite common that diffraction and interference do appear to have similar explanations and could be due to the same underlying principles of physics.

3.  Is it important to distinguish between interference and diffraction because there is some very different underlying physics you (especially Hamdani Yusuf) are trying to present?   Answer:  No.

4.   Is it important to distinguish the difference just for improved communication and consistency between specialists?   Answer:  (You both claim) Yes.   Your main example is that aeroplane pilots and navigators can die otherwise.
   My view:  Sure, in some areas of work there will have to be some very precise use of terminology.  In more general science there is also some benefit in recognising that some apparently disparate things are actually closely related and the same underlying principles could apply.   For example,  Beta particles are not the same as high velocity electrons.  Beta particles can ONLY be produced by changes that take place in the nucleus of an atom.   However, it is much more useful just to recognise that they behave so much like high velocity electrons that they almost certainly are high velocity electrons.

 Best Wishes.
Title: Re: why do a lot of people confuse between interference and diffraction?
Post by: Eternal Student on 28/12/2022 15:59:52
Hi.
   Now just some basic tests to see if everyone really has thought this through:

When monochromatic light passes through a single slit and produces a pattern on a screen.  What do you call that pattern on the screen?
   (a)   A diffraction pattern.    (Note that Feynmann suggested diffraction applies only for multiple sources interacting.  However, light does spread out when passing through a slit, I mean this single slit thing is the text-book example of a diffraction process isn't it?).
   (b)   An interference pattern.   (Can you have interference with one source of light?  Yes you can and you do @alancalverd  whenever that light passes through an aperture).

When monochromatic light passes through two slits and produces a pattern.... What do you call that?
   (a)  A diffraction pattern.
   (b)  An interference pattern.

When ...light  ... passes.... diffraction grating... screen.  What do you call that?
    (a)  A diffraction pattern.   (I mean, it had the name "diffraction" in "diffraction grating" didn't it?)
    (b)  An interference pattern.    (Feynmann suggestion is that you really should be calling this diffraction by now).

   I think common usage of the terms would have the answers   a (for 1 slit),  b (for 2)  then back to a (for diffraction grating).  @hamdani yusuf  asked about school pupils and what they are taught in schools.  I've just spent 20 minutes going through the AQA syllabus for physics at "A" level.  They do describe the patterns with these terms.

   Meanwhile the Feynmann suggestion should give   b, b, a.

    If I've got the gist of what @hamdani yusuf  wants the definitions of diffraction and interference to be.   (One is about bending light, the other includes producing bright and dark bands in a pattern)  then the answers   are  b, b, b.

    My view is... what the figgy pudding does it matter?

Best Wishes.
Title: Re: why do a lot of people confuse between interference and diffraction?
Post by: alancalverd on 28/12/2022 16:17:49
2.   In those models, especially Huygens principle, it is quite common that diffraction and interference do appear to have similar explanations and could be due to the same underlying principles of physics.

3.  Is it important to distinguish between interference and diffraction because there is some very different underlying physics you (especially Hamdani Yusuf) are trying to present?   Answer:  No.
I think not.

Whilst the Huygens wavelet model can be used to predict the downstream waveform for both diffraction and interference, you can't ignore the fact that diffraction requires only a single source and interference requires at least two sources.

Unless 1 ≥ 2, they cannot have the same explanation or be based on the same physics!

There is an additional problem with the wavelet model, to which HY's reference alluded: it predicts an equal and opposite wavefront travelling back towards the source(s), which is not observed. It is a convenient geometric trick but it ain't physics!

And sadly, beta particles are indeed high speed electrons, just as gamma and x-rays are identical. They are named differently on account of their different sources, and again the distinction has safety implications: you can switch off an electron or x-ray source, but not a beta or gamma source!  (Superpedants may object to my ignoring the betatron, but you won't get many bikes out of a cyclotron - sometimes we just have to live with the legacy). 
Title: Re: why do a lot of people confuse between interference and diffraction?
Post by: alancalverd on 28/12/2022 16:58:21
When monochromatic light passes through a single slit and produces a pattern on a screen.  What do you call that pattern on the screen?
If the slit is wide, most of what you see is the unobstructed primary beam with some interference where the edge-diffracted beam has superimposed on the primary beam, and some diffraction outside the slit width.

As the slit gets narrower the diffraction contribution begins to dominate until most of what you see is the interference pattern of both edge-diffracted beams.

Wikipedia discusses this in detail under "Airy disc".

What comes out of an ideal diffraction grating is a diffraction pattern, but most of what passes through most real gratings is primary beam, so you get a convolution of diffraction and interference, like the single slit but with a larger element of  diffraction.  The mathematics of x-ray diffraction from powdered solids and stretched fibers is fascinating and ends up with the structure of DNA, and hemoglobin. I have a delightfully disgusting story about x-ray diffraction which can wait for another day.
Title: Re: why do a lot of people confuse between interference and diffraction?
Post by: Eternal Student on 28/12/2022 18:25:16
Hi.

you can't ignore the fact that diffraction requires only a single source and interference requires at least two sources.
    I'm fairly sure you started that reply before seeing the other post....

Can you have interference with one source of light?  Yes you can and you do @alancalverd  whenever that light passes through an aperture

Here's your later reply:
If the slit is wide, most of what you see is the unobstructed primary beam with some interference...
   So you fundamentally admit and you are aware that there can be interference when there was only one source of light.    You then did some carefull tip-toeing around the issue.   There were not two or three (or more) beams that interacted.   There was only the one beam of a certain width, it all came from just one source of light.   There is some diffraction at the aperture and there is then some interference effect that gives rise to an intensity on the screen that is the usual  Sinc function.
   (You also mentioned "airy disc", which as I'm sure you know you'll only get for a 2 dimensional aperture, a round hole instead of a slit.  I'm deeply suspicious that's an attempt to sidetrack the issue.   The key issue is that ONE source can still give you an interference effect.  You do get interference from just ONE beam of coherent light whenever that beam goes through an aperture).

Here's the more geometric interpretation using rays of light, which I'm sure you've seen before:

(https://upload.wikimedia.org/wikipedia/commons/thumb/0/0d/Single_Slit_Diffraction.svg/2389px-Single_Slit_Diffraction.svg.png)
[image from Wikimedia]

    Looking at this representation, the light rays are bent in various directions and there is a path length difference to where they converge on the screen, giving a phase difference and hence a constructive / destructive interference effect.    I'm sure you know this.  Anyway, as far as I'm concerned, you can legitimately call the single slit "diffraction pattern" you see on the screen an "interference pattern" because it is due an interference effect.

    I suppose if you wish to be pedantic, for a classical model of light, there NEVER is just one source of light.   There is always a beam of some width which we can divide and treat as separate sources or rays.   In the diagram above the beam was usefully divided into two halves.

   Now even if you slim the beam of light down as much as possible, so that you actually only had one photon,  then you are in the territory of Quantum mechanics and the usual single and double slit experiments.   You still have both the effects of diffraction (bending of a path away from a straight line) and also interference observable.   In this situation the interference is really an interference in the wave function ψ(x,t) and only becomes noticeable when the experiment is run several times and the usual interference pattern is built up on the screen.  (I'm sure you know all of that and there's no need to fill this thread with the details).   The key point is that even with just ONE photon being sent through an aperture, interference of some kind is still exhibited.    Your assertion that interference requires two or more sources is not something that I will accept without a (non physical, purely academic) fight.

Best Wishes.
Title: Re: why do a lot of people confuse between interference and diffraction?
Post by: alancalverd on 28/12/2022 23:27:50
So you fundamentally admit and you are aware that there can be interference when there was only one source of light.   

Oh dear, we are in danger of arguing about the meaning of "one" and "source"! In my analysis, and indeed in the Huygens model, an edge becomes a source.

Fortunately for the artist, I haven't seen this diagram previously. Look at the three rays entering from the left. The upper one turns left. The lower one, apparently diffracted from an antisymmetric edge, also turns left- why?  But most mysterious of all, the one in the middle also turns left for no reason at all! Doesn't light travel through space in straight lines any more?

The single-edge diffraction pattern is the effect of a semi-infinite aperture. In my universe a light ray passing more than a millimeter or so from the edge has no reason to deflect - a millimeter is "infinite" at optical wavelengths! But a ray grazing the edge will be diffracted, so the secondary wavefront can interfere with the primary ray  in the geometrically illuminated region, but the primary ray does not interfere with the diffracted ray in the geometric shadow region.

The Airy disc is of practical importance because most useful lenses are circular and we are usually concerned with the resolution of 2D sources, but the mathematics is no more than a function of the wavelength of the light divided by the aperture diameter. The geometry is irrelevant: it just happens that circular symmetry obviously gives you a circularly symmetric pattern of fringes.

 
Title: Re: why do a lot of people confuse between interference and diffraction?
Post by: alancalverd on 28/12/2022 23:45:39
I must take exception to your interpretation of the double-slit experiment.

If you count single photons arriving at the target, they turn up as "whole photons" with the same energy as the source, separated in time and with their spatial distribution determined by the probability function, or indeed a Huygens superposition function. But interference by definition requires the simultaneous presence of two entities, A interfering with B, or X superposed on Y. In the single-photon experiment A and B are never present at the same time so they can't interfere.

Let us suppose for a moment that a photon interfered with itself. Constructive interference would produce....a photon with twice the energy?  Not observed - we start with a red laser and end up with a red pattern.Two photons with the same energy? Not observed - one in, one out. Pity, that would give us free energy for ever! Destructive interference - where does the energy go?  You can't cheat by integrating over time because I am using a really, really weak source and you don't know when the next photon will arrive, if ever.

So whilst an interference model describes the result, it doesn't explain or represent the actual events that lead to it.
Title: Re: why do a lot of people confuse between interference and diffraction?
Post by: hamdani yusuf on 29/12/2022 02:29:49
I wrote that interference and diffraction are distinct phenomena, but both are effects of superposition.
Which is wrong. Superposition requires two sources, by definition. There is only one in diffraction.
Which is wrong. Superposition requires at least two sources, which can be more.
How many oscillating electrically charged particles in a diffracting obstacle?
Title: Re: why do a lot of people confuse between interference and diffraction?
Post by: hamdani yusuf on 29/12/2022 03:12:42
In my video, I mentioned color mixing, beat, and superposition of two light waves with perpendicular polarizations as some examples of superposition that doesn't produce interference
Color mixing is due to physiology, not physics! If you illuminate a surface with two light beams of different frequencies, the eye may perceive one different color but a spectrometer will show the original frequencies only.

This is different from true frequency modulation or mixing in a nonlinear medium. 

"Beat" is true superposition. When one frequency is much higher than the other, it is called "amplitude modulation".
A broadband light detector, such as LDR, detects combined light intensity. It has nothing to do with physiology.

Beat is a result of wave addition, while amplitude modulation is a result of wave multiplication.
Title: Re: why do a lot of people confuse between interference and diffraction?
Post by: hamdani yusuf on 29/12/2022 04:13:41
1.   We do not know exactly what the underlying process for diffraction or interference is.   There are some models.
The fact that there is a disagreement doesn't necessarily mean that no one knows.
FYI, there's still disagreement about the shape of the earth, or how its rotational axis precesses.
Title: Re: why do a lot of people confuse between interference and diffraction?
Post by: hamdani yusuf on 29/12/2022 04:28:00
2.   In those models, especially Huygens principle, it is quite common that diffraction and interference do appear to have similar explanations and could be due to the same underlying principles of physics.
Demonstration of electromagnetic phenomena often presents analogy from mechanical phenomena, because they are easier to observe with simpler tools. Wave diffraction and interference can be convincingly shown using surface water wave. But many mechanical analogies to explain electromagnetic wave are known to be misleading, such as rope and fence analogy to demonstrate polarization or a car moving from concrete to sand to demonstrate refraction.
The best way to learn about the difference between diffraction and interference is by studying both phenomena independently through experiments that only involve one of them and not the other.

Regarding Huygen's principle,
If you think that single slit interference pattern is produced by the space between edges of the slit which act as wave sources, you’ll be in trouble explaining the same pattern produced by a thin wire.
Title: Re: why do a lot of people confuse between interference and diffraction?
Post by: hamdani yusuf on 29/12/2022 04:39:50
3.  Is it important to distinguish between interference and diffraction because there is some very different underlying physics you (especially Hamdani Yusuf) are trying to present?   Answer:  No.
Yes, as I and Alan explained earlier. They are two distinct phenomena. Just because some experiments involve both of them, it doesn't mean that they are the same thing and inseparable.
Title: Re: why do a lot of people confuse between interference and diffraction?
Post by: hamdani yusuf on 29/12/2022 06:26:33
My view:  Sure, in some areas of work there will have to be some very precise use of terminology.  In more general science there is also some benefit in recognising that some apparently disparate things are actually closely related and the same underlying principles could apply.   For example,  Beta particles are not the same as high velocity electrons.  Beta particles can ONLY be produced by changes that take place in the nucleus of an atom.   However, it is much more useful just to recognise that they behave so much like high velocity electrons that they almost certainly are high velocity electrons.
They were originally called Becquerel's rays.
Here's some history.
Title: Re: why do a lot of people confuse between interference and diffraction?
Post by: alancalverd on 29/12/2022 10:33:36
How many oscillating electrically charged particles in a diffracting obstacle?
According to Huygens, none. According to your microwave experiments, lots and lots of free electrons or none at all.
Title: Re: why do a lot of people confuse between interference and diffraction?
Post by: hamdani yusuf on 29/12/2022 11:12:18
How many oscillating electrically charged particles in a diffracting obstacle?
According to Huygens, none. According to your microwave experiments, lots and lots of free electrons or none at all.
Obviously, not just one.
Title: Re: why do a lot of people confuse between interference and diffraction?
Post by: hamdani yusuf on 29/12/2022 11:38:43
When monochromatic light passes through a single slit and produces a pattern on a screen.  What do you call that pattern on the screen?
Interference pattern caused by diffraction.
In reflection mode, a single slit aperture which is made of reflective material also produces interference pattern. But this time, it doesn't involve diffraction.

When monochromatic light passes through two slits and produces a pattern.... What do you call that?
When ...light  ... passes.... diffraction grating... screen.  What do you call that?

Same as above answer.
Title: Re: why do a lot of people confuse between interference and diffraction?
Post by: hamdani yusuf on 29/12/2022 11:52:41
My view is... what the figgy pudding does it matter?
Does it matter if interference patterns in the cases of Lloyd's mirror and thin film interference are called reflection patterns instead?
Title: Re: why do a lot of people confuse between interference and diffraction?
Post by: hamdani yusuf on 29/12/2022 11:55:33
I have a delightfully disgusting story about x-ray diffraction which can wait for another day.
I can't wait to learn from your experience.
Title: Re: why do a lot of people confuse between interference and diffraction?
Post by: hamdani yusuf on 29/12/2022 12:01:25
Here's the more geometric interpretation using rays of light, which I'm sure you've seen before:
(https://upload.wikimedia.org/wikipedia/commons/thumb/0/0d/Single_Slit_Diffraction.svg/2389px-Single_Slit_Diffraction.svg.png)
The middle light beam seems to deflect for no obvious reason, which doesn't seem reasonable and suspiciously misleading. Will it behave the same way if the top and bottom obstacles are removed?
Title: Re: why do a lot of people confuse between interference and diffraction?
Post by: Eternal Student on 29/12/2022 15:04:14
Hi.

   Alancalverd and Hamdani Yusuf have asked similar things...

The middle light beam seems to deflect for no obvious reason, which doesn't seem reasonable and suspiciously misleading.

and,

But most mysterious of all, the one in the middle also turns left for no reason at all!

   I can't help but feel that you have misunderstood the Huygens-Fresnel principle.   The centre of the aperture isn't having just one beam of light sent away from it.   Neither is the left edge or right edge of the aperture.   Every point on the wavefront is a source of secondary spherical wavelets.   It's "spherical" as in light rays are sent in all directions.   This diagram may help:

(https://upload.wikimedia.org/wikipedia/commons/thumb/a/a5/Radial_vector_field.svg/1200px-Radial_vector_field.svg.png)

   Imagine the centre of that diagram is the centre of the aperture.   The red circles indicate peaks in the wave,  so any red circle is what can be called a wavefront.   The wave spreads out from the secondary source like this... in a spherical manner.   (Well, at the very least a forward travelling hemi-sphere but these details were discussed elsewhere and previously).   Now Huygens principle doesn't say anything about rays, only spherical wavelets BUT you can draw the rays just by noticing that they will always intersect the wavefront at 90 degrees.   So in the diagram above, the spherical wavelet can be considered as rays projected out radially  (that's the blue radial lines).
    So just to be clear:     All along the aperture,   at the left edge of it,  one third of the way along it,  at the centre of it,  four-fifths of the way along it   and  at the far right edge of it...... there are secondary spherical wavelets like this being produced.

   So the earlier diagram  (which I'll repeat below) is NOT really showing one incoming ray reach the aperture and only be deflected "left".   There are rays being sent out from every point along the aperture in ALL directions.    The diagram is just focusing attention on those rays which will converge on a point on the screen labelled   -θmin,0.    You determine the behaviour at other places on the screen by considering those rays which will converge on that part of the screen.
(https://upload.wikimedia.org/wikipedia/commons/thumb/0/0d/Single_Slit_Diffraction.svg/2389px-Single_Slit_Diffraction.svg.png)

   If you haven't seen the conventional derivation for the far-field diffraction pattern from a single slit  (sometimes called Fraunhofer Single Slit Diffraction) then I can only apologise and I can understand why your confusion has arisen.   Yes, of course it would seem odd that rays passing through the centre of the aperture are deflected left,  they aren't, there's a spherical secondary wavelet source there and hence rays are being thrown out in ALL directions.   I don't think I have the time to go over the conventional derivation at the moment and the mathematics might be quite dull for most people anyway.   If anyone is interested then ask for it, otherwise that's fine - have a good new year.

Best Wishes.
Title: Re: why do a lot of people confuse between interference and diffraction?
Post by: alancalverd on 29/12/2022 16:32:23
The rectilinear propagation of light derives from the Huygens wavelet model because an infinite number of infinitesimal adjacent wavelets interfere, so the middle of a wide parallel beam  proceeds as a linear wave. Your interpretation prohibits the existence of a parallel beam, but your diagram shows a parallel beam coming from the left, the middle of which suddenly takes it into its mind to diverge. You can't have your cake and eat it!

Fraunhofer diffraction ideally demands a narrow slit where the intensity of diffracted light from both edges is significant compared with the undisturbed primary beam. If that were not so, we wouldn't have geometric shadows or a bright center spot in the pattern you showed. You can eliminate the primary beam with an Arago geometry https://en.wikipedia.org/wiki/Arago_spot which produces a bright center spot caused by interference of the edge diffracted light only, and develop it into an inverse zone plate that adds more intensity to the center spot from more distant diffractive edges.
 
Title: Re: why do a lot of people confuse between interference and diffraction?
Post by: Eternal Student on 29/12/2022 23:54:34
Hi.

Your interpretation prohibits the existence of a parallel beam, but your diagram shows a parallel beam coming from the left, the middle of which suddenly takes it into its mind to diverge. You can't have your cake and eat it!
    That bit is sensible, well done.
    There are, of course, some minor notes:  (i) It's not really my interpretation, it's a mainstream explanation (and I don't know who or how many people developed it).   
    (ii)  The diagram (which also wasn't mine) actually does show a proper (infinite) plane wave approaching from the left if you look carefully.   There are red lines running vertically and  ~  wiggly red lines drawn that intersect those wavefronts at 90 degrees (just like the earlier diagram with radial projected rays) which are effectively treated as light rays.   As you probably know, when the plane wave is infinite then Huygens principle does allow it to progress linearly.
    The full derivation does assume the incident wave was a proper (i.e. infinite width) plane wave.   Exactly as you stated, a wide beam, especially at its centre, is well approximated as a plane wave.   Similarly, if the original light source was a point light source then it does really hit the aperture as a spherical wave BUT this is well approximated as a plane wave (in the region around the equipment) provided the distance from the point source to the aperture is large.

   
Fraunhofer diffraction ideally demands a narrow slit.... (with further comments about beams)....
    The main condition for Fraunhofer (far-field) diffraction mathematics to apply is that    fbca0f6c70aa053f5359245ac50724ad.gif
    with W = maximum distance between two points in the aperture (e.g. its diameter for a round hole);   λ = wavelength of light used;    L = the minimum of length (i) and (ii)  described below:
(i) The distance between the point light source (if the incident wave wasn't genuinely a plane wave but was only approximated as such) and the aperture.     
(ii) The distance between the aperture and the screen.

   So while the easiest way to meet that criteria is to have a small aperture, it is by no means the only way that Fraunhofer diffraction is valid.   Making the distance between the aperture and the screen large compared to the aperture size is also just fine (and is why this thing is often called "far field" diffraction).
    Quick Reference:     https://en.wikipedia.org/wiki/Fraunhofer_diffraction#Derivation_of_Fraunhofer_condition    see especially the condition boxed to the right of that Wikipedia page if you only have 30 seconds to spare.

------
    There is then some mention of a Poission spot (Arago spot was your preferred term).  I'm not sure exactly what that was about.  It sort of blended in with a notion of treating the e-m wave as a set of beams.  You seem to really like a model where you have separate and easily labelled beams.   There's a primary beam and some edge diffracted beams.   I expect you can develop a simple model based on multiple beams with some near the edge of the aperture as the "diffracted beam" but you just don't need to.    There's a perfectly good model for the far field diffraction pattern that doesn't worry itself about "primary beams" and "diffracted beams".  In particular the centre of the incident wave on the aperture (which I think is what you would consider as "the primary beam") is not left alone or assumed to have any special reason to be exempted from Huygens principle.   Everywhere follows that principle. 

Let's take another statement:
   
If that were not so (the slit being narrow?), we wouldn't have geometric shadows or a bright center spot in the pattern you showed.
    You'll always have a geometric shadow because that is just geometry.  I think what you're saying is that it wouldn't always be dark and shadowy in the geometric shadow.   
    The usual single slit diffraction pattern or Fraunhofer diffraction pattern does hold for wide slits, provided the Fraunhofer condition is still met which means the screen is still far away by comparison to the slit width.   The wider the slit, the narrower the pattern on the screen  (e.g the central or 0th order bright region is narrower and correspondingly all other peaks are narrower and also closer -  the whole thing is just squashed in the  "along the screen" axis).  See http://hyperphysics.phy-astr.gsu.edu/hbase/phyopt/sinslitd.html    for some diagrams.    It is a little counter-intuitive that making the aperture wider actually narrows the central bright region but that's how it is.   The central bright region is never going to be narrower than the aperture because that's only going to happen when the Fraunhofer diffraction condition is no longer met, you can't model everything you need to with plane waves - and at that point a different model for the diffraction must be used.   (There's a Fresnel integral for near field diffraction for example).   
    Another thing which can help to reassure people that the model is still sensible, is that the width of the diffraction pattern is not telling you about the total intensity.  A very narrow aperture does produce a very wide central "bright patch" on the screen but it's not actually bright,  it's just as bright as the pattern is going to get.  In absolute terms it's a very low intensity when the aperture is small and the central bright region will fade below human perception rapidly as you move away from the centre (if indeed you can see anything in the centre at all to start with).  Meanwhile for a larger aperture, even the half-way positions between minima and maxima patches on the screen are still bright enough to be noticed.

    Anyway, if you like an interpretation with "primary beams" and "diffracted beams" then I guess that's fine.   I suppose, it's more intuitive to be thinking that the central part of a beam does just come straight through the centre of aperture unaffected.  Personally, I prefer the Huygens approach - there's nothing special or different about any bit of light anywhere, it all creates secondary spherical wavelets.   (I'm thinking that we might have drifted a long way off the OP,  so I'm just going to stop).

Best Wishes.
Title: Re: why do a lot of people confuse between interference and diffraction?
Post by: alancalverd on 30/12/2022 00:21:48
I spent several happy years making and interpreting x-ray diffraction photographs, every one of which literally had a hole in the middle to let the primary beam out. If we didn't do that, the scattered radiation from the majority of the x-rays that passed straight through the sample would fog the film and make the diffraction spots difficult to find. Hence an abiding interest in knowing the difference.
Title: Re: why do a lot of people confuse between interference and diffraction?
Post by: hamdani yusuf on 30/12/2022 06:18:02
This diagram may help:

(https://upload.wikimedia.org/wikipedia/commons/thumb/a/a5/Radial_vector_field.svg/1200px-Radial_vector_field.svg.png)

Is there a wavelet propagating from outer circle back to the center?
Title: Re: why do a lot of people confuse between interference and diffraction?
Post by: hamdani yusuf on 30/12/2022 06:23:37
If you haven't seen the conventional derivation for the far-field diffraction pattern from a single slit  (sometimes called Fraunhofer Single Slit Diffraction) then I can only apologise and I can understand why your confusion has arisen.   Yes, of course it would seem odd that rays passing through the centre of the aperture are deflected left,  they aren't, there's a spherical secondary wavelet source there and hence rays are being thrown out in ALL directions.   I don't think I have the time to go over the conventional derivation at the moment and the mathematics might be quite dull for most people anyway. 
I have seen many of them.

This is from Khan academy, which can be considered a mainstream.

And a follow up video.

Quote
If anyone is interested then ask for it, otherwise that's fine - have a good new year.

Since you don't seem to be confused by this experiment result, perhaps you can explain it better than Salman Khan.

Can you derive the interference pattern produced by a thin wire diffraction?
Title: Re: why do a lot of people confuse between interference and diffraction?
Post by: alancalverd on 30/12/2022 10:54:26
I think the answer to "why people confuse them" is because they are badly taught!

Sadly, Khan seems to  be no better than anyone else.

Right from the start, Huygen is a model, not a reason!
The incoming wavefront is curved, as if from a point source. But SSI also occurs with a parallel beam - this is important later on*.
At 3:02 the wavefront in vacuo suddenly terminates at the top. The commentary talks about a barrier but it hasn't reached the barrier yet - no wonder the audience is lost by this stage.
Then he ignores the crucial fact that Huygens requires an infinite number of infinitesimal sources, and constructs a diagram with a finite number of sources that magically take their place as the beam passes through the slit!
*Why did he surmise that, in the absence of diffraction, the central spot would  have a Gaussian intensity distribution? In classical linear optics you would simply get a hard shadow from a parallel beam.

So by 5 minutes into a 15 minute presentation, the audience has already swallowed several red herrings and is rapidly losing its appetite for physics.

Title: Re: why do a lot of people confuse between interference and diffraction?
Post by: hamdani yusuf on 30/12/2022 12:09:42
I think the answer to "why people confuse them" is because they are badly taught!

Sadly, Khan seems to  be no better than anyone else.
Khan made the video based on textbooks that he read. Based on his resume, he should have read some of the best textbooks available on this issue.
Quote
He attended Grace King High School, where, as he recalls, "a few classmates were fresh out of jail and others were bound for top universities."[11] He was a cartoonist for the high school's newspaper.[12] Khan took upper-level mathematics courses at the University of New Orleans while he was in high school and graduated as valedictorian in 1994.[13][14]

He attended the Massachusetts Institute of Technology (MIT), graduating with Bachelor of Science and Master of Science degrees in Course 6 (electrical engineering and computer science), and another bachelor's degree in Course 18 (mathematics), in 1998.[15] In his final year, Khan was the president of the "Senior Gift Committee," a philanthropy program of the graduating class.[16]
https://en.wikipedia.org/wiki/Sal_Khan

This demonstration video from MIT shows what we observe in real life instead of drawing or animation, but unfortunately doesn't even try to offer an explanation.
Title: Re: why do a lot of people confuse between interference and diffraction?
Post by: hamdani yusuf on 30/12/2022 12:17:33
Here's another video trying to explain single slit diffraction.
Single Slit Diffraction is like getting surprised by a text you just sent yourself | Doc Physics

And here's the channel's description.
Quote
https://www.youtube.com/@DocSchuster/about
Description
I am one of four passionate physics teachers at Webster Groves High School in St. Louis, MO.  I teach AP Physics 1 and 2, but I teach calculus with the class as well, since physics makes no sense without it.  I use Walker's excellent textbook, Physics 2nd Edition.  My videos are organized into playlists for each chapter of that book, and really should be watched sequentially if you have the time.

I try to align my class with a first-year physics class at top universities.  You may find the videos useful for tutoring and review.  Please do problems immediately after watching a video.  This is how you will become a stronger physicist.
Title: Re: why do a lot of people confuse between interference and diffraction?
Post by: hamdani yusuf on 30/12/2022 12:21:26
The incoming wavefront is curved, as if from a point source. But SSI also occurs with a parallel beam - this is important later on*.
A collimating lens or mirror can make a spreading light beam turn into a parallel light beam, which virtually relocate the position of the point source much far away from the slit.
Title: Re: why do a lot of people confuse between interference and diffraction?
Post by: alancalverd on 30/12/2022 13:22:58
Here's another video trying to explain single slit diffraction.
And again, the central ray suddenly diverts to the left for no obvious reason!
Title: Re: why do a lot of people confuse between interference and diffraction?
Post by: Bored chemist on 30/12/2022 17:45:52
A collimating lens or mirror can make a spreading light beam turn into a parallel light beam, which virtually relocate the position of the point source much far away from the slit.
No.
No matter how many times you say that, it still will never be true because of diffraction.
Title: Re: why do a lot of people confuse between interference and diffraction?
Post by: hamdani yusuf on 31/12/2022 03:18:02
No.
No matter how many times you say that, it still will never be true because of diffraction.

What do you think collimators are for?
Are they all useless due to diffraction?

(https://cdn.britannica.com/09/158609-050-7730CD21/collimator-light-point-source-beam.jpg)
Quote
https://www.britannica.com/technology/collimator
collimator, device for changing the diverging light or other radiation from a point source into a parallel beam. This collimation of the light is required to make specialized measurements in spectroscopy and in geometric and physical optics.


Quote
Collimators are optical systems used to imitate standard targets placed in "optical infinity" (very long distance). The collimators are used for projection of image of reference targets into direction of tested imagers.  According to type of optical elements used in design, collimators are divided into two groups: reflective collimators and refractive collimators. Reflective collimators due to their wide spectral range are almost exclusively used in systems for testing thermal imagers and are also preferable in systems testing TV cameras, SWIR imagers, laser systems or multi-sensor surveillance systems. Refractive collimators are mostly used in systems for testing night vision devices or TV cameras working in visible/near infrared range.

From optical designer view, the reflective collimators are inverted telescopes. Therefore it can be claimed that there are many types of reflective collimators depending on mirrors configurations (Newton, Cassegrain, Schwarzschild, Maksutov, etc). However, practically reflective collimators are typically built using Newton design (big parabolic primary, collimating mirror and smaller secondary flat mirror). Next, the reflective collimators can be divided into two basic types:  off axis collimators and on-axis collimators. 
(https://www.inframet.com/Images/Level_2/TestModules/Collimators/CDT_diagram.jpg)
https://www.inframet.com/collimators.htm

Title: Re: why do a lot of people confuse between interference and diffraction?
Post by: hamdani yusuf on 31/12/2022 03:30:44
Here's another video trying to explain single slit diffraction.
And again, the central ray suddenly diverts to the left for no obvious reason!
Unfortunately, that's what commonly taught in high schools. It would take some considerable efforts to unlearn it and relearn the better explanations.
Title: Re: why do a lot of people confuse between interference and diffraction?
Post by: Eternal Student on 31/12/2022 17:34:21
Hi.

     I only watched bits of the video (and at about double speed) but my opinion was that the video from Khan academy is not perfect.   It's good enough for school level but there are some simplifications that can't really be so easily swept under the rug at University level.   I suppose a 15 minute video is already "long enough" and the presenter probably had their own good reasons for keeping the video short.
     The follow-up video had even more hand-waving or vague argument without much mathematics and I abandoned watching it half way through.  The second half might have been good.

Since you don't seem to be confused by this experiment result, perhaps you can explain it better than Salman Khan.
   Probably not.   I mean let's give the presenter their fair credit here and recognise just how much work would have been required.  At my rate of production it would take hours (days?) to create sufficient text, diagrams and video.   I also probably don't have the right voice or presentation style.   More-over it's sufficiently long and complicated that only someone really interested will watch or read it.   Those wanting a quick impression will just go watch YT videos like that one from Khan academy.

Quote
[From Hamdani]    Can you derive the interference pattern produced by a thin wire diffraction?
   Yes, I think so,  although I don't think it's entirely "my" work.   (I'm sure it's in some text books but you would need to get the big ones that specialise in physical optics and not some University textbook that just skims the topic).  Keywords to look up in the index might include "Babinet's principle",  "Aperture function",  "Franhofer integral" and not just "thin wire diffraction".
    Did you really want to see the mathematics?   It's hours (days?) of work to re-create it here on the forum.   Meanwhile there will be one reader and it still only takes a few minutes to ignore it.

 
This demonstration video from MIT shows what we observe in real life instead of drawing or animation, but unfortunately doesn't even try to offer an explanation.
   Yes, that's pretty much what you would get from the mathematics that is being proposed.   That one is a nice video.

Unfortunately, that    (...confusing ideas with rays bending....) is what commonly taught in high schools. It would take some considerable efforts to unlearn it and relearn the better explanations.
    I can basically agree with that.   Schools do not attempt to show the mathematics used and there is inevitably some hand-waving and waffle.   You'd like your pupils to have access to complex numbers, calculus and some appreciation of the wave equation (at least enough to accept that the ODE is linear and sums of solutions are solutions etc.).   It would be much faster if your pupils were already familiar with Fourier Transformations...   So very roughly this is just unlikely to get on a first year University course in Physics,   2nd year+ maybe,  certainly getting it on a school syllabus is really asking too much.

Best Wishes.
Title: Re: why do a lot of people confuse between interference and diffraction?
Post by: Eternal Student on 31/12/2022 18:40:16
Hi.

   I wonder if this is quicker way to convince you that mathematics exists to describe diffraction by a thin wire instead of by a thin slit.

    You (Hamdani) suggested a video by MIT.   They have another video with the same presenter demonstrating diffraction by thin wire.    (Duration about 5 minutes,  available on You Tube,   "Optics: Fraunhofer diffraction - thin wires | MIT Video Demonstrations in Lasers and Optics").

    Now, just spend a moment and ask yourself.... why would the presenter / lecturer have asked the students to calculate the width of the wire(s) if there wasn't some mathematics already available to model this situation? 

   No one is suggesting that Huygens principle is faultless or exactly what is happening at some fundamental level.   However, a few simple assumptions does produce a model which can predict the diffraction patterns you get really well.

Best Wishes.
Title: Re: why do a lot of people confuse between interference and diffraction?
Post by: hamdani yusuf on 31/12/2022 22:08:08
I suppose a 15 minute video is already "long enough" and the presenter probably had their own good reasons for keeping the video short.
He could split it into several short videos. Just make sure that important points are delivered to prevent misunderstandings.
Title: Re: why do a lot of people confuse between interference and diffraction?
Post by: hamdani yusuf on 31/12/2022 22:11:45
The follow-up video had even more hand-waving or vague argument without much mathematics and I abandoned watching it half way through.  The second half might have been good.
You can finish it anytime you like whenever you are interested in this problem and have the time to spend.
Title: Re: why do a lot of people confuse between interference and diffraction?
Post by: hamdani yusuf on 31/12/2022 22:13:39
Probably not.   I mean let's give the presenter their fair credit here and recognise just how much work would have been required.  At my rate of production it would take hours (days?) to create sufficient text, diagrams and video.   I also probably don't have the right voice or presentation style.   More-over it's sufficiently long and complicated that only someone really interested will watch or read it.   Those wanting a quick impression will just go watch YT videos like that one from Khan academy.
You don't have to create a video. Just mention what's wrong with Khan's videos, and explain what needs to change to make it right.
Title: Re: why do a lot of people confuse between interference and diffraction?
Post by: hamdani yusuf on 31/12/2022 22:37:20
Yes, I think so,  although I don't think it's entirely "my" work.   (I'm sure it's in some text books but you would need to get the big ones that specialise in physical optics and not some University textbook that just skims the topic).  Keywords to look up in the index might include "Babinet's principle",  "Aperture function",  "Franhofer integral" and not just "thin wire diffraction".
    Did you really want to see the mathematics?   It's hours (days?) of work to re-create it here on the forum.   Meanwhile there will be one reader and it still only takes a few minutes to ignore it.
I've been searching for years. I only found explanations based on Babinet's principle, and then derive the equation based on single slit experiment, assuming that they are really equivalent. I can't find any explanation based on actual geometry of a thin wire.

BTW, I 've made an experiment showing the difference between thin wire and single slit experiment. I'll share it as soon as it becomes available on line.
Title: Re: why do a lot of people confuse between interference and diffraction?
Post by: hamdani yusuf on 01/01/2023 00:23:31
No one is suggesting that Huygens principle is faultless or exactly what is happening at some fundamental level.   However, a few simple assumptions does produce a model which can predict the diffraction patterns you get really well.
A few "tweaks" can predict the motion of planets pretty well using geocentric model. They are called epicycles. Just like the explanation on single slit experiment, epicycles also involve changes of movement without physical cause.
Title: Re: why do a lot of people confuse between interference and diffraction?
Post by: alancalverd on 01/01/2023 10:18:30
So here's how it should be taught.

We observe that shadows do not have a true geometric hard edge, so we need something other than a linear ray diagram to describe the phenomenon*.

Huygens proposed a mathematical model in which every point on a wavefront can be considered to be the source of another wave. Note "every". We can then predict the progress of a wave as the integral of an infinity of infinitesimal wavelets.

Consider a tangent to the wavefront. Wavelets orginating from infinitesimally adjacent points will be in phase as the tangent advances, so the net effect of their superposition will be an expanding spherical wavefront from a point source, or a progressive plane wavefront from an infinitely wide parallel beam.

Now consider an infinitely wide parallel beam, interrupted by an edge on the right - a seminfinite front. There being no  sources to the right of the edge, the only contributions downstream come from those "Huygens sources" to the left of the edge, so some of the reinforcing or cancelling wavelets are absent from the downstream wavefront. If we solve the integral at any point downstream it no longer looks like a plane but has ripples caused by the absence of those contributors on the right. We can calculate the distribution of intensity with angle from Huygens' model  if we know the wavelength.

Note that this is an analytic model that gives the observed answer. It does not purport to describe the underlying physics of electromagnetic or any other wave propagation. 

*That's where Khan and all the above diagrams go wrong: they use a finite number of wavelet sources and then have to introduce an absurd ray diagram. You can't mix ray optics with Huygens analysis!
Title: Re: why do a lot of people confuse between interference and diffraction?
Post by: hamdani yusuf on 01/01/2023 11:38:58
Huygens proposed a mathematical model in which every point on a wavefront can be considered to be the source of another wave. Note "every". We can then predict the progress of a wave as the integral of an infinity of infinitesimal wavelets.
Do you think that Huygen's model is accurate to represent the propagation of light?
Why, or why not?
Does it have any limitations beyond which it's no longer accurate?
Title: Re: why do a lot of people confuse between interference and diffraction?
Post by: alancalverd on 01/01/2023 14:10:43
Do you think that Huygen's model is accurate to represent the propagation of light?
Why, or why not?
Yes, because it does.

As I've shown, it predicts diffraction.

It has a serious limitation because it predicts an equal and opposite backward flow of energy in a parallel beam, which is not observed, so it is a model, not an explanation - you need Maxwell and Poynting to explain unidirectional propagation, and Huygens also becomes less intuitive when the slit is small compared with the wavelength of the light, but strict adherence to it the "infinity if infintesimals" should avoid such nonsense as
 
Quote
All the waves passing through the slit interfere to produce a diffraction pattern consisting of bright and dark fringes.
from an internet source that will remain nameless.
Title: Re: why do a lot of people confuse between interference and diffraction?
Post by: alancalverd on 02/01/2023 11:07:06
On further consideration, a proper Huygens analysis avoids the physical nonsense of an edge becoming a source. An edge is a sink, not a source. This is obvious because there is a shadow behind the bulk material, and only one energy source in the system: when we switch off the laser, the edge does not continue to emit light!

The forward wave propagates because each infinitesimal  source of spherical wavelets is surrounded by an infinity of others, so there is only one direction (for a parallel beam - or "any radial direction" for a point source) in which all the secondary wavelets are in phase.

So the wavelets generated near (not "by") the edge are not "surrounded by an infinity of others" and therefore not subject to destructive interference in all but one direction of propagation.

It's still a model but obviously a lot closer to the truth and easier to teach than the drivel in the textbooks. The joy of mathematics is that we can talk about "semi-infinite" wavefronts without embarrassment, and any slit wider than a few microns is two overlapping semi-infinite systems for all practical purposes. It's an odd case of leaping from maths to engineering without passing through any validating physics!
Title: Re: why do a lot of people confuse between interference and diffraction?
Post by: hamdani yusuf on 02/01/2023 15:00:25
Yes, because it does.

Does it explain diffraction and refraction of light correctly?

As I've shown, it predicts diffraction.
How does it derive equation for diffraction? Can it predict positions of maximum constructive interference?
In the commonly used formula for destructive interference in single slit experiment, why the formula doesn't apply for θ=0?
(https://openstax.org/apps/archive/20221109.213337/resources/e1ae2242e98739a095573c9bbebfc5220a717777)
Title: Re: why do a lot of people confuse between interference and diffraction?
Post by: hamdani yusuf on 02/01/2023 15:30:35
Here are some effects that happen to a a light beam when it interacts with a macroscopic object (significantly larger than the wavelength).
- It's transmitted. The direction doesn't change.
- It's absorbed. Some of the light energy is converted to some other forms.

- It's reflected. The direction changes, it comes back to the first medium.
- It's refracted. The direction changes, it enters the second medium.
- It's diffracted. The direction changes, it enters the second medium at the edge, and comes out again to the shadow region.

The original light rays and the rays experiencing change of direction due to reflection, refraction, and diffraction can interfere with one another, producing destructive and constructive interference. AFAIK, no one ever mistakenly called interference pattern as reflection pattern nor refraction pattern, even when they are the cause of interfering light rays.
Title: Re: why do a lot of people confuse between interference and diffraction?
Post by: Bored chemist on 02/01/2023 19:40:39
No.
No matter how many times you say that, it still will never be true because of diffraction.

What do you think collimators are for?
Are they all useless due to diffraction?

(https://cdn.britannica.com/09/158609-050-7730CD21/collimator-light-point-source-beam.jpg)
Quote
https://www.britannica.com/technology/collimator
collimator, device for changing the diverging light or other radiation from a point source into a parallel beam. This collimation of the light is required to make specialized measurements in spectroscopy and in geometric and physical optics.


Quote
Collimators are optical systems used to imitate standard targets placed in "optical infinity" (very long distance). The collimators are used for projection of image of reference targets into direction of tested imagers.  According to type of optical elements used in design, collimators are divided into two groups: reflective collimators and refractive collimators. Reflective collimators due to their wide spectral range are almost exclusively used in systems for testing thermal imagers and are also preferable in systems testing TV cameras, SWIR imagers, laser systems or multi-sensor surveillance systems. Refractive collimators are mostly used in systems for testing night vision devices or TV cameras working in visible/near infrared range.

From optical designer view, the reflective collimators are inverted telescopes. Therefore it can be claimed that there are many types of reflective collimators depending on mirrors configurations (Newton, Cassegrain, Schwarzschild, Maksutov, etc). However, practically reflective collimators are typically built using Newton design (big parabolic primary, collimating mirror and smaller secondary flat mirror). Next, the reflective collimators can be divided into two basic types:  off axis collimators and on-axis collimators. 
(https://www.inframet.com/Images/Level_2/TestModules/Collimators/CDT_diagram.jpg)
https://www.inframet.com/collimators.htm

Saying it one more time still won't make it true.
Title: Re: why do a lot of people confuse between interference and diffraction?
Post by: hamdani yusuf on 02/01/2023 21:52:22
Saying it one more time still won't make it true.
What do you think collimators are for?
Title: Re: why do a lot of people confuse between interference and diffraction?
Post by: hamdani yusuf on 02/01/2023 22:01:11
Yes, because it does.

As I've shown, it predicts diffraction.
How do you explain half interference pattern that I have shown in another thread?
Title: Re: why do a lot of people confuse between interference and diffraction?
Post by: alancalverd on 03/01/2023 08:54:17
Does it explain diffraction and refraction of light correctly?
It models diffraction and refraction well enough that it is used to design lenses. It doesn't purport to explain anything, as I pointed out earlier.


How does it derive equation for diffraction? Can it predict positions of maximum constructive interference?
Yes. It is the basis for all the textbook derivations of diffraction equations.

Your diagram for θ = 0 is incorrect as it doesn't show the effect of a noninfinite beam.

You have made the usual textbook errors of assuming Huygens' infinite array of spherical wavelets  then attempting to represent it with ray propagation of truncated wavefronts.

Beware of being misled by your own models - you could end up like King Canute or even Vladimir Putin!
Title: Re: why do a lot of people confuse between interference and diffraction?
Post by: alancalverd on 03/01/2023 11:57:57
The original light rays and the rays experiencing change of direction due to reflection, refraction, and diffraction can interfere with one another, producing destructive and constructive interference.
Another example of confusing two models. Rays can't interfere - you need waves. We often use rays to model refraction and reflection but they can't model any phenomenon like interference or diffraction.
Title: Re: why do a lot of people confuse between interference and diffraction?
Post by: hamdani yusuf on 04/01/2023 08:06:30
It models diffraction and refraction well enough that it is used to design lenses. It doesn't purport to explain anything, as I pointed out earlier.
Lenses has been made long before Huygen even lived. They can be designed based on simple ray tracing.

Does it model polarization of light?
Title: Re: why do a lot of people confuse between interference and diffraction?
Post by: hamdani yusuf on 04/01/2023 08:13:24
The original light rays and the rays experiencing change of direction due to reflection, refraction, and diffraction can interfere with one another, producing destructive and constructive interference.
Another example of confusing two models. Rays can't interfere - you need waves. We often use rays to model refraction and reflection but they can't model any phenomenon like interference or diffraction.
Do you know Lloyd's mirror?
Title: Re: why do a lot of people confuse between interference and diffraction?
Post by: hamdani yusuf on 04/01/2023 08:16:30
Yes. It is the basis for all the textbook derivations of diffraction equations.
Khan's Academy video shows that it's not the case.
Title: Re: why do a lot of people confuse between interference and diffraction?
Post by: hamdani yusuf on 04/01/2023 08:18:26
Your diagram for θ = 0 is incorrect as it doesn't show the effect of a noninfinite beam.
What is the correct explanation?
How should the diagram look like?
Title: Re: why do a lot of people confuse between interference and diffraction?
Post by: hamdani yusuf on 04/01/2023 08:21:00
You have made the usual textbook errors of assuming Huygens' infinite array of spherical wavelets  then attempting to represent it with ray propagation of truncated wavefronts.
Is there unusual textbook that doesn't make the same error?
Title: Re: why do a lot of people confuse between interference and diffraction?
Post by: hamdani yusuf on 04/01/2023 08:21:46
Yes, because it does.

As I've shown, it predicts diffraction.
How do you explain half interference pattern that I have shown in another thread?
In case you are wondering which one.

Frequently. I  am a physicist.
It looks like we are lucky to have a physicist on board. I'd like to hear what you think about this problem.
Or may be you know someone who is more suitable to give an answer.
Apologies for picking this up so late in the discussion. Please remind me which problem? 

Here. See the pattern shown in 0:07. Do you have a better explanation?
Here's my newest video investigating diffraction of light by producing single side interference pattern.

Title: Re: why do a lot of people confuse between interference and diffraction?
Post by: alancalverd on 04/01/2023 19:30:22
Do you know Lloyd's mirror?

Wikipedia gives a good summary buut rather overcooks it, I feel:
Quote
The Lloyd mirror effect has been implicated as having an important role in explaining why marine animals such as manatees and whales have been repeatedly hit by boats and ships. Interference due to Lloyd's mirror results in low frequency propeller sounds not being discernible near the surface, where most accidents occur.

Manatees and whales are mammals so come to the surface to breathe, and also to catch pelagic prey. Ships are also mostly on the surface. Where else could they possibly collide?

But I digress. The ray construction shows that photons can arrive at a given point via two different path lengths, but doesn't imply interference because it would be equally applicable to particles as to waves. There is no mechanism for the destructive interference of particles.
Title: Re: why do a lot of people confuse between interference and diffraction?
Post by: alancalverd on 04/01/2023 19:33:21
Is there unusual textbook that doesn't make the same error?
I haven't looked very hard, but I doubt it.
Title: Re: why do a lot of people confuse between interference and diffraction?
Post by: hamdani yusuf on 05/01/2023 04:32:34
Do you know Lloyd's mirror?

Wikipedia gives a good summary buut rather overcooks it, I feel:
Quote
The Lloyd mirror effect has been implicated as having an important role in explaining why marine animals such as manatees and whales have been repeatedly hit by boats and ships. Interference due to Lloyd's mirror results in low frequency propeller sounds not being discernible near the surface, where most accidents occur.

Manatees and whales are mammals so come to the surface to breathe, and also to catch pelagic prey. Ships are also mostly on the surface. Where else could they possibly collide?

But I digress. The ray construction shows that photons can arrive at a given point via two different path lengths, but doesn't imply interference because it would be equally applicable to particles as to waves. There is no mechanism for the destructive interference of particles.
Are you intentionally making simple things more complicated?

Just in case you can't find the correct link with the correct context.
Quote
https://en.wikipedia.org/wiki/Lloyd%27s_mirror
(https://upload.wikimedia.org/wikipedia/commons/1/18/%D0%97%D0%B5%D1%80%D0%BA%D0%B0%D0%BB%D0%BE_%D0%9B%D0%BB%D0%BE%D0%B9%D0%B4%D0%B0.png)
Lloyd's mirror is an optics experiment that was first described in 1834 by Humphrey Lloyd in the Transactions of the Royal Irish Academy.[1] Its original goal was to provide further evidence for the wave nature of light, beyond those provided by Thomas Young and Augustin-Jean Fresnel. In the experiment, light from a monochromatic slit source reflects from a glass surface at a small angle and appears to come from a virtual source as a result. The reflected light interferes with the direct light from the source, forming interference fringes.[2][3] It is the optical wave analogue to a sea interferometer.[4]

The reason I mentioned this experiment is to provide an example where interference occurs without involving diffraction, thus supporting the assertion that interference and diffraction are two distinct phenomena. They can occur at the same time in an experimental setup, but they can also occur independently.
In a Venn diagram, their relationship will look like this.
(https://upload.wikimedia.org/wikipedia/commons/thumb/9/99/Venn0001.svg/270px-Venn0001.svg.png)
Commonly discussed single slit and double slit experiments are represented by the red area in the diagram.
Title: Re: why do a lot of people confuse between interference and diffraction?
Post by: alancalverd on 05/01/2023 10:56:16
The reason I mentioned this experiment is to provide an example where interference occurs without involving diffraction
I think we agreed on that a long while ago!

I also mentioned interference between independent but synchronised  radio sources used for long range navigation, and the problems caused by edge diffraction of a homing beacon, with no interference.

But the fact remains that mixing ray optics with wave models in a textbook can lead to some serious confusion.
Title: Re: why do a lot of people confuse between interference and diffraction?
Post by: hamdani yusuf on 05/01/2023 14:27:12
The reason I mentioned this experiment is to provide an example where interference occurs without involving diffraction
I think we agreed on that a long while ago!

I also mentioned interference between independent but synchronised  radio sources used for long range navigation, and the problems caused by edge diffraction of a homing beacon, with no interference.

But the fact remains that mixing ray optics with wave models in a textbook can lead to some serious confusion.
How do you explain Lloyd's mirror interference using Huygen's principle?
How do you calculate positions of destructive and constructive interference on the screen?
Title: Re: why do a lot of people confuse between interference and diffraction?
Post by: alancalverd on 05/01/2023 17:46:55
The same way that you explain calculate everything else to which Huygens applies.

I repeat for the nth time, Huygens is a mathematical model, not an explanation.
Title: Re: why do a lot of people confuse between interference and diffraction?
Post by: hamdani yusuf on 06/01/2023 05:40:35
The same way that you explain calculate everything else to which Huygens applies.

I repeat for the nth time, Huygens is a mathematical model, not an explanation.
Is it falsifiable?
Title: Re: why do a lot of people confuse between interference and diffraction?
Post by: alancalverd on 06/01/2023 08:09:09
Yes. All you have to do is conduct an experiment whose results are not correctly predicted by the Huygens model. Your report is eagerly awaited!
Title: Re: why do a lot of people confuse between interference and diffraction?
Post by: hamdani yusuf on 07/01/2023 12:50:25
Yes. All you have to do is conduct an experiment whose results are not correctly predicted by the Huygens model. Your report is eagerly awaited!
What does it predict, exactly?
Does it predict half interference pattern?
Does it predict polarized single slit diffraction?
Title: Re: why do a lot of people confuse between interference and diffraction?
Post by: Bored chemist on 07/01/2023 13:51:52
Yes. All you have to do is conduct an experiment whose results are not correctly predicted by the Huygens model. Your report is eagerly awaited!
What does it predict, exactly?
Does it predict half interference pattern?
Does it predict polarized single slit diffraction?
Do you really not understand that those questions were asked when Huygens put the idea forward and, if it hadn't worked, the idea would have been discarded?
Title: Re: why do a lot of people confuse between interference and diffraction?
Post by: hamdani yusuf on 08/01/2023 03:51:04
Yes. All you have to do is conduct an experiment whose results are not correctly predicted by the Huygens model. Your report is eagerly awaited!
What does it predict, exactly?
Does it predict half interference pattern?
Does it predict polarized single slit diffraction?
Do you really not understand that those questions were asked when Huygens put the idea forward and, if it hadn't worked, the idea would have been discarded?
Do you have any reference?
Should we take your words for granted?
Title: Re: why do a lot of people confuse between interference and diffraction?
Post by: Bored chemist on 08/01/2023 10:35:00
Yes. All you have to do is conduct an experiment whose results are not correctly predicted by the Huygens model. Your report is eagerly awaited!
What does it predict, exactly?
Does it predict half interference pattern?
Does it predict polarized single slit diffraction?
Do you really not understand that those questions were asked when Huygens put the idea forward and, if it hadn't worked, the idea would have been discarded?
Do you have any reference?
Should we take your words for granted?
So, that's a "No".
You do not understand how science works.

Have you considered learning?
You are asking me to find a published record of someone saying "yes, it still works".
Why would anyone bother to publish that?

I'm not asking you to take my word for anything.
I'm saying that, in the last 300 years or more, plenty of people* have compared the calculation with the experiments and found that Huygens' construction works (within it's field of applicability).

Because, if they had found that it didn't work they would be famous as "the guy who proved Huygens was wrong.".

Do you really not see that?

* and I mean clever people- not you and me.
Title: Re: why do a lot of people confuse between interference and diffraction?
Post by: hamdani yusuf on 08/01/2023 15:45:37
So, that's a "No".
You do not understand how science works.
How do you think science work? It looks like your understanding is different than mine.
(https://upload.wikimedia.org/wikipedia/commons/thumb/8/82/The_Scientific_Method.svg/390px-The_Scientific_Method.svg.png)
(https://upload.wikimedia.org/wikipedia/commons/thumb/5/5c/The_Scientific_Method_as_an_Ongoing_Process.svg/345px-The_Scientific_Method_as_an_Ongoing_Process.svg.png)
Title: Re: why do a lot of people confuse between interference and diffraction?
Post by: hamdani yusuf on 08/01/2023 15:49:50
Have you considered learning?
You are asking me to find a published record of someone saying "yes, it still works".
Why would anyone bother to publish that?
I'm learning by asking question in a science forum, among other ways, such as making experiments and reading online resources.
In order to say "yes, it still works", there should be a record showing that it did work in the first time.
Title: Re: why do a lot of people confuse between interference and diffraction?
Post by: hamdani yusuf on 08/01/2023 15:55:33
I'm not asking you to take my word for anything.
I'm saying that, in the last 300 years or more, plenty of people* have compared the calculation with the experiments and found that Huygens' construction works (within it's field of applicability).
Some theories which survived for thousands years turned out to be false.
Everything works until it doesn't.
Have you watched Sal Khan's explanation on single slit experiment? Do you agree with him? Can you find something to improve the explanation?
Title: Re: why do a lot of people confuse between interference and diffraction?
Post by: Bored chemist on 08/01/2023 16:13:09
Some theories which survived for thousands years turned out to be false.
Not sure that's true of any scientific thory given that science has only been around for a few centuries.

But, more to the point.
Some calculations have been tested for 300 years and found to be true.
That group includes Huygens' construction.
Title: Re: why do a lot of people confuse between interference and diffraction?
Post by: Bored chemist on 08/01/2023 16:14:41
In order to say "yes, it still works", there should be a record showing that it did work in the first time.
There is
https://en.wikipedia.org/wiki/Treatise_on_Light

Did you not realise that?
Title: Re: why do a lot of people confuse between interference and diffraction?
Post by: hamdani yusuf on 08/01/2023 16:15:07
Because, if they had found that it didn't work they would be famous as "the guy who proved Huygens was wrong.".

Do you really not see that?

* and I mean clever people- not you and me.
They did find the weaknesses of Huygen's principle.
They are not famous because they can't show a better explanation.
Quote
Feynman defines the generalized principle in the following way:

"Actually Huygens’ principle is not correct in optics. It is replaced by Kirchoff’s [sic] modification which requires that both the amplitude and its derivative must be known on the adjacent surface. This is a consequence of the fact that the wave equation in optics is second order in the time. The wave equation of quantum mechanics is first order in the time; therefore, Huygens’ principle is correct for matter waves, action replacing time."
https://en.wikipedia.org/wiki/Huygens%E2%80%93Fresnel_principle#Generalized_Huygens'_principle
A simple Google search for limitations of Huygens' principle gives these answers.
Quote
The limitations of Huygens Wave Theory of Light are as follows:
It could not explain rectilinear propagation of light
It could not explain phenomenon of polarisation of light and phenomenon like Compton Effect, photoelectric effect.

https://www.toppr.com/ask/content/concept/drawbacks-of-huygens-wave-theory-210352/

Quote
Huygens’ principle failed to provide reasons for wave propagation in backward directions.The principle failed to explain the rectilinear propagation of light.It also failed to prove the concept of polarisation of light, emission of light, absorption of light, and the photoelectric effect. (Source: https://protonstalk.com/light/huygens-principle/#Limitations)
https://protonstalk.com/light/huygens-principle/#Limitations
Title: Re: why do a lot of people confuse between interference and diffraction?
Post by: Bored chemist on 08/01/2023 16:18:51
How do you think science work? It looks like your understanding is different than mine.

 [ Invalid Attachment ]

OK, now, when you get to the experiment and you analyse the data and it just tells you "what happened is exactly what was predicted" where do you publish the results?
Is there a journal of "stating the obvious" or " stuff that anyone who understands the field would have guessed"?
Title: Re: why do a lot of people confuse between interference and diffraction?
Post by: Bored chemist on 08/01/2023 16:20:31
A simple Google search for limitations of Huygens' principle gives these answers.
Being able to read this (especially the bit in brackets) gives you an insight into why it's irrelevant.
Huygens' construction works (within it's field of applicability).
Title: Re: why do a lot of people confuse between interference and diffraction?
Post by: alancalverd on 08/01/2023 17:33:08
It works well enough that people have used it in the design of everything from microscopes to space telescopes, and I have used it in radio navigation (wavelength around 1000 m) and x-ray crystallography (10-7 m). 

Even though some textbooks make a pig's ear of their derivation, if you use Huygens correctly as I have shown in this discussion, it predicts all the diffraction patterns you (and I) have seen.

And remember it's not a "principle" but a geometric construction. It does model the rectilinear propagation of light if you understand what you are doing with it. It does not purport to explain anything - it's geometry, not physics.

The folk who have found its "failings" seem to have all fallen into the trap of calling it a principle and thus deluding themselves into thinking it might predict things that it can't.  The only tool I can think of that can really do two unrelated tasks well is the Kalashnikov rifle, which incorporates a beer bottle opener, by design. 
Title: Re: why do a lot of people confuse between interference and diffraction?
Post by: hamdani yusuf on 08/01/2023 23:00:08
Being able to read this (especially the bit in brackets) gives you an insight into why it's irrelevant.
Quote from: Bored chemist on Today at 10:35:00
Huygens' construction works (within it's field of applicability).
If we follow your assertion above, we would find no need for better alternatives to geocentric and phlogiston models.
Title: Re: why do a lot of people confuse between interference and diffraction?
Post by: hamdani yusuf on 08/01/2023 23:14:29
Not sure that's true of any scientific thory given that science has only been around for a few centuries.
Have you read about Dalton's model?
Title: Re: why do a lot of people confuse between interference and diffraction?
Post by: hamdani yusuf on 08/01/2023 23:20:45
It works well enough that people have used it in the design of everything from microscopes to space telescopes, and I have used it in radio navigation (wavelength around 1000 m) and x-ray crystallography (10-7 m). 

Even though some textbooks make a pig's ear of their derivation, if you use Huygens correctly as I have shown in this discussion, it predicts all the diffraction patterns you (and I) have seen.
Geocentric model works well enough for many applications.

Which textbooks do you think explain it correctly?

How does it explain non-diffractive edges?
Title: Re: why do a lot of people confuse between interference and diffraction?
Post by: hamdani yusuf on 09/01/2023 02:45:38
And remember it's not a "principle" but a geometric construction. It does model the rectilinear propagation of light if you understand what you are doing with it. It does not purport to explain anything - it's geometry, not physics.

The folk who have found its "failings" seem to have all fallen into the trap of calling it a principle and thus deluding themselves into thinking it might predict things that it can't.  The only tool I can think of that can really do two unrelated tasks well is the Kalashnikov rifle, which incorporates a beer bottle opener, by design.
It's correctly called Huygen's principle.
https://en.m.wikipedia.org/wiki/Huygens%E2%80%93Fresnel_principle
Quote
every point on a wavefront is itself the source of spherical wavelets, and the secondary wavelets emanating from different points mutually interfere.[1] The sum of these spherical wavelets forms a new wavefront.
The principle can be mathematically expressed.

Not every principle represents physical reality.

My disagreement is in applying the principle for all kinds of waves, and assuming that light behaves just like mechanical waves. Using surface water wave analogy to introduce the concept of wave diffraction for light is fine as long as the limitations and differences are also clearly mentioned.
Title: Re: why do a lot of people confuse between interference and diffraction?
Post by: alancalverd on 09/01/2023 08:26:30
Whatever you call it, the construction still gives you the correct answer AFAIK for all wave systems. There is no point in using an analogy to describe something you can see anyway. This silliness seems to have arisen in the 1980s with the advent of wave tanks in school laboratories and the general loss of rigor in the science curriculum.
Title: Re: why do a lot of people confuse between interference and diffraction?
Post by: Bored chemist on 09/01/2023 08:35:16
If we follow your assertion above, we would find no need for better alternatives to geocentric and phlogiston models.
That conclusion is a non sequitur.
Have you read about Dalton's model?
Yes; it was pre-scientific, and it showed.

How does it explain non-diffractive edges?
You still have to tell us what these are.

Title: Re: why do a lot of people confuse between interference and diffraction?
Post by: hamdani yusuf on 13/01/2023 12:03:32

Have you read about Dalton's model?
Yes; it was pre-scientific, and it showed.

Is Bohr's model pre-scientific?
Title: Re: why do a lot of people confuse between interference and diffraction?
Post by: hamdani yusuf on 13/01/2023 12:05:13

How does it explain non-diffractive edges?
You still have to tell us what these are.


I better show you.

video #4 Non-diffractive Obstacle
I have uploaded new video showing diffraction in microwave frequency.


Basically, the experiment result leads us to conclude that diffraction comes from the material blocking the microwave path. When the obstruction is opaque enough, we find no diffraction. It's similar to my experiment using laser showing non-diffractive obstruction.

This result is not widely known yet.
 
Title: Re: why do a lot of people confuse between interference and diffraction?
Post by: Bored chemist on 13/01/2023 12:26:10
Some theories which survived for thousands years turned out to be false.
Not sure that's true of any scientific thory given that science has only been around for a few centuries.
Is Bohr's model pre-scientific?
Why would it be relevant since it hasn't survived for thousands of years?
Title: Re: why do a lot of people confuse between interference and diffraction?
Post by: hamdani yusuf on 13/01/2023 12:27:31
Whatever you call it, the construction still gives you the correct answer AFAIK for all wave systems. There is no point in using an analogy to describe something you can see anyway. This silliness seems to have arisen in the 1980s with the advent of wave tanks in school laboratories and the general loss of rigor in the science curriculum.
There are some significant differences between light wave and mechanical waves like sound, water surface,  or slinky. Light wave propagates fastest in vacuum, while mechanical waves need a medium.
Title: Re: why do a lot of people confuse between interference and diffraction?
Post by: hamdani yusuf on 13/01/2023 12:29:18
Some theories which survived for thousands years turned out to be false.
Not sure that's true of any scientific thory given that science has only been around for a few centuries.
Is Bohr's model pre-scientific?
Why would it be relevant since it hasn't survived for thousands of years?
because you said this about Dalton's model.
Yes; it was pre-scientific, and it showed.
Title: Re: why do a lot of people confuse between interference and diffraction?
Post by: theThinker on 14/01/2023 05:52:30
Whatever you call it, the construction still gives you the correct answer AFAIK for all wave systems. There is no point in using an analogy to describe something you can see anyway. This silliness seems to have arisen in the 1980s with the advent of wave tanks in school laboratories and the general loss of rigor in the science curriculum.
There are some significant differences between light wave and mechanical waves like sound, water surface,  or slinky. Light wave propagates fastest in vacuum, while mechanical waves need a medium.
There are not just "some significant differences" between light wave and mechanical waves like sound, water surface, etc...The difference is like the difference between heaven and hell!

Light in contemporary physics assumes it propagates through "empty" space whereas all mechanical waves are waves in material mediums. The difference has no comparison. Light would be just another wave only if we reintroduce the luminiferous aether of space, but mainstream physics is not willing to consider this alternative.   
Title: Re: why do a lot of people confuse between interference and diffraction?
Post by: Kryptid on 14/01/2023 06:38:23
Light would be just another wave only if we reintroduce the luminiferous aether of space, but mainstream physics is not willing to consider this alternative. 

It was considered. When experiment failed to find it, it was abandoned.
Title: Re: why do a lot of people confuse between interference and diffraction?
Post by: evan_au on 14/01/2023 06:56:39
A bit of a tangent...
Quote from: TheThinker
light in contemporary physics assumes it propagates through "empty" space whereas all mechanical waves...
Gravitational waves also travel through empty space.

Quote from: OP
why do a lot of people confuse between interference and diffraction?
When people talk about interference & diffraction, they are talking about continuous waves.

The gravitational wave events we can detect today are transients*
- But their path can still be bent by massive objects in space (eg black holes, galaxies or galaxy clusters)
- And their arrival times on Earth phase-shifted relative to each other
- Would you call this interference or diffraction?

Astronomers have looked for gravitationally-lensed gravitational wave events, without success:
https://www.ligo.org/science/Publication-O3aLensing/

*We might be able to detect more continuous gravitational waves when LISA is launched. Then the distinction between interference and diffraction might become more relevant.
https://en.wikipedia.org/wiki/Laser_Interferometer_Space_Antenna

This tangent was triggered by a conversation on a rather quirky astronomy podcast:
https://spacenutspodcast.com/podcast/space-nuts-or-astronomy-space-and-science-news/episode/03-space-nuts-best-of-2022-first-image-from-the-jwst
Title: Re: why do a lot of people confuse between interference and diffraction?
Post by: theThinker on 14/01/2023 07:40:37
@evan-au, I only have elementary physics background - gravitational waves are way beyond me.

But if you want an "opinion", I can easily provide one. 
Title: Re: why do a lot of people confuse between interference and diffraction?
Post by: hamdani yusuf on 14/01/2023 10:58:35
When people talk about interference & diffraction, they are talking about continuous waves.

The gravitational wave events we can detect today are transients*
- But their path can still be bent by massive objects in space (eg black holes, galaxies or galaxy clusters)
- And their arrival times on Earth phase-shifted relative to each other
- Would you call this interference or diffraction?
As I explained in my video, the distinguishing characteristic of interference is the existence of dark area produced by destructive interference. While defining characteristic of diffraction is bright spot on the shadow area.
Some experimental setups may have both of them, such as single slit and double slit experiments.
Some other experiments only show one of them, while some others show none of them.
Title: Re: why do a lot of people confuse between interference and diffraction?
Post by: Bored chemist on 14/01/2023 11:14:11
because you said this about Dalton's model.
I know I said that.
So what?

You said "Some theories which survived for thousands years turned out to be false.".
And that may be true.
But they were not scientific theories, so they are barely relevant on a science site.

Title: Re: why do a lot of people confuse between interference and diffraction?
Post by: hamdani yusuf on 14/01/2023 11:16:23
because you said this about Dalton's model.
I know I said that.
So what?
When did pre-scientific age stop?
Title: Re: why do a lot of people confuse between interference and diffraction?
Post by: Bored chemist on 14/01/2023 11:17:53
because you said this about Dalton's model.
I know I said that.
So what?
When did pre-scientific age stop?
It doesn't matter unless it was "thousands of years" ago.
Any idea  that was believed for thousands of years (whether it was right or wrong) wasn't scientific.

Science is commonly said to have started around 1600 with people like Francis Bacon.
Title: Re: why do a lot of people confuse between interference and diffraction?
Post by: evan_au on 15/01/2023 08:50:32
Quote from: evan_au
interference & diffraction...gravitational wave
I heard an explanation today from Sean Carroll. .. As I understand his explanation:
- Interference (black spots in a light field) occurs because electromagnetism has positive and negative charges
- The Electric field of Electromagnetic radiation (eg light) oscillates positive and negative (electromagnetic radiation is a bipolar wave).
- Where positive polarity coincides with negative polarity, you get interference and "black spots" (if the amplitude is equal)

However, gravitational fields have only positive masses, so you don't get positives & negatives in gravitational waves (gravitational waves are a quadripolar wave).
- I presume from this that gravitational waves are extremely unlikely to produce cancellation? (maybe impossible?)
- But diffraction is certainly possible.

Quote from: Sean Carroll
Murray Dunn says, “Is there anything that can absorb, reflect or refract gravitational waves? Or do they propagate undisturbed until the end of time?” Well, gravitational waves interact with matter, they do, just like electromagnetic waves do, I shouldn’t say just like, but not exactly just like.

0:53:41.2 SC: The crucially, crucially important difference, and I’ve emphasized this in many different contexts is that in electromagnetism, you have positive charges and negative charges. And by subtly mixing up the configuration of the positive charges and negative charges, you can basically create or cancel out any electromagnetic field you want. In particular, you can absorb or reflect, okay? Whereas gravitational charges are just the masses of the particles that you’re making up your system from, and those are always positive. Gravity is always attractive, there’s no repulsive gravitational force between two ordinary particles.

0:54:17.2 SC: And so it is enormously harder to manipulate gravitational waves, that plus, of course, the fact that gravity is just a really weak force, so that also makes it very, very hard. So therefore, to a really good approximation, gravitational waves just propagate undisturbed until the end of time. They will, of course, be deflected by other gravitational fields. There’s gravitational lensing of gravitational waves, but reflecting them is almost impossible to do because they will lose energy. As a gravitational wave passes through a cloud of gas or whatever, it exerts energy on the cloud and therefore loses energy itself, but that effect is super duper really tiny. It’s not one that you really have to take into account as a working astronomer.

https://www.preposterousuniverse.com/podcast/2022/12/05/ama-december-2022/
Title: Re: why do a lot of people confuse between interference and diffraction?
Post by: alancalverd on 15/01/2023 15:15:11
- Interference (black spots in a light field) occurs because electromagnetism has positive and negative charges
- The Electric field of Electromagnetic radiation (eg light) oscillates positive and negative (electromagnetic radiation is a bipolar wave).
- Where positive polarity coincides with negative polarity, you get interference and "black spots" (if the amplitude is equal)
I really hope he didn't say that.
If you cancel the electric charge, what happens to the magnetic field? And since the intensity of the diffracted beam varies sinusoidally, does that mean that you can detect partial charges in the grey bits? Perhaps it was some other Sean Carroll, not a physicist. Common enough name I guess, but Irish education includes elementary physics to at least age 14, so it's probably a pseudonym.
Title: Re: why do a lot of people confuse between interference and diffraction?
Post by: evan_au on 15/01/2023 16:57:27
Quote
I really hope he didn't say that
The bit you quoted was my paraphrase/interpretation, applied to the topic of this thread (diffraction and interference).
What he actually said is in the podcast transcript with time stamp “0:53:41.2 SC”.

As I understand it, in an electromagnetic wave, if you reverse the current in the transmitting antenna (which you do twice every cycle), then the electromagnetic wave reverses phase by 180°. It is this phase reversal which allows cancellation in an electromagnetic field (eg in the dual-slit experiment).

But you can’t reverse gravitational mass, so I interpreted this as saying that if you have two sources of gravitational waves, they are unlikely to cancel.

I can almost get my head around gravitational waves, but I am struggling to visualise the interaction of two gravitational wave sources (eg if 1 distant source takes 2 paths around a black hole).
Title: Re: why do a lot of people confuse between interference and diffraction?
Post by: hamdani yusuf on 15/01/2023 17:39:38
because you said this about Dalton's model.
I know I said that.
So what?
When did pre-scientific age stop?
It doesn't matter unless it was "thousands of years" ago.
Any idea  that was believed for thousands of years (whether it was right or wrong) wasn't scientific.

Science is commonly said to have started around 1600 with people like Francis Bacon.
What's your reason to set that time frame?
Archimedes made some scientific discoveries.
Newton made some unscientific assertions.
Title: Re: why do a lot of people confuse between interference and diffraction?
Post by: hamdani yusuf on 15/01/2023 17:47:25
However, gravitational fields have only positive masses, so you don't get positives & negatives in gravitational waves (gravitational waves are a quadripolar wave).
- I presume from this that gravitational waves are extremely unlikely to produce cancellation? (maybe impossible?)
- But diffraction is certainly possible.
Consider this. A room filled with air at 1 atm pressure. A pair of loudspeakers are generating sound with the same frequency. The air pressure at any point in the room is always positive. Yet we can still observe interference effect.
Title: Re: why do a lot of people confuse between interference and diffraction?
Post by: evan_au on 15/01/2023 20:20:35
Quote from: hamdani yusuf
The air pressure at any point in the room is always positive. Yet we can still observe interference effect.
The constant air pressure is a constant offset to the (say) 440Hz tone put out by the speakers
- The interference is experienced at 440Hz, which does impose positive and negative variations on the constant air pressure.
- Air pressure could be considered a "scalar field", ie it has a value at every point in the room, but no direction.

Extrapolating that to gravitational waves...
- In my primitive understanding, gravitational waves impose a strain on spacetime at some frequency (say 440Hz).
- This strain has a direction, making it a vector field, rather than a scalar field(?), ie you need more variables to describe it
- So I guess if you had two gravitational wave sources at the same frequency, they could add at certain positions, and oppose each other at other positions (if the phase and direction of the strain were opposite)
- But more things have to line up to get cancellation at the frequency of interest: They would have to have equal amplitude, frequency and direction (?)

I am groping beyond the limits of my knowledge, here...
Title: Re: why do a lot of people confuse between interference and diffraction?
Post by: Bored chemist on 15/01/2023 21:28:00
What's your reason to set that time frame?
How did you come to the conclusion that I had set the time frame?

Archimedes made some scientific discoveries.
Like what?
I mean, I know he made some discoveries but which ones relied on this?
https://en.wikipedia.org/wiki/Scientific_method


Newton made some unscientific assertions.
So what?
Title: Re: why do a lot of people confuse between interference and diffraction?
Post by: Bored chemist on 15/01/2023 21:55:35
"Old fashioned textbooks of optics have separate chapters
for ‘interference’ and ‘diffraction’. But this distinction
is artificial. "
From
https://www.ias.ac.in/article/fulltext/reso/020/05/0389-0400
Title: Re: why do a lot of people confuse between interference and diffraction?
Post by: alancalverd on 15/01/2023 23:05:36
But you can’t reverse gravitational mass, so I interpreted this as saying that if you have two sources of gravitational waves, they are unlikely to cancel.
and there is the elephant dinosaur in the room that defecates over astrophysics!  A force that propagates at c but only sucks. I can't help thinking that we are only seeing half of the phenomenon.
Title: Re: why do a lot of people confuse between interference and diffraction?
Post by: alancalverd on 15/01/2023 23:08:37
What's your reason to set that time frame?
There is little if any written evidence of the scientific method prior to 1600. 
Title: Re: why do a lot of people confuse between interference and diffraction?
Post by: hamdani yusuf on 17/01/2023 06:26:05
"Old fashioned textbooks of optics have separate chapters
for ‘interference’ and ‘diffraction’. But this distinction
is artificial. "
From
https://www.ias.ac.in/article/fulltext/reso/020/05/0389-0400


How can the distinction artificial?
Here's a quote from the article.
Quote
Diffraction refers to light not traveling in a straight line and bending into the shadow.

It was observed by Grimaldi in Italy in the 17th century. Newton also studied diffraction and tried, unsuccessfully, to explain it in terms of his ‘corpuscles’ (particles) of light.
How can you say that it's the same as interference?
Title: Re: why do a lot of people confuse between interference and diffraction?
Post by: Bored chemist on 17/01/2023 08:26:29
How can you say that it's the same as interference?
Because the thing that causes the light to bend is interference.
Title: Re: why do a lot of people confuse between interference and diffraction?
Post by: Bored chemist on 17/01/2023 08:28:01
Quote from: hamdani yusuf on 15/01/2023 17:39:38
What's your reason to set that time frame?
How did you come to the conclusion that I had set the time frame?

Quote from: hamdani yusuf on 15/01/2023 17:39:38
Archimedes made some scientific discoveries.
Like what?
I mean, I know he made some discoveries but which ones relied on this?
https://en.wikipedia.org/wiki/Scientific_method
Please answer the questions.
Title: Re: why do a lot of people confuse between interference and diffraction?
Post by: hamdani yusuf on 17/01/2023 08:53:03
Historically, first diffraction grating was a bird feather, which is rightly called diffraction grating.
Quote
James Gregory (1638–1675) observed the diffraction patterns caused by a bird feather, which was effectively the first diffraction grating (in a natural form) to be discovered, about a year after Isaac Newton's prism experiments.[8] The first man-made diffraction grating was made around 1785 by Philadelphia inventor David Rittenhouse, who strung hairs between two finely threaded screws.[9][10] This was similar to notable German physicist Joseph von Fraunhofer's wire diffraction grating in 1821.[11][12] The principles of diffraction were discovered by Thomas Young[13] and Augustin-Jean Fresnel.[14][15] Using these principles, Fraunhofer was the first who used a diffraction grating to obtain line spectra and the first who measured the wavelengths of spectral lines with a diffraction grating.https://en.wikipedia.org/wiki/Diffraction_grating

But newer versions of diffraction gratings don't involve diffraction because they don't contain opaque object to create shadow area. They should be called interference grating instead, because that's the intended phenomenon to be produced by using them, either through refraction or reflection of incoming light beam.

Quote
(https://www.newport.com/mam/celum/celum_assets/Figure_78-Photonics_Handbook_800w.jpg)
Figure 2. Depictions of top-down view of diffraction grating showing groove pattern (left, top) and side view showing different groove profiles (left, bottom). Scanning electron microscope image of diffraction grating (right).

https://www.newport.com/n/diffraction-grating-physics
Title: Re: why do a lot of people confuse between interference and diffraction?
Post by: hamdani yusuf on 17/01/2023 08:59:00
Quote from: hamdani yusuf on 15/01/2023 17:39:38
What's your reason to set that time frame?
How did you come to the conclusion that I had set the time frame?

Quote from: hamdani yusuf on 15/01/2023 17:39:38
Archimedes made some scientific discoveries.
Like what?
I mean, I know he made some discoveries but which ones relied on this?
https://en.wikipedia.org/wiki/Scientific_method
Please answer the questions.
You can find it in the Wikipedia article.
Quote
"As regards his method, Aristotle is recognized as the inventor of scientific method because of his refined analysis of logical implications contained in demonstrative discourse, which goes well beyond natural logic and does not owe anything to the ones who philosophized before him." – Riccardo Pozzo[10]
Quote
Ibn al-Haytham (965–1039). A polymath, considered by some to be the father of modern scientific methodology, due to his emphasis on experimental data and reproducibility of its results.[11][12][13]
Title: Re: why do a lot of people confuse between interference and diffraction?
Post by: hamdani yusuf on 17/01/2023 10:10:43
How can you say that it's the same as interference?
Because the thing that causes the light to bend is interference.
Is it the same as refraction and reflection?
Title: Re: why do a lot of people confuse between interference and diffraction?
Post by: hamdani yusuf on 17/01/2023 10:27:10
https://www.ias.ac.in/article/fulltext/reso/020/05/0389-0400
Quote
The simplest problem is diffraction by a single straight edge, regarded as infinite

Quote
This was studied experimentally by the French physicist A Fresnel, around 1810. He was able to give a theory for the phenomenon. The resulting intensity distribution is shown in Figure 2. The pattern of intensity on the wall is similar at different distances, but the size of the pattern increases as the square root of the distance from the screen

Quote
Fresnel’s theory is based on Huygens’ idea that every point on a wavefront acts as a source of ‘secondary waves’. It is sketched in Figure 3a and outlined in Box 1. This theory was very successful in explaining the early experiments. However, the idea of secondary waves raises many questions. Why should a wave itself act as a source
of secondary waves? Why do we not include the secondary wave traveling in the backward direction from a given wavefront?

Quote
Around the same time, Young in England gave a different formulation in which the original wave falling on the screen travels unaltered into the region accessible to rays, An additional wave originates from the edge of the aperture, and enters the geometric shadow (Figure 3b). It might appear impossible to reconcile these two points of view, as different as the English and the French nations! Young did not give a mathematical formulation. Maggi (1890) and Sommerfeld’s student Rubinowicz (1912) were able to show the equivalence of these two very different looking pictures. When the expression given by the Fresnel theory was transformed using integration by parts, it precisely gave rise to the Young edge wave!
It's unfortunate that the derivation is not shown in the article.
Title: Re: why do a lot of people confuse between interference and diffraction?
Post by: alancalverd on 17/01/2023 14:31:30
As regards his method, Aristotle is recognized as the inventor of scientific method because of his refined analysis of logical implications contained in demonstrative discourse
Discourse is not experiment, nor does Aristotelian discourse seek counterexamples to the prevailing hypothesis.
Title: Re: why do a lot of people confuse between interference and diffraction?
Post by: alancalverd on 17/01/2023 14:33:33
Why should a wave itself act as a source
of secondary waves? Why do we not include the secondary wave traveling in the backward direction from a given wavefront?
(a) because it doesn't - this is just a geometric construction
(b) because there is no evidence for a backward wave

Never confuse a useful model or geometric construction with reality or underlying physics!
Title: Re: why do a lot of people confuse between interference and diffraction?
Post by: alancalverd on 17/01/2023 14:49:35
Quote from: Bored chemist on Today at 08:26:29
Quote from: hamdani yusuf on Today at 06:26:05
How can you say that it's the same as interference?
Because the thing that causes the light to bend is interference.
Is it the same as refraction and reflection?

I think BC has slightly oversimplified his response. Huygens says that forward propagation is the result of an infinity of infintesimal hemispherical wavelets interfering - the only nonzero resultant is "forwards". If you truncate the infinity, say on the left, there is no source to the left to provide any destructive interference so to the left of the primary beam you can see the resultant of all the interfering wavelets from the right.   
Title: Re: why do a lot of people confuse between interference and diffraction?
Post by: Bored chemist on 17/01/2023 18:08:54
newer versions of diffraction gratings don't involve diffraction
Yes they do.

If you don't want to believe in the definition of diffraction, that's your problem.
Title: Re: why do a lot of people confuse between interference and diffraction?
Post by: Bored chemist on 17/01/2023 18:20:41
As regards his method, Aristotle is recognized as the inventor of scientific method because of his refined analysis of logical implications contained in demonstrative discourse,
That's not got much to do with the current definition of science, has it?
This guy
https://www.huffpost.com/entry/6-things-aristotle-got-wr_b_5920840
was not a founder of a principle that says "start with an observation", was he?

Ibn al-Haytham  may have a better claim but it seems that society wasn't ready for science back then. He seems to have been the only one who understood how science works.
That slightly validates my point. You can't, in general, do science on your own.
It typically requires independent confirmation.


Some theories which survived for thousands years turned out to be false.

Were any of them scientific theories, or are you just wasting time on irrelevances?
Title: Re: why do a lot of people confuse between interference and diffraction?
Post by: alancalverd on 18/01/2023 00:15:52
Nobody has falsified the hypothesis that human sacrifice guarantees good weather. And look what happens when you stop - global warming!
Title: Re: why do a lot of people confuse between interference and diffraction?
Post by: hamdani yusuf on 18/01/2023 09:23:38
As regards his method, Aristotle is recognized as the inventor of scientific method because of his refined analysis of logical implications contained in demonstrative discourse
Discourse is not experiment, nor does Aristotelian discourse seek counterexamples to the prevailing hypothesis.
If you follow the debate between Aristotle and Plato, it was Aristotle who emphasized the importance of observation.
On the other hand, Plato thought that the truth can be obtained through contemplation alone.
Title: Re: why do a lot of people confuse between interference and diffraction?
Post by: hamdani yusuf on 18/01/2023 10:25:23
Nobody has falsified the hypothesis that human sacrifice guarantees good weather. And look what happens when you stop - global warming!
Stopping human sacrifice without causing significant change in the weather should be enough.
Title: Re: why do a lot of people confuse between interference and diffraction?
Post by: hamdani yusuf on 18/01/2023 10:32:00
(a) because it doesn't - this is just a geometric construction
A geometric construction must be based on a clear rule, whether or not it reflects the physical reality.
What's the rule for Huygen's geometric construction?
Title: Re: why do a lot of people confuse between interference and diffraction?
Post by: hamdani yusuf on 18/01/2023 10:34:22
(b) because there is no evidence for a backward wave
Is it predicted by Huygen's geometric construction?
Title: Re: why do a lot of people confuse between interference and diffraction?
Post by: hamdani yusuf on 18/01/2023 10:43:29
Were any of them scientific theories, or are you just wasting time on irrelevances?
Even newer scientific theories can still be false. Atomic theories from Dalton, Thomson, Rutherford, Bohr, to Schrodinger are scientific, based on observations made by the tools available for them.
Title: Re: why do a lot of people confuse between interference and diffraction?
Post by: hamdani yusuf on 18/01/2023 10:44:54
How can you say that it's the same as interference?
Because the thing that causes the light to bend is interference.
Is it the same as refraction and reflection?
I think it's different.
Title: Re: why do a lot of people confuse between interference and diffraction?
Post by: hamdani yusuf on 18/01/2023 11:10:20
Huygens says that forward propagation is the result of an infinity of infintesimal hemispherical wavelets interfering - the only nonzero resultant is "forwards".
What cancels the backwards wave?
Title: Re: why do a lot of people confuse between interference and diffraction?
Post by: hamdani yusuf on 18/01/2023 11:16:19
If you truncate the infinity, say on the left, there is no source to the left to provide any destructive interference so to the left of the primary beam you can see the resultant of all the interfering wavelets from the right.   
I truncated a microwave beam using an aluminum plate, and found no diffracted wave.
I got diffracted microwave when the aluminum was shaped as a grid to allow some transmission. Just in case you are wondering what I mean,

How does it explain non-diffractive edges?
You still have to tell us what these are.


I better show you.

video #4 Non-diffractive Obstacle
I have uploaded new video showing diffraction in microwave frequency.


Basically, the experiment result leads us to conclude that diffraction comes from the material blocking the microwave path. When the obstruction is opaque enough, we find no diffraction. It's similar to my experiment using laser showing non-diffractive obstruction.

This result is not widely known yet.
 

Title: Re: why do a lot of people confuse between interference and diffraction?
Post by: Bored chemist on 18/01/2023 12:45:44
A metal plate in a microwave field is potentially an emitter rather than "truncating" the beam.
Title: Re: why do a lot of people confuse between interference and diffraction?
Post by: alancalverd on 18/01/2023 12:59:13
What microwave wavelength were you using?

I remarked previously that there is an increase in received signal at about 1:23, when the aluminum plate is to the right of the centerline, and I don't remember offering an explanation. Something to worry about!

Title: Re: why do a lot of people confuse between interference and diffraction?
Post by: hamdani yusuf on 18/01/2023 13:51:02
What microwave wavelength were you using?

I remarked previously that there is an increase in received signal at about 1:23, when the aluminum plate is to the right of the centerline, and I don't remember offering an explanation. Something to worry about!


10.5 GHz. I mentioned this in some of earlier videos.
The signal is due to reflection. It's definitely not diffraction, because it occurs not in the shadow region of the plate.
Title: Re: why do a lot of people confuse between interference and diffraction?
Post by: hamdani yusuf on 18/01/2023 13:52:45
A metal plate in a microwave field is potentially an emitter rather than "truncating" the beam.

What makes the difference?
Title: Re: why do a lot of people confuse between interference and diffraction?
Post by: hamdani yusuf on 18/01/2023 13:57:04
I got diffracted microwave when the aluminum was shaped as a grid to allow some transmission.
Just in case you are wondering what I was talking about.
I've made a new video investigating diffraction of microwave using metamaterials. Previously, it was shown that an aluminum plate, which virtually reflects all of microwave power, didn't show any observable diffraction effect. This time, we tested if diffraction effect can be observed in a partially transparent obstacle which is constructed as metamaterial. The metamaterial obstacle is made from an array of aluminum stripes. There will be some follow up videos to eliminate some uncertainties around diffraction phenomenon.

Here it is. I hope you enjoy it.




Edge effect on diffraction of microwave.
Title: Re: why do a lot of people confuse between interference and diffraction?
Post by: alancalverd on 18/01/2023 16:04:06
The signal is due to reflection.
The plate is perpendicular to the beam axis, which means that any reflected energy will be sent back upstream, surely? And the pulse is very narrow compared with the cone of the primary beam

Full marks for an experiment that was sufficiently precise  to display an anomaly!
Title: Re: why do a lot of people confuse between interference and diffraction?
Post by: alancalverd on 18/01/2023 16:08:01
Stopping human sacrifice without causing significant change in the weather should be enough.
Human sacrifice pretty well died out in 1800, around the date that climate "scientists" consider to be the beginning of anthropogenic global warming. The facts speak for themselves.
Title: Re: why do a lot of people confuse between interference and diffraction?
Post by: alancalverd on 18/01/2023 16:14:08
What cancels the backwards wave?
The question is not answered, because it is physically meaningless! Maxwell calculates the Poynting propagation vector from physics, Huygens predicts how it will be affected by reflection, refraction and diffraction. As there is no backwards vector from Maxwell, there is no meaning in the back-projection of Huygens.
Title: Re: why do a lot of people confuse between interference and diffraction?
Post by: alancalverd on 18/01/2023 16:16:15
What's the rule for Huygen's geometric construction?
Every point on a wavefront is the origin of wavelets of the same phase and frequency.
Title: Re: why do a lot of people confuse between interference and diffraction?
Post by: hamdani yusuf on 19/01/2023 02:17:41
Stopping human sacrifice without causing significant change in the weather should be enough.
Human sacrifice pretty well died out in 1800, around the date that climate "scientists" consider to be the beginning of anthropogenic global warming. The facts speak for themselves.
Do you count holocaust as human sacrifice?
How do you define it?
Title: Re: why do a lot of people confuse between interference and diffraction?
Post by: hamdani yusuf on 19/01/2023 02:21:44
The signal is due to reflection.
The plate is perpendicular to the beam axis, which means that any reflected energy will be sent back upstream, surely? And the pulse is very narrow compared with the cone of the primary beam

Full marks for an experiment that was sufficiently precise  to display an anomaly!
At that distance, the primary beam is very strong, well over the maximum range of the receiver. That's why a very small portion of it can still be detected.
Title: Re: why do a lot of people confuse between interference and diffraction?
Post by: hamdani yusuf on 19/01/2023 02:25:26
Huygens predicts how it will be affected by reflection,
How is a wave reflected in Huygen's model?
What makes it different than transmission?
Title: Re: why do a lot of people confuse between interference and diffraction?
Post by: hamdani yusuf on 19/01/2023 02:28:44
What's the rule for Huygen's geometric construction?
Every point on a wavefront is the origin of wavelets of the same phase and frequency.
But every wavelet only consists of a half sphere, instead of full sphere, for no clear reason.
Title: Re: why do a lot of people confuse between interference and diffraction?
Post by: hamdani yusuf on 19/01/2023 06:58:50
Huygens predicts how it will be affected by reflection,
How is a wave reflected in Huygen's model?
What makes it different than transmission?
Let's not forget about my experiments in my other thread.
I've finished editing a new video on horizontally tilted diffraction. The first part is just fixing my old clips, while the last part contains new material. I'll share it here when I finished uploading it to my Youtube channel.
Many sources say that diffraction-interference pattern in a single slit experiment require the slit to be narrow, and comparable to the wavelength of the light wave. The word comparable in this context is not well defined.
Some of them also mention that the edges of the slit must be sharp.
My experiments will show that they are not necessarily true.
Here you are.
I just got an even stronger evidence that diffracted light is produced by the edges of the obstacle, instead of the space between those edges. The experiment involves linear polarization.
I've finally uploaded the video.
Title: Re: why do a lot of people confuse between interference and diffraction?
Post by: evan_au on 19/01/2023 09:19:49
Quote from: hamdani yusuf
half sphere, instead of full sphere, for no clear reason.
This accounts for the direction of propagation.

Huygens didn't know about electromagnetic waves, where the interaction of electric and magnetic fields produces a direction of propagation. So he represented the direction of propagation geometrically.

Title: Re: why do a lot of people confuse between interference and diffraction?
Post by: alancalverd on 19/01/2023 09:53:44
Do you count holocaust as human sacrifice?
How do you define it?
Sacrifice involves ritually killing a small number of chosen victims or volunteers in order to propitiate a supernatural being. The victim is usually a chicken or a sheep but for a really good harvest you need to kill a single human.

Mass murder is about politics, not agriculture. 
Title: Re: why do a lot of people confuse between interference and diffraction?
Post by: alancalverd on 19/01/2023 09:58:14
But every wavelet only consists of a half sphere, instead of full sphere, for no clear reason.


Because, for the umpteenth time, it is a model, not an explanation!

If you plot wind, tide and thrust vectors on a navigation chart, you can work out where the boat will be in an hour's time. It's a perfect model but it doesn't explain why the air, water or boat actually move at those particular velocities..
Title: Re: why do a lot of people confuse between interference and diffraction?
Post by: alancalverd on 19/01/2023 10:01:29
At that distance, the primary beam is very strong, well over the maximum range of the receiver. That's why a very small portion of it can still be detected.
Then why is the primary beam not detected at 1:21 or 1:25? Something rather special happens at 1:23.

A lot of scientific progress comes from the investigation of anomalies, and we must be grateful to experimental geniuses like yourself for finding them. The phenomenon occurs over a movement commensurate with your 2.8 cm wavelength, which is why I asked for confirmation of that value.
Title: Re: why do a lot of people confuse between interference and diffraction?
Post by: hamdani yusuf on 19/01/2023 22:51:19
Then why is the primary beam not detected at 1:21 or 1:25? Something rather special happens at 1:23.
You're confusing because you skipped 0:24
Title: Re: why do a lot of people confuse between interference and diffraction?
Post by: hamdani yusuf on 20/01/2023 14:39:28
Huygens predicts how it will be affected by reflection,
How is a wave reflected in Huygen's model?
What makes it different than transmission?
Here's how reflection is explained using antenna model, starting at 5:45.

This is the model I used to come up with blocking mechanism, and designed polarization twister.
Title: Re: why do a lot of people confuse between interference and diffraction?
Post by: alancalverd on 20/01/2023 18:43:39
Then why is the primary beam not detected at 1:21 or 1:25? Something rather special happens at 1:23.
You're confusing because you skipped 0:24
No, I watched what happened as you slid the plate from one side to the other. Fairly smoothly and continuously, And something happened at 1:23.
Title: Re: why do a lot of people confuse between interference and diffraction?
Post by: hamdani yusuf on 21/01/2023 15:53:51
Then why is the primary beam not detected at 1:21 or 1:25? Something rather special happens at 1:23.
You're confusing because you skipped 0:24
No, I watched what happened as you slid the plate from one side to the other. Fairly smoothly and continuously, And something happened at 1:23.
At 1:21, the plate completely blocked the microwave.
At 1:25, the plate doesn't interact with microwave at all.
Title: Re: why do a lot of people confuse between interference and diffraction?
Post by: hamdani yusuf on 21/01/2023 16:02:32
Huygens predicts how it will be affected by reflection,
How is a wave reflected in Huygen's model?
What makes it different than transmission?
Here's how reflection is explained using antenna model, starting at 5:45.

This is the model I used to come up with blocking mechanism, and designed polarization twister.
There's a substantial difference between the antenna array model and Huygen's model. In antenna model, the wavelets are always originated from an antenna, which must consist of charged particles. On the other hand, Huygen's principle says that the wavelets can originate from vacuum of space.
Title: Re: why do a lot of people confuse between interference and diffraction?
Post by: alancalverd on 21/01/2023 16:03:05
And at 1:23 there is a signal at the receiver.

Don't be modest - you have demonstrated a phenomenon you didn't expect, by very careful experimentation! That is the best science.
Title: Re: why do a lot of people confuse between interference and diffraction?
Post by: William Hardy on 21/01/2023 17:45:55
What I understand is diffraction is bending of a single wave of light around an obstacle,
whereas interference is the superimposition of two waves to form one wave.
* difference_between_diffraction_and_interference.jpg (167.77 kB . 590x312 - viewed 1396 times)
You can find some detailed differences here: https://dewwool.com/difference-between-diffraction-and-interference/
Title: Re: why do a lot of people confuse between interference and diffraction?
Post by: hamdani yusuf on 22/01/2023 02:32:22
What I understand is diffraction is bending of a single wave of light around an obstacle,
whereas interference is the superimposition of two waves to form one wave.
* difference_between_diffraction_and_interference.jpg (167.77 kB . 590x312 - viewed 1396 times)
You can find some detailed differences here: https://dewwool.com/difference-between-diffraction-and-interference/

I found some inconsistencies in the article, where what it says in one part contradicts what it says in another part. I'll point out one by one in following posts.
Title: Re: why do a lot of people confuse between interference and diffraction?
Post by: Bored chemist on 22/01/2023 09:45:38
But every wavelet only consists of a half sphere, instead of full sphere, for no clear reason.
It's interesting to note that you can do essentially the same maths with water waves as with light waves.
And you get comparable results- diffraction, refraction interference etc.

If you are trying to explain the lack of a "backwards" half of the wavelet, you can invoke the conservation or energy and momentum. There is a "clear reason".

What stops you doing the same with light?



Huygens didn't know about electromagnetic waves
No.
In his day they would have said that they were waves in the luminiferous aether.

The stuff doesn't exist but if you just want to calculate the propagation of light waves, the aether model works.

Title: Re: why do a lot of people confuse between interference and diffraction?
Post by: hamdani yusuf on 22/01/2023 14:38:47
If you are trying to explain the lack of a "backwards" half of the wavelet, you can invoke the conservation or energy and momentum. There is a "clear reason".

What stops you doing the same with light?

Because they are different. The media of mechanical waves have inertial mass, while the vacuum of space doesn't.

Title: Re: why do a lot of people confuse between interference and diffraction?
Post by: hamdani yusuf on 22/01/2023 14:44:21
There's a substantial difference between the antenna array model and Huygen's model. In antenna model, the wavelets are always originated from an antenna, which must consist of charged particles. On the other hand, Huygen's principle says that the wavelets can originate from vacuum of space.
Every derivation of single slit diffraction using Huygen's principle assumes that the obstacles are completely opaque. But my experiments show that diffraction requires partial opacity/transparency.
Title: Re: why do a lot of people confuse between interference and diffraction?
Post by: hamdani yusuf on 22/01/2023 14:51:42
What I understand is diffraction is bending of a single wave of light around an obstacle,
whereas interference is the superimposition of two waves to form one wave.
* difference_between_diffraction_and_interference.jpg (167.77 kB . 590x312 - viewed 1396 times)
You can find some detailed differences here: https://dewwool.com/difference-between-diffraction-and-interference/

I found some inconsistencies in the article, where what it says in one part contradicts what it says in another part. I'll point out one by one in following posts.

Here's what I found in the opening paragraph.
Quote
Interference can be defined as the combination of two or more waves.
And this is from the table.
Quote
It occurs due to the superposition of two waves coming from two different coherent sources.
Title: Re: why do a lot of people confuse between interference and diffraction?
Post by: hamdani yusuf on 22/01/2023 15:02:49
Quote
They are of two types:
•Fresnel’s diffraction: If the source of light and the screen are kept at a finite distance from the diffracting obstacle then Fresnel’s diffraction occurs.
•Fraunhofer diffraction: Fraunhofer diffraction can be observed if the source of light and the screen are both at an infinite distance from the diffracting obstacle.    

There are two combination of distances left uncovered. Finite source but infinite screen, and infinite source but finite screen.
Title: Re: why do a lot of people confuse between interference and diffraction?
Post by: hamdani yusuf on 22/01/2023 15:07:50
Quote
They are (interference) also of two types:
•Constructive interference: Constructive interference is defined as the superposition of two waves to get a greater amplitude like we use speakers to get the greater amplitude of sounds.
•Destructive interference: When two waves in opposite polarity or direction superimpose on each other in a way that they cancel each other, destructive
Both must occur in the same pattern to call it an interference pattern.
Title: Re: why do a lot of people confuse between interference and diffraction?
Post by: hamdani yusuf on 22/01/2023 15:26:26
Quote
Fringes are of varying width in diffraction.   The fringe widths are constant in interference.
Many sources mentioning this difference offer single slit experiment as an example of diffraction, and double slit experiment as an example of interference. This is one of the most common misconceptions.

The formula commonly found in the explanation of single slit experiment indicates constant fringe width. The only fringe with different width is the center bright, which is twice as wide as the others.
In double slit experiment where the width of the slits are not much different than the distance between them, it's possible for the minima of the one slit "envelope" overlap with maxima of double slit interference pattern, producing dark fringes much wider than the other. The envelope can also reduce the width of some bright fringe.
Title: Re: why do a lot of people confuse between interference and diffraction?
Post by: Bored chemist on 22/01/2023 17:41:47
The media of mechanical waves have inertial mass, while the vacuum of space doesn't.
Light carries energy and momentum.
The conservation laws mean that it can't suddenly go backwards for no reason.
Title: Re: why do a lot of people confuse between interference and diffraction?
Post by: hamdani yusuf on 23/01/2023 02:24:21
I answered another question in Quora, Why are there dark fringes in a single slit experiment?
https://www.quora.com/Why-are-there-dark-fringes-in-a-single-slit-experiment

Because in a single slit apperture, there are two edges. They can act as light sources which then interfere constructively and destructively to create dark and bright fringes on a screen. Double slit apperture has 4 edges, hence there are more light sources which can interfere with each other.

(https://qph.fs.quoracdn.net/main-qimg-dc85396cec36db5b1b5a9d84e6debb80)
Top picture is the pattern produced by the same double slit aperture as the bottom picture, but one slit is covered.


Many sources call the top picture as single slit diffraction pattern, while the bottom picture is called double slit interference pattern. In both cases, the intensity of the fringes vary. It's contrary to the statements in the link.

Quote
diffraction :
Bright fringes are of the same intensity.   

interference :
The intensity of successive fringes is not the same. The intensity decreases with fringes.
Title: Re: why do a lot of people confuse between interference and diffraction?
Post by: hamdani yusuf on 23/01/2023 02:47:19
Here's the next difference.
Quote
Obstacles are  necessary for diffraction    

An obstacle is not necessary for direction (interference) 
There's a typo, but apart from that,  the statement is correct.
Title: Re: why do a lot of people confuse between interference and diffraction?
Post by: hamdani yusuf on 23/01/2023 02:55:22
Quote
diffraction :
The contrast between maxima and minima is poor   

interference :
The contrast between maxima and minima is certainly good

It depends on the light source and the material of the obstacle.
Title: Re: why do a lot of people confuse between interference and diffraction?
Post by: hamdani yusuf on 23/01/2023 03:06:07
Quote
Wave propagation direction:   
It changes after diffraction   
It does not change after superposition
I don't know where it came from. Why superposition slips into the table, instead of interference.
But in case of double slit and single slit experiment, they both change the direction of light propagation.
Title: Re: why do a lot of people confuse between interference and diffraction?
Post by: hamdani yusuf on 23/01/2023 05:19:15
The media of mechanical waves have inertial mass, while the vacuum of space doesn't.
Light carries energy and momentum.
The conservation laws mean that it can't suddenly go backwards for no reason.
Can it go sideways for no reason?
Title: Re: why do a lot of people confuse between interference and diffraction?
Post by: Bored chemist on 23/01/2023 08:47:23
The media of mechanical waves have inertial mass, while the vacuum of space doesn't.
Light carries energy and momentum.
The conservation laws mean that it can't suddenly go backwards for no reason.
Can it go sideways for no reason?
On average, it doesn't.
Title: Re: why do a lot of people confuse between interference and diffraction?
Post by: alancalverd on 23/01/2023 09:18:17
Because in a single slit apperture, there are two edges. They can act as light sources
This is clearly nonsense. I have a comb, with hundreds of edges. It does not emit light, due to some ridiculous conservation law, but it can act as a diffraction grating.
Title: Re: why do a lot of people confuse between interference and diffraction?
Post by: Bored chemist on 23/01/2023 10:44:35
It does not emit light,
How can you see it?
Title: Re: why do a lot of people confuse between interference and diffraction?
Post by: alancalverd on 23/01/2023 11:42:34
Not at all, in a dark room. Unlike anything else I would call a light source.

Indeed, as it is a black comb, I tend to infer its existence from a periodic absence of white light.
Title: Re: why do a lot of people confuse between interference and diffraction?
Post by: hamdani yusuf on 23/01/2023 14:46:25
Because in a single slit apperture, there are two edges. They can act as light sources
This is clearly nonsense. I have a comb, with hundreds of edges. It does not emit light, due to some ridiculous conservation law, but it can act as a diffraction grating.
Why do you accept that a point in the vacuum of space can act as source of the wavelets?
Title: Re: why do a lot of people confuse between interference and diffraction?
Post by: alancalverd on 23/01/2023 16:58:04
Only a fool would do so. Huygens is geometry, not physics.
Title: Re: why do a lot of people confuse between interference and diffraction?
Post by: Bored chemist on 23/01/2023 19:02:24
Because in a single slit apperture, there are two edges. They can act as light sources
This is clearly nonsense. I have a comb, with hundreds of edges. It does not emit light, due to some ridiculous conservation law, but it can act as a diffraction grating.
Why do you accept that a point in the vacuum of space can act as source of the wavelets?
I might accept that the EM field at a point in space is a source of wavelets.
Title: Re: why do a lot of people confuse between interference and diffraction?
Post by: hamdani yusuf on 24/01/2023 03:24:40
Only a fool would do so. Huygens is geometry, not physics.
There are other cases where the geometry doesn't reflect physical objects, such us epicycles to describe planetary motion. They can produce precise predictions, but require ad hoc assumptions when the objects or situations are changed.
IMO, they will be replaced by some better models.
Title: Re: why do a lot of people confuse between interference and diffraction?
Post by: alancalverd on 24/01/2023 10:56:47
So far, nobody has produced a better geometric construction than Huygens for predicting diffraction.  If it ain't busted, don't fix it.

The problem with epicycles is that they purported to be an explanation, not a predictive construction, and implied a mechanical structure for which there was no evidence. We still use a sort of epicycle mathematics to make remarkably accurate tide tables but they are under continuous revision as everyone realises that there are nth order effects that are easier to measure than to calculate..   
Title: Re: why do a lot of people confuse between interference and diffraction?
Post by: hamdani yusuf on 25/01/2023 11:33:25
So far, nobody has produced a better geometric construction than Huygens for predicting diffraction.  If it ain't busted, don't fix it.

The problem with epicycles is that they purported to be an explanation, not a predictive construction, and implied a mechanical structure for which there was no evidence. We still use a sort of epicycle mathematics to make remarkably accurate tide tables but they are under continuous revision as everyone realises that there are nth order effects that are easier to measure than to calculate..   
How does it produce reflection?
Title: Re: why do a lot of people confuse between interference and diffraction?
Post by: alancalverd on 25/01/2023 11:49:17
Every point at which the wavefront is reflected is treated as the source of the reflected wavelets.

Remember, as always, that Huygens is a geometric construction that mimics what happens, not an explanation of how it happens.
Title: Re: why do a lot of people confuse between interference and diffraction?
Post by: hamdani yusuf on 26/01/2023 04:08:05
Every point at which the wavefront is reflected is treated as the source of the reflected wavelets.

Remember, as always, that Huygens is a geometric construction that mimics what happens, not an explanation of how it happens.
Can it explain reflection by a polarizer?
My experiment shows that incoming microwave is almost completely reflected by the polarizer, despite the gap between conductors in the polarizer grid are much larger than the diameter of the conductors themselves.
Title: Re: why do a lot of people confuse between interference and diffraction?
Post by: alancalverd on 26/01/2023 07:58:33
Can it explain reflection by a polarizer?
Once again, it is a geometric construction, not an explanation.
Title: Re: why do a lot of people confuse between interference and diffraction?
Post by: hamdani yusuf on 26/01/2023 09:29:16
Can it explain reflection by a polarizer?
Once again, it is a geometric construction, not an explanation.
How do you geometrically construct the result of reflection by a polarizer?
Title: Re: why do a lot of people confuse between interference and diffraction?
Post by: hamdani yusuf on 02/06/2023 14:12:20
I asked Google Bard about the difference between diffraction and interference of light. IMO, the answer is better than many other sources I quoted in earlier posts.

what are the differences between diffraction and interference of light?
Quote
Diffraction and interference are both wave phenomena that occur when light passes through an opening or around an obstacle. However, there are some key differences between the two.

Diffraction is the bending of waves around an obstacle or through an opening. This occurs because waves can interfere with themselves, and when they do, they can create new patterns of light and dark. Diffraction is a property of all waves, including light, sound, and water waves.
Interference is the superposition of two or more waves. When two waves interfere, they can either add together to create a brighter wave (constructive interference) or cancel each other out to create a darker wave (destructive interference). Interference is a property of waves that have the same frequency and wavelength.

Diffraction   
Bending of waves around an obstacle or through an opening   
Property of all waves   
Creates new patterns of light and dark   

Interference
Superposition of two or more waves
Property of waves that have the same frequency and wavelength
Can create brighter or darker waves
Title: Re: why do a lot of people confuse between interference and diffraction?
Post by: hamdani yusuf on 02/06/2023 14:21:07
Property of all waves
This answer made me ask follow up question. Does diffraction occur in guitar string wave?
Quote
Yes, diffraction occurs in guitar string waves. When a guitar string is plucked, it vibrates and produces a wave that travels along the string. This wave is a longitudinal wave, which means that the particles of the string move back and forth in the same direction that the wave is traveling.

As the wave travels along the string, it diffracts around the bridge and nut, which are the two points where the string is attached to the guitar. This diffraction causes the wave to spread out and create a wider pattern of vibration.

The amount of diffraction that occurs depends on the wavelength of the wave and the size of the opening that the wave is passing through. The shorter the wavelength and the smaller the opening, the more diffraction will occur.

Diffraction is an important phenomenon that affects the sound of a guitar. It helps to create the wide, rich sound that is characteristic of a guitar.

I was interested about the waving guitar string, not the sound caused by vibration of the string. Thus I modified the question. Does diffraction occur in guitar string wave in vacuum?
Quote

No, diffraction does not occur in guitar string waves in a vacuum. Diffraction is the bending of waves around an obstacle or through an opening. This occurs because waves can interfere with themselves, and when they do, they can create new patterns of light and dark. However, sound waves, such as the waves created by a guitar string, cannot travel through a vacuum. Sound waves require a medium to travel through, such as air or water. Without a medium, sound waves cannot propagate and therefore cannot diffract.

However, if the guitar string were in a medium, such as air, then diffraction would occur. For example, if the guitar string were plucked in a room, the sound waves would diffract around the walls of the room and create a pattern of sound waves in the room. This pattern would be similar to the interference pattern created by light waves.
It doesn't seem to notice that its answer contradicts its previous claim.
Title: Re: why do a lot of people confuse between interference and diffraction?
Post by: varsigma on 04/06/2023 01:21:08
Two distinct phenomena shouldn't be confused just because they often come together. As long as they can occur separately, the distinction should be kept clear.
I think, in the age of quantum information science, that it doesn't pay to get too attached to a particular view or picture.
It shouldn't be something that becomes . . . precious to you.

For instance, the Aharonov-Bohm effect. Is this distinct from an interference effect? A diffraction effect? Is it something  else?

I know a guy (who probably knows a guy) who appears to believe it's about whether you can put things in bottles--a test of physicality. He's looking for one bottle to rule them all. pfft.

p.s. ChatGPT, or Bard, is bound to contradict itself, it isn't able to understand what that is.
Title: Re: why do a lot of people confuse between interference and diffraction?
Post by: hamdani yusuf on 04/06/2023 15:01:30
For instance, the Aharonov-Bohm effect. Is this distinct from an interference effect? A diffraction effect? Is it something  else?
https://en.m.wikipedia.org/wiki/Aharonov%E2%80%93Bohm_effect
Quote
The Aharonov?Bohm effect, sometimes called the Ehrenberg?Siday?Aharonov?Bohm effect, is a quantum mechanical phenomenon in which an electrically charged particle is affected by an electromagnetic potential (φ, A), despite being confined to a region in which both the magnetic field B and electric field E are zero.[1] The underlying mechanism is the coupling of the electromagnetic potential with the complex phase of a charged particle's wave function, and the Aharonov?Bohm effect is accordingly illustrated by interference experiments.
The experiment shows an additional effect on top of diffraction and interference effects.
Basically, we can add other effects as well, such as reflection and polarization to our designed experiments. But they should not confuse us about the definitions and boundaries of each effects.
Title: Re: why do a lot of people confuse between interference and diffraction?
Post by: hamdani yusuf on 04/06/2023 15:05:32
I think, in the age of quantum information science, that it doesn't pay to get too attached to a particular view or picture.
It shouldn't be something that becomes . . . precious to you.
Machine learning is said to be statistics on steroids. The more frequent a false information is trained to the AI model, the harder it gets to correct it later. Hence there's an incentive to feed them the correct information in the first place.
Title: Re: why do a lot of people confuse between interference and diffraction?
Post by: hamdani yusuf on 04/06/2023 15:08:40
p.s. ChatGPT, or Bard, is bound to contradict itself, it isn't able to understand what that is.
There's a technique called self reflection, which has been shown to significantly improve answers of AI models.
Title: Re: why do a lot of people confuse between interference and diffraction?
Post by: alancalverd on 04/06/2023 16:16:44
When a guitar string is plucked, it vibrates and produces a wave that travels along the string. This wave is a longitudinal wave, which means that the particles of the string move back and forth in the same direction that the wave is traveling.
Wrong, obviously. Not a good starting point for an essay.
Title: Re: why do a lot of people confuse between interference and diffraction?
Post by: alancalverd on 04/06/2023 16:17:38
There's a technique called self reflection, which has been shown to significantly improve answers of AI models.
However much you polish a turd, you end up with a turd.
Title: Re: why do a lot of people confuse between interference and diffraction?
Post by: hamdani yusuf on 05/06/2023 13:18:48
There's a technique called self reflection, which has been shown to significantly improve answers of AI models.
However much you polish a turd, you end up with a turd.
Have you read the research paper on it? It shows statistically significant improvement. What's your explanation?
Title: Re: why do a lot of people confuse between interference and diffraction?
Post by: alancalverd on 05/06/2023 17:20:01
My daughter-in-law is a civil engineer with an interest in sewage. I'll ask her about the statistics of turds, both raw and polished.
Title: Re: why do a lot of people confuse between interference and diffraction?
Post by: paul cotter on 05/06/2023 19:16:18
That's an interesting snippet of off topic information. Your good daughter in law must then be familiar with sir Joseph Bazelgette. It' a small world in terms of degrees of separation. PS I hope your ticker is behaving itself-you must feel like fish out of water with your license suspended. And Hamdani, apologies for any resultant distraction.
Title: Re: why do a lot of people confuse between interference and diffraction?
Post by: alancalverd on 05/06/2023 22:58:26
If you are ever in London, I can highly recommend a visit to Bazalgette's Crossness Pumping Station - amazing Victorian engineering and decorative ironwork and tiles worthy of a cathedral, being restored by brilliant and talented enthusiasts. I took a guided tour last summer beginning with a walk around City public toilets and the history of Cockney poo since the Romans, a lecture on the Great Stink (1858, not the current government) then a boat trip along the Thames to the Station and Museum of Toilets. Marvellous day out!

Interesting that Bazalgette's descendants now produce TV shows like Big Brother - they must have sh1t in their DNA!
Title: Re: why do a lot of people confuse between interference and diffraction?
Post by: hamdani yusuf on 06/06/2023 07:33:39
My daughter-in-law is a civil engineer with an interest in sewage. I'll ask her about the statistics of turds, both raw and polished.
Plants can turn turds into fruits.
Title: Re: why do a lot of people confuse between interference and diffraction?
Post by: hamdani yusuf on 06/06/2023 14:21:17
When a guitar string is plucked, it vibrates and produces a wave that travels along the string. This wave is a longitudinal wave, which means that the particles of the string move back and forth in the same direction that the wave is traveling.
Wrong, obviously. Not a good starting point for an essay.
The AI model only learned from selected curated text sources. It can't cross check with real experimental observations where it can play around with.
Perhaps the data it was trained with don't adequately explain about wave propagation on a string, which is why when I asked about it, I get an explanation about sound wave instead.
Title: Re: why do a lot of people confuse between interference and diffraction?
Post by: alancalverd on 06/06/2023 16:04:31
Wave propagation in a string depends on whether you waggle one end of a long, soft, heavy rope, so you propagate sine waves along its length, or pluck the middle of a short, stiff, light wire whose ends are fixed, so you can only produce standing waves with nodes at the ends.

Apparently AI doesn't  know what a guitar is, or the difference between pluck and shake. Not that it matters at all, because anyone who wants to play a guitar knows exactly what they mean, and anyone who understands physics can spot bullshit. As usual, the contribution of AI is negative.
Title: Re: why do a lot of people confuse between interference and diffraction?
Post by: hamdani yusuf on 07/06/2023 14:51:07
Wave propagation in a string depends on whether you waggle one end of a long, soft, heavy rope, so you propagate sine waves along its length, or pluck the middle of a short, stiff, light wire whose ends are fixed, so you can only produce standing waves with nodes at the ends.
If the string is long enough, the wave doesn't have to be a standing wave.
Title: Re: why do a lot of people confuse between interference and diffraction?
Post by: alancalverd on 08/06/2023 17:36:39
It does on a guitar, because the ends are fixed. You might somehow induce a travelling or compression wave with a magnetic excitation of a steel string but the resulting frequency (MHz) will be far too high to contribute to the "wide rich sound...."
Title: Re: why do a lot of people confuse between interference and diffraction?
Post by: hamdani yusuf on 09/06/2023 08:34:37
(https://s3-us-west-2.amazonaws.com/courses-images/wp-content/uploads/sites/2952/2018/01/31201322/CNX_UPhysics_16_05_HardSoft.jpg)
Quote
(a) One end of a string is fixed so that it cannot move. A wave propagating on the string, encountering this fixed boundary condition, is reflected 180o(π rad) out of phase with respect to the incident wave.

(b) One end of a string is tied to a solid ring of negligible mass on a frictionless lab pole, where the ring is free to move. A wave propagating on the string, encountering this free boundary condition, is reflected in phase 0o(0 rad)  with respect to the wave.
I mentioned these merely to show a counter example of Bard's claim that all waves experience diffraction.
Title: Re: why do a lot of people confuse between interference and diffraction?
Post by: alancalverd on 09/06/2023 08:48:23
What has reflection to do with diffraction?
Title: Re: why do a lot of people confuse between interference and diffraction?
Post by: Bored chemist on 09/06/2023 08:48:49
It does on a guitar, because the ends are fixed. You might somehow induce a travelling or compression wave with a magnetic excitation of a steel string but the resulting frequency (MHz) will be far too high to contribute to the "wide rich sound...."
I'd like to see the maths on that. Take the average guitar string as having a length of 1 metre and the speed of sound in steel to be 5km/sec
Title: Re: why do a lot of people confuse between interference and diffraction?
Post by: Bored chemist on 09/06/2023 08:50:01
I mentioned these merely to show a counter example of Bard's claim that all waves experience diffraction.
The pictures do not show waves.
Title: Re: why do a lot of people confuse between interference and diffraction?
Post by: hamdani yusuf on 09/06/2023 10:14:02
What has reflection to do with diffraction?
It does on a guitar, because the ends are fixed.
Title: Re: why do a lot of people confuse between interference and diffraction?
Post by: hamdani yusuf on 09/06/2023 10:16:22
I mentioned these merely to show a counter example of Bard's claim that all waves experience diffraction.
The pictures do not show waves.
What do you think they show?
Title: Re: why do a lot of people confuse between interference and diffraction?
Post by: Bored chemist on 09/06/2023 12:28:29
I mentioned these merely to show a counter example of Bard's claim that all waves experience diffraction.
The pictures do not show waves.
What do you think they show?
Pulses.
Title: Re: why do a lot of people confuse between interference and diffraction?
Post by: alancalverd on 09/06/2023 14:11:28
It does on a guitar, because the ends are fixed. You might somehow induce a travelling or compression wave with a magnetic excitation of a steel string but the resulting frequency (MHz) will be far too high to contribute to the "wide rich sound...."
I'd like to see the maths on that. Take the average guitar string as having a length of 1 metre and the speed of sound in steel to be 5km/sec
So the lowest frequency that might propagate as a reflected longitudinal wave is 15 times the fundamental of the highest-pitched open string - that's not "rich" but "squeaky", and the harmonics are mostly beyond human auditory range.

Longitudinal standing compression waves are generated in wind instruments, not stringed ones.
Title: Re: why do a lot of people confuse between interference and diffraction?
Post by: hamdani yusuf on 09/06/2023 15:05:24
Longitudinal standing compression waves are generated in wind instruments, not stringed ones.
It can also be generated in a slinky.
Title: Re: why do a lot of people confuse between interference and diffraction?
Post by: hamdani yusuf on 09/06/2023 15:06:37
Pulses.
How many oscillations are needed to produce a wave?
Title: Re: why do a lot of people confuse between interference and diffraction?
Post by: alancalverd on 09/06/2023 22:50:43
One. The difference is that an oscillation has both positive and negative excursions about the equilibrium, whereas a pulse is a unidirectional displacement followed by a return to equilibrium.
Title: Re: why do a lot of people confuse between interference and diffraction?
Post by: hamdani yusuf on 10/06/2023 15:42:01
One. The difference is that an oscillation has both positive and negative excursions about the equilibrium, whereas a pulse is a unidirectional displacement followed by a return to equilibrium.
If there are more than 1 oscillations, would it make any difference?
Title: Re: why do a lot of people confuse between interference and diffraction?
Post by: alancalverd on 10/06/2023 16:06:25
No.
Title: Re: why do a lot of people confuse between interference and diffraction?
Post by: hamdani yusuf on 10/06/2023 23:34:26
No.

It means that a string or a slinky can produce traveling wave which doesn't diffract.
Title: Re: why do a lot of people confuse between interference and diffraction?
Post by: alancalverd on 11/06/2023 18:01:26
It can't diffract because it is constrained to travel along the length of the string. If you "bend" the string around a rigid fixture, the wave may reflect from the fixture but won't go around it, but if you bend the string with a flexible post, the attenuated wave will travel beyond the post.

So what?   
Title: Re: why do a lot of people confuse between interference and diffraction?
Post by: hamdani yusuf on 12/06/2023 12:25:55
if you bend the string with a flexible post, the attenuated wave will travel beyond the post.
Will you call it diffraction?
Title: Re: why do a lot of people confuse between interference and diffraction?
Post by: alancalverd on 12/06/2023 14:34:32
To the same extent that any other wave is diffracted by an obstacle.
Title: Re: why do a lot of people confuse between interference and diffraction?
Post by: hamdani yusuf on 14/06/2023 14:56:52
If the string behind the flexible post is in line with the string in front of it, will it still be called diffraction?
Title: Re: why do a lot of people confuse between interference and diffraction?
Post by: alancalverd on 14/06/2023 23:15:01
Why not?
Title: Re: why do a lot of people confuse between interference and diffraction?
Post by: hamdani yusuf on 15/06/2023 03:33:52
Why not?
Because we need to use definition of words consistently, or at least explicitly, in order to avoid miscommunication.

It can't diffract because it is constrained to travel along the length of the string. If you "bend" the string around a rigid fixture, the wave may reflect from the fixture but won't go around it, but if you bend the string with a flexible post, the attenuated wave will travel beyond the post.

So what?   
Title: Re: why do a lot of people confuse between interference and diffraction?
Post by: alancalverd on 15/06/2023 13:13:32
Exactly.  "Flexible" has its usual meaning, and "bend" is in inverted commas for a good reason - I didn't specify any angle.
Title: Re: why do a lot of people confuse between interference and diffraction?
Post by: hamdani yusuf on 16/06/2023 15:31:16
Do you agree that perfect reflectors can avoid diffraction?
Title: Re: why do a lot of people confuse between interference and diffraction?
Post by: alancalverd on 17/06/2023 00:09:16
By definition, a perfect reflector is one for which the angle of incidence equals the angle of reflection for all wavelengths, which is obviously not the case for diffraction.
Title: Re: why do a lot of people confuse between interference and diffraction?
Post by: hamdani yusuf on 09/12/2023 15:44:49
After a while, I asked Google Bard again about the difference between diffraction and interference. This time I specifically mentioned light.

What's the difference between diffraction and interference of light?
Quote

While both diffraction and interference are phenomena related to light waves, they have key differences in their origin and characteristics.

Diffraction:

Cause: Occurs when a light wave encounters an obstacle or opening.
Mechanism: The edges of the obstacle or opening act as sources of secondary wavelets, which spread out and interfere with each other.
Pattern: The resulting pattern consists of bright and dark bands, with the intensity and width of the bands varying depending on the size and shape of the obstacle.
Examples: Rainbows, the diffraction pattern of a single slit or double slit, and the formation of Fresnel zones.

Interference:

Cause: Occurs when two or more coherent light waves overlap.
Mechanism: The crests and troughs of the waves combine, leading to constructive interference (where the amplitudes add up) and destructive interference (where the amplitudes cancel out).
Pattern: The resulting pattern consists of alternating bright and dark bands with equal widths and high contrast.
Examples: Young's double-slit experiment, thin-film interference in soap bubbles and oil slicks, and the formation of Newton's rings.
It doesn't seem to improve much from the last time I asked. Bard's alternative answers don't give much better answers.
Title: Re: why do a lot of people confuse between interference and diffraction?
Post by: alancalverd on 09/12/2023 17:35:53
It's all perfectly true, if a bit simplistic. What's your problem?
Title: Re: why do a lot of people confuse between interference and diffraction?
Post by: Origin on 09/12/2023 18:49:12
It doesn't seem to improve much from the last time I asked. Bard's alternative answers don't give much better answers.
Wow, it's almost like Bard is just a chat bot, I wonder why that is?

In answer to the OP, I would say the problem is 'a lot of people' don't care enough to take the time to find out the difference or they have reading comprehension issues.
Title: Re: why do a lot of people confuse between interference and diffraction?
Post by: hamdani yusuf on 10/12/2023 11:45:04
It's all perfectly true, if a bit simplistic. What's your problem?
It's a bit surprising that you can't find the problems in Bard's answer above.

The resulting pattern consisting of bright and dark bands is the defining characteristics of interference. Diffraction without interference doesn't produce such patterns.

It says "the intensity and width of the bands varying depending on the size and shape of the obstacle."  That's also characteristics of interference.

Rainbow isn't generally said to be a result of diffraction. It's a result of dispersion.
Title: Re: why do a lot of people confuse between interference and diffraction?
Post by: hamdani yusuf on 10/12/2023 12:22:02
When specifically asked to describe the difference between two phenomena, it should mention examples which can emphasize the difference, instead of mixing them up. It should mention some examples involving one phenomenon which doesn't involve the other at the same timetime, like knife edge diffraction and thin film interference.
Single and double slit diffraction/interference involve both phenomena at once, thus aren't that useful to explain the difference between them.
Title: Re: why do a lot of people confuse between interference and diffraction?
Post by: hamdani yusuf on 10/12/2023 13:13:10
Mechanism for interference is good.

But the cause is incomplete. The light sources need to have the same frequency and polarization.

The pattern is misleading. The width of the bands don't have to be equal, and the contrast don't always high, which is not well defined anyway.
Title: Re: why do a lot of people confuse between interference and diffraction?
Post by: hamdani yusuf on 10/12/2023 13:46:51
Wow, it's almost like Bard is just a chat bot, I wonder why that is?
The model I used is still in experimental phase, which is expected to be improved over time, before it's released for public use. It's expected/prepared to be a study companion to students thus it should be reliably accurate to avoid unnecessary confusion.
Title: Re: why do a lot of people confuse between interference and diffraction?
Post by: hamdani yusuf on 10/12/2023 13:49:49
In answer to the OP, I would say the problem is 'a lot of people' don't care enough to take the time to find out the difference or they have reading comprehension issues.
IMO, misleading online sources also played significant role here.
Title: Re: why do a lot of people confuse between interference and diffraction?
Post by: Origin on 10/12/2023 15:01:17
IMO, misleading online sources also played significant role here.
Sources are easy enough to check.  An easy way to check if something on the Internet is science or not is to go to a university site and see if they are teaching that 'something'.
Title: Re: why do a lot of people confuse between interference and diffraction?
Post by: alancalverd on 10/12/2023 19:00:30
Diffraction without interference doesn't produce such patterns.
Look at an x-ray diffraction pattern.
Title: Re: why do a lot of people confuse between interference and diffraction?
Post by: hamdani yusuf on 11/12/2023 12:11:03
Diffraction without interference doesn't produce such patterns.
Look at an x-ray diffraction pattern.
It involves interference. There are more than one edges contributing to the resulting pattern.
Title: Re: why do a lot of people confuse between interference and diffraction?
Post by: hamdani yusuf on 11/12/2023 12:18:02
IMO, misleading online sources also played significant role here.
Sources are easy enough to check.  An easy way to check if something on the Internet is science or not is to go to a university site and see if they are teaching that 'something'.

Though practical, we need to be aware of its limitations.
Quote
https://en.wikipedia.org/wiki/Argument_from_authority
An argument from authority (argumentum ab auctoritate), also called an appeal to authority, or argumentum ad verecundiam (argument against shame), is a form of argument in which the mere fact that an influential figure holds a certain position is used as evidence that the position itself is correct.[1] While it is not a valid form of logical proof,[1] it is a practical and sound way of obtaining knowledge that is generally likely to be correct when the authority is real, pertinent, and universally accepted.

This argument has been considered a logical fallacy since its introduction by John Locke and Richard Whately.[4] In particular, this is a form of genetic fallacy; in which the conclusion about the validity of a statement is justified by appealing to the characteristics of the person who is speaking, such as in the ad hominem fallacy.[5] For this argument, Locke coined the term argumentum ad verecundiam (appeal to shamefacedness/modesty) because it appeals to the fear of humiliation by appearing disrespectful to a particular authority.[6]

This qualification as a logical fallacy implies that this argument is invalid when using the deductive method, and therefore it can't be presented as infallible.[7] In other words, it's logically invalid to prove a claim is true because an authority has said it. The explanation is simple: authorities can be wrong, and the only way of logically proving a claim is providing real evidence and/or a valid logical deduction of the claim from the evidence.
Title: Re: why do a lot of people confuse between interference and diffraction?
Post by: hamdani yusuf on 20/02/2024 11:57:38
After a while, I asked Google Bard again about the difference between diffraction and interference. This time I specifically mentioned light.

What's the difference between diffraction and interference of light?
Quote

While both diffraction and interference are phenomena related to light waves, they have key differences in their origin and characteristics.

Diffraction:

Cause: Occurs when a light wave encounters an obstacle or opening.
Mechanism: The edges of the obstacle or opening act as sources of secondary wavelets, which spread out and interfere with each other.
Pattern: The resulting pattern consists of bright and dark bands, with the intensity and width of the bands varying depending on the size and shape of the obstacle.
Examples: Rainbows, the diffraction pattern of a single slit or double slit, and the formation of Fresnel zones.

Interference:

Cause: Occurs when two or more coherent light waves overlap.
Mechanism: The crests and troughs of the waves combine, leading to constructive interference (where the amplitudes add up) and destructive interference (where the amplitudes cancel out).
Pattern: The resulting pattern consists of alternating bright and dark bands with equal widths and high contrast.
Examples: Young's double-slit experiment, thin-film interference in soap bubbles and oil slicks, and the formation of Newton's rings.
It doesn't seem to improve much from the last time I asked. Bard's alternative answers don't give much better answers.
Now, Bard has been superseded by Gemini. I asked the same question to it. What's the difference between interference and diffraction of light?
Quote
Interference and diffraction in light, while related, are distinct phenomena caused by different mechanisms:

Interference:

Occurs when two or more light waves interact with each other.
Requires coherent sources, meaning the waves have the same wavelength and are in phase (aligned peaks and troughs).
Can be constructive (bright areas) or destructive (dark areas) depending on the relative phase difference between the waves.
Creates distinct fringe patterns with alternating bright and dark bands.
Examples: double-slit experiment, Newton's rings.


Diffraction:

Occurs when a light wave encounters an obstacle or opening.
Caused by the bending of the wavefront due to secondary wavelets generated from different parts of the wave passing the obstacle.
Affects the wave's propagation by spreading it out beyond its original path.
Creates bending and fading around the edges of the obstacle.
Examples: light passing through a narrow slit, shadows with blurred edges.

It has shown some improvements.
Title: Re: why do a lot of people confuse between interference and diffraction?
Post by: hamdani yusuf on 20/02/2024 12:04:10
I also asked about the answer to same question according to Richard Feynman.
Quote
Interestingly, Richard Feynman had a unique perspective on the difference between interference and diffraction, suggesting that it's more a matter of context and usage than a clear physical distinction. He famously stated:

"No one has ever been able to define the difference between interference and diffraction satisfactorily. It is just a question of usage, and there is no specific, important physical difference between them."

Feynman argued that both phenomena arise from the same underlying principle ? wave interaction. What we call interference or diffraction depends on the number of wave sources involved:

Interference: When only a few sources (typically two) interact, we typically refer to it as interference. This leads to the characteristic fringe patterns observed in experiments like the double-slit experiment.

Diffraction: When we deal with a large number of sources (like the edges of an object), the interaction becomes more complex and diffuse. This is generally referred to as diffraction. The spreading and bending of light around edges exemplify this phenomenon.

While Feynman acknowledged the practical distinction in usage, he emphasized that both aspects arise from the same fundamental wave behavior. This perspective highlights the interconnectedness of wave phenomena and encourages deeper understanding beyond rigid labels.

It's important to note that while Feynman's view offers a nuanced perspective, many other physicists do differentiate between interference and diffraction based on the specific mechanisms involved. The table I provided earlier reflects this more common approach, which offers a clear framework for understanding both phenomena.

Ultimately, both perspectives are valid and have their own merits. Choosing the approach that best suits your learning style and level of understanding is key.
IMO it's a problem when two contradicting statements are said to be equally valid. Because the information content would be zero.
Note that one bit of information reduces uncertainty by a half.

Fortunately, it only appeared when it's forced to by my later question.
Title: Re: why do a lot of people confuse between interference and diffraction?
Post by: alancalverd on 20/02/2024 13:05:21
A chatbot that doesn't recognise the difference between an edge (passive, absorptive) and a source (active, generative) is not worth consulting.
Title: Re: why do a lot of people confuse between interference and diffraction?
Post by: hamdani yusuf on 20/02/2024 13:20:12
A chatbot that doesn't recognise the difference between an edge (passive, absorptive) and a source (active, generative) is not worth consulting.
Which statement indicates the mixing between those things?
Title: Re: why do a lot of people confuse between interference and diffraction?
Post by: Origin on 20/02/2024 19:26:39
why do a lot of people confuse between interference and diffraction?  Because they do not have a good understanding of either of the terms.
Title: Re: why do a lot of people confuse between interference and diffraction?
Post by: alancalverd on 20/02/2024 20:47:42
Which statement indicates the mixing between those things?


Quote
When we deal with a large number of sources (like the edges of an object)

The Huygens construction treats an edge as though it is a line source. It obviously isn't. Common problem of confusing a model with reality.
Title: Re: why do a lot of people confuse between interference and diffraction?
Post by: hamdani yusuf on 21/02/2024 09:00:46
This was Gemini's answer when I asked about the source it used.
Quote
My response on the differences between interference and diffraction of light is based on information from a variety of sources, including:

Scientific textbooks:
"Fundamentals of Optics" by Francis A. Jenkins and Harvey E. White
"Physics for Scientists and Engineers: Optics" by Raymond A. Serway and John W. Jewett Jr.
Online resources:
Khan Academy articles on interference and diffraction
The Physics Classroom website
The MIT OpenCourseware website
Academic research papers:
"The double-slit experiment: A review" by Thomas Young
"Diffraction of light: A historical perspective" by Edmund Hecht
While I cannot directly cite specific pages or paragraphs due to my nature as a large language model, I can assure you that my response is consistent with the established scientific understanding of these phenomena.

If you'd like to explore any specific sources in more detail, I'd be happy to assist you in finding them!
Title: Re: why do a lot of people confuse between interference and diffraction?
Post by: alancalverd on 21/02/2024 09:22:02
In other words, "I don't know and can't be bothered to find out".

I've always defined intelligence as "constructive laziness". Well, it seems that Gemini certainly meets half of that definition.
Title: Re: why do a lot of people confuse between interference and diffraction?
Post by: hamdani yusuf on 21/02/2024 15:32:01
In other words, "I don't know and can't be bothered to find out".

I've always defined intelligence as "constructive laziness". Well, it seems that Gemini certainly meets half of that definition.
At least it has shown some improvements compared to its predecessor, Bard. It has unlearned some erroneous information and acquired more correct information. What's unclear is whether it made those changes through self learning by analyzing all relevant information, and then rejected contradicting statements, or someone acted as its supervisors and forced it to accept newer information.
Title: Re: why do a lot of people confuse between interference and diffraction?
Post by: alancalverd on 21/02/2024 18:28:58
As if newer equals better.

The laws of physics are very old, as are Huygens' very effective construction and the simple mathematics of superposition. Why bother with the new, or multiple sources of dubious validity, when every physicist knows the answer?
Title: Re: why do a lot of people confuse between interference and diffraction?
Post by: hamdani yusuf on 21/02/2024 22:16:28
Quote
Huygens' principle, while groundbreaking and valuable in understanding wave propagation, does have some limitations when it comes to explaining the entirety of light's behavior:

1. Inability to explain the particle nature of light: Huygens conceived light as purely wave-like, with wavefronts emanating from each point on the initial wave. However, modern physics reveals that light also exhibits particle-like behavior, explained by quantum mechanics. Huygens' principle doesn't encompass this duality.

2. Dependence on the hypothetical "ether": The principle proposed that light needed a medium, called the "ether," to propagate. However, numerous experiments failed to detect such a medium, rendering this aspect of the theory obsolete.

3. Incomplete explanation of light interactions: While useful for diffraction and wavefront construction, Huygens' principle doesn't fully explain why light bends (refraction) as it enters different mediums. Additionally, it falls short in explaining phenomena like light polarization and the photoelectric effect.

4. Difficulty handling specific wave behaviors: The principle struggles to accurately predict the exact intensity distribution in diffraction patterns, especially near sharp edges or narrow slits. This is because it doesn't fully account for the constructive and destructive interference between secondary wavelets in these scenarios.

5. Lack of explanation for energy transfer: Huygens' principle describes wave propagation but doesn't delve into how light interacts with matter and transfers energy. This aspect is crucial for understanding phenomena like absorption and scattering, which remain outside the scope of the principle.

Overall, Huygens' principle remains a valuable tool for understanding wave behavior in light, particularly diffraction and wavefront construction. However, its limitations highlight the need for more comprehensive theories like quantum mechanics to explain the full spectrum of light's characteristics and interactions.
That's its answer when I asked about limitations of Huygens' principle.
Title: Re: why do a lot of people confuse between interference and diffraction?
Post by: Origin on 22/02/2024 00:23:35
That's its answer when I asked about limitations of Huygens' principle.
What part do you not understand about Bard being a chatbot?
Title: Re: why do a lot of people confuse between interference and diffraction?
Post by: hamdani yusuf on 22/02/2024 21:56:34
That's its answer when I asked about limitations of Huygens' principle.
What part do you not understand about Bard being a chatbot?
What makes you think I don't understand it?
Btw, it's Gemini, the successor of Bard. It has better math skill.
Title: Re: why do a lot of people confuse between interference and diffraction?
Post by: alancalverd on 22/02/2024 22:12:17
Huygens' principle, while groundbreaking and valuable in understanding wave propagation, does have some limitations when it comes to explaining the entirety of light's behavior:
Wrong, right from the start.
It doesn't purport to be an explanation of the entirety of light's behavior. It is a mathematical model of the propagation of a wave.
I have previously suggested that ChatGPT produces text that might merit a B or C mark for a 14-year-old, but this crap falls below that standard.
Try asking what are the limitations of a fish.
Title: Re: why do a lot of people confuse between interference and diffraction?
Post by: varsigma on 22/02/2024 23:18:55
Your average photon is a quantum thingy.

What does that mean, aside from my laconic description?

It means light diffracts around solid objects, and it interferes when it interacts with itself.
Photons (of, you know, light) are waves or particles, right?

Almost there, sorta, QM says if a photon is a wave or a particle, then it must be the case that a photon is also a wave and a particle, also, a photon is neither a wave nor a particle.

Sorta does your head in, until you do those which-way experiments with lasers and stuff. Then it's sorta the only possible explanation.
Title: Re: why do a lot of people confuse between interference and diffraction?
Post by: Origin on 23/02/2024 03:47:04
Btw, it's Gemini, the successor of Bard. It has better math skill.
You don't seem to get it.  It's a chatbot.
Title: Re: why do a lot of people confuse between interference and diffraction?
Post by: alancalverd on 23/02/2024 08:03:35
Photons (of, you know, light) are waves or particles, right?
Wrong!

A photon is a mathematical model of how (mostly high energy) electromagnetic radiation can be generated or interact with matter.

A travelling wave is another model, useful for describing the propagation of em radiation or its generation at low frequencies.

Huygens' construction is a usefully predictive development of the wave model.
Title: Re: why do a lot of people confuse between interference and diffraction?
Post by: hamdani yusuf on 23/02/2024 08:07:09
Huygens' principle, while groundbreaking and valuable in understanding wave propagation, does have some limitations when it comes to explaining the entirety of light's behavior:
Wrong, right from the start.
It doesn't purport to be an explanation of the entirety of light's behavior. It is a mathematical model of the propagation of a wave.
I have previously suggested that ChatGPT produces text that might merit a B or C mark for a 14-year-old, but this crap falls below that standard.
Try asking what are the limitations of a fish.
Flat earth model doesn't purport to be an explanation of the entirety of space objects.
Something called principle should be a general pattern useful to explain diverse phenomena.
Title: Re: why do a lot of people confuse between interference and diffraction?
Post by: hamdani yusuf on 23/02/2024 08:42:05
Btw, it's Gemini, the successor of Bard. It has better math skill.
You don't seem to get it.  It's a chatbot.
The veracity of information doesn't depend on the form of its source. It can be statement of a teacher, or colleague, textbooks, online encyclopedia, news, TV, website, blogs, social media, or AI models.
Title: Re: why do a lot of people confuse between interference and diffraction?
Post by: alancalverd on 23/02/2024 10:18:20
Something called principle should be a general pattern useful to explain diverse phenomena.

Huygen's construction doesn't purport to explain anything. Nor does a timetable explain how a bus works. But both are quite good at predicting what will happen in specified circumstances. (In the UK, train timetables are a waste of paper.)
Title: Re: why do a lot of people confuse between interference and diffraction?
Post by: alancalverd on 23/02/2024 10:24:27
The veracity of information doesn't depend on the form of its source. It can be statement of a teacher, or colleague, textbooks, online encyclopedia, news, TV, website, blogs, social media, or AI models.
But it has to be correct, and if it is in response to a question, it has to be relevant and informative. Better still if it is actually helpful.

One line of code:

IF CHAR=<?> THEN PRINT "the answer may be found in textbooks". 

How's that for efficient programming, in genuine 1960s Algol!
Title: Re: why do a lot of people confuse between interference and diffraction?
Post by: varsigma on 23/02/2024 20:16:43
Wrong!

A photon is a mathematical model of how (mostly high energy) electromagnetic radiation can be generated or interact with matter.
Actually, that is not even wrong. As I go on to say. And the experiments say it too.

A Feynman diagram can also say it, of course.
Title: Re: why do a lot of people confuse between interference and diffraction?
Post by: alancalverd on 23/02/2024 21:00:44
I would class a Feynman diagram as a mathematical model, as, I think, would Feynman.

Experimental evidence is that some electromagnetic radiation is generated or absorbed in discrete amounts, as evidenced by line spectra and photon-counting receptors, but there are no inherent discontinuities in the radiofrequency spectrum, and the photon model doesn't predict diffraction.

Some folk still talk about "duality" but that's clearly nonsense. Only a deity can change its form ad lib (in Greek mythology) or exist in three distinct physical forms at once (Christian mythology). Real phenomena sometimes need more than one descriptive and predictive model.
Title: Re: why do a lot of people confuse between interference and diffraction?
Post by: Origin on 23/02/2024 23:30:06
The veracity of information doesn't depend on the form of its source. It can be statement of a teacher, or colleague, textbooks, online encyclopedia, news, TV, website, blogs, social media, or AI models.
Perhaps that's the issue.  Without critical thinking skills, all sources no matter how unreliable seem to be on equal footing?
Title: Re: why do a lot of people confuse between interference and diffraction?
Post by: varsigma on 24/02/2024 05:49:19
I would class a Feynman diagram as a mathematical model, as, I think, would Feynman.
I would say, if I was asked, that a Feynman diagram is a representation of a(n element of a) symmetry group. Namely U(1) symmetry. The fermions are there for closure (that's over the algebra).

With the caveat that the symmetries are not exact according to modern theories.
Title: Re: why do a lot of people confuse between interference and diffraction?
Post by: hamdani yusuf on 24/02/2024 09:56:28
Huygen's construction doesn't purport to explain anything.
It would make it useless.
Title: Re: why do a lot of people confuse between interference and diffraction?
Post by: hamdani yusuf on 24/02/2024 10:06:12
The veracity of information doesn't depend on the form of its source. It can be statement of a teacher, or colleague, textbooks, online encyclopedia, news, TV, website, blogs, social media, or AI models.
But it has to be correct, and if it is in response to a question, it has to be relevant and informative. Better still if it is actually helpful.

One line of code:

IF CHAR=<?> THEN PRINT "the answer may be found in textbooks". 

How's that for efficient programming, in genuine 1960s Algol!
Gemini's answer is better than most sources found online which I posted on the first page. Some of them are physics courses for students. It's even better than Feynman's answer, which was a leading scientist.
Title: Re: why do a lot of people confuse between interference and diffraction?
Post by: hamdani yusuf on 24/02/2024 10:17:02
I would class a Feynman diagram as a mathematical model, as, I think, would Feynman.
I would say, if I was asked, that a Feynman diagram is a representation of a(n element of a) symmetry group. Namely U(1) symmetry. The fermions are there for closure (that's over the algebra).

With the caveat that the symmetries are not exact according to modern theories.
How can Feynman's diagram be used to explain diffraction and interference?
Title: Re: why do a lot of people confuse between interference and diffraction?
Post by: hamdani yusuf on 24/02/2024 10:26:37
The veracity of information doesn't depend on the form of its source. It can be statement of a teacher, or colleague, textbooks, online encyclopedia, news, TV, website, blogs, social media, or AI models.
Perhaps that's the issue.  Without critical thinking skills, all sources no matter how unreliable seem to be on equal footing?
That's why Jordan Peterson said that people who are not that smart are better to be conservative, and follow the mainstream.
Title: Re: why do a lot of people confuse between interference and diffraction?
Post by: alancalverd on 24/02/2024 16:30:11
Gemini's answer is better than most sources found online which I posted on the first page.
Which is a great shame, because it is complete garbage.
Title: Re: why do a lot of people confuse between interference and diffraction?
Post by: alancalverd on 24/02/2024 16:35:49
I would say, if I was asked, that a Feynman diagram is a representation of a(n element of a) symmetry group. Namely U(1) symmetry.
Or to quote Wikipedia
Quote
In theoretical physics, a Feynman diagram is a pictorial representation of the mathematical expressions describing the behavior and interaction of subatomic particles.
which is pretty much what I said.
How can Feynman's diagram be used to explain diffraction and interference?
It can't, because it is a representation of particle interactions, not waves.
Title: Re: why do a lot of people confuse between interference and diffraction?
Post by: alancalverd on 24/02/2024 16:40:42
Huygen's construction doesn't purport to explain anything.
It would make it useless.
We have very accurate models for predicting sunrise and high tides, but don't explain either. Try telling a mariner that they are useless.
Title: Re: why do a lot of people confuse between interference and diffraction?
Post by: varsigma on 24/02/2024 18:13:38
which is pretty much what I said.
There it is. We are saying basically the same thing. At least I think we are.

Feynman diagrams are composable, There are sums over all the ways a photon can be emitted and absorbed. How many do you need to glue together to represent this? That was I think, what Feynman was using his "new" notation for.
Title: Re: why do a lot of people confuse between interference and diffraction?
Post by: hamdani yusuf on 25/02/2024 07:50:40
Huygen's construction doesn't purport to explain anything.
It would make it useless.
We have very accurate models for predicting sunrise and high tides, but don't explain either. Try telling a mariner that they are useless.
What is predicted by Huygens construction, when a laser beam hit the edge of a totally reflective surface?
Title: Re: why do a lot of people confuse between interference and diffraction?
Post by: alancalverd on 25/02/2024 08:34:05
Edge (dimension → 0) or surface (dimension → ∞)? One produces diffraction, the other produces reflection. Real objects fit somewhere between.
Title: Re: why do a lot of people confuse between interference and diffraction?
Post by: hamdani yusuf on 25/02/2024 09:37:11
Edge (dimension → 0) or surface (dimension → ∞)? One produces diffraction, the other produces reflection. Real objects fit somewhere between.
Point, 0 dimension.
Edge, 1 dimension.
Surface, 2 dimensions.
Volume, 3 dimensions.
Title: Re: why do a lot of people confuse between interference and diffraction?
Post by: alancalverd on 25/02/2024 14:17:59
OK,I wasn't being pedantic enough!

The length of an edge is immaterial to the Huygens construction of diffraction. All that matters is that, perpendicular to the wavefront, it can be mapped to a point without losing important information.

You can't usefully map a surface to a point.
Title: Re: why do a lot of people confuse between interference and diffraction?
Post by: hamdani yusuf on 29/02/2024 10:04:32
I searched for an image of knife edge diffraction.
Here's what I found.

http://www.mike-willis.com/Tutorial/PF7.htm

The question is, how much of the wave is diffracted? How to determine the angle?
(https://www.thenakedscientists.com/forum/index.php?action=dlattach;topic=77918.0;attach=34266)
Title: Re: why do a lot of people confuse between interference and diffraction?
Post by: alancalverd on 29/02/2024 11:29:03
A rather blunt knife, but at least commensurate with the wavelength, so Huygens is a good approximation.
Title: Re: why do a lot of people confuse between interference and diffraction?
Post by: hamdani yusuf on 01/03/2024 02:24:44
A rather blunt knife, but at least commensurate with the wavelength, so Huygens is a good approximation.
Do the characteristics of the barrier have any effect?
What does Huygens' principle say about this? Is there any quantitative statement?
What's your threshold for being good?
Title: Re: why do a lot of people confuse between interference and diffraction?
Post by: alancalverd on 01/03/2024 09:41:43
Huygens is a mathematical construction involving points, lines, and absolute absorption and reflection. Anything that isn't a mathematical point (zero dimension) or line (1 dimension), or is a real material, is an approximation.

If the "edge" is actually a surface, and/or the source has finite width, you need to consider the contribution of all the points on the surface and the source. Intuitively (though I guess ES probably has an appropriate integral) this means that the diffraction pattern becomes more diffuse if either is wider than λ.   
Title: Re: why do a lot of people confuse between interference and diffraction?
Post by: hamdani yusuf on 02/03/2024 09:39:30
A rather blunt knife, but at least commensurate with the wavelength, so Huygens is a good approximation.
Do the characteristics of the barrier have any effect?
What does Huygens' principle say about this? Is there any quantitative statement?
What's your threshold for being good?
Don't forget that I also have made the same experiment in microwave frequency.

I have uploaded new video showing diffraction in microwave frequency.


Basically, the experiment result leads us to conclude that diffraction comes from the material blocking the microwave path. When the obstruction is opaque enough, we find no diffraction. It's similar to my experiment using laser showing non-diffractive obstruction.

This result is not widely known yet.
 
Title: Re: why do a lot of people confuse between interference and diffraction?
Post by: varsigma on 02/03/2024 10:21:15
Just a question:

Have you heard of those guys Fraunhoffer and Fresnel?
Title: Re: why do a lot of people confuse between interference and diffraction?
Post by: hamdani yusuf on 02/03/2024 10:59:21
Just a question:

Have you heard of those guys Fraunhoffer and Fresnel?

Yes
Title: Re: why do a lot of people confuse between interference and diffraction?
Post by: hamdani yusuf on 02/03/2024 14:22:48
https://en.m.wikipedia.org/wiki/Fresnel_diffraction

(https://upload.wikimedia.org/wikipedia/commons/6/61/Comparison_Sommerfled-Fresnel-Fraunhofer.gif)
Quote
Comparison between the diffraction pattern obtained with the Rayleigh?Sommerfeld equation, the (paraxial) Fresnel approximation, and the (far-field) Fraunhofer approximation

Calling it a diffraction pattern contributed to the widespread confusion. It should be called interference pattern from diffracted light rays.


Title: Re: why do a lot of people confuse between interference and diffraction?
Post by: hamdani yusuf on 02/03/2024 15:20:02
When the obstruction is opaque enough, we find no diffraction.
It's related to penetration depth, compared to the wavelength. For common metal plates, such as aluminum, penetration depth in microwave frequency is much smaller than the wavelength.
While in visible light, the penetration depth is larger than the wavelength.
Although in both cases, the penetration depth isn't that much different, in the range of microns. The vast difference in wavelength causes the difference in the results.
Title: Re: why do a lot of people confuse between interference and diffraction?
Post by: varsigma on 04/03/2024 22:43:30
The appearance of this thread, in say, the context of a court of some kind, is that of someone making their case about a particular effect, observed in good old nature.

Except, they are stuck it seems, with the one approach. It's classical, this approach. And yet the petitioner, if you will, is trying to understand an entirely local effect. The effect of diffraction of light at the boundary of a material surface, is localized, by ah, the surface.

Material means made out of atoms. Light is made out of particles of light, but a particle isn't a little solid ball of whatever. it just isn't. The classical approach only gets you near to the "surfaces" you desperately want to characterize.

You need to switch donkeys. Read Don Quixote maybe. Study Picasso. Hell, I don't know, get a degree in Quantum Physics.
Title: Re: why do a lot of people confuse between interference and diffraction?
Post by: varsigma on 04/03/2024 22:58:23
Point, 0 dimension.
Edge, 1 dimension.
Surface, 2 dimensions.
Volume, 3 dimensions.

And physics in the classical world we perceive, stops there. Mathematics, and in particular, the mathematics of quantized interaction 'spaces', doesn't stop. Why stop with 4 dimensions? Riemann once asked why stop going backwards once you get to zero? what about -1 dimensions etc?

He was saying we can be moving forward at the end of four dimensions, and moving backward at the beginning, or something, at the end of the day
Title: Re: why do a lot of people confuse between interference and diffraction?
Post by: hamdani yusuf on 05/03/2024 04:02:48
You need to switch donkeys. Read Don Quixote maybe. Study Picasso. Hell, I don't know, get a degree in Quantum Physics.
Can't you distinguish between interference and diffraction without invoking quantum physics?
Title: Re: why do a lot of people confuse between interference and diffraction?
Post by: varsigma on 05/03/2024 08:28:54
Can't you distinguish between interference and diffraction without invoking quantum physics?
No. At least most modern day physicists will tell you it can't be done. The classical theory of diffraction obviously is a good enough result, to first order, ('chortle').

About Don Quixote and Picasso. Sometimes learning mathematics is like reading one or trying to understand the other.

Then you realize, you are reading one and trying to understand the other. The explanations are straight out of that wonderfully paradoxical story, and the diagrams and equations are Picasso on bloody steroids I tell ye.
Title: Re: why do a lot of people confuse between interference and diffraction?
Post by: alancalverd on 05/03/2024 10:21:22
Can't you distinguish between interference and diffraction without invoking quantum physics?
I can, and have done several times in this thread.

The phenomenon of diffraction can be modelled with a single wavefront and Huygens' construction

Modelling interference requires at least two wavefronts and the principle of superposition

Sometimes, both happen.
Title: Re: why do a lot of people confuse between interference and diffraction?
Post by: varsigma on 05/03/2024 12:22:57
'sigh'

I thought there was a reasonably big clue about measuring an effect. It means you break a symmetry.

So what is the measured difference between interference and diffraction, what are the measurements in either effect so you can say what their difference is? Would that answer the question about the confusion between?

Another problem with light doing this, is that all waves interfere and diffract around barriers. Ocean waves do it, radio waves do it. All waves do it.

Title: Re: why do a lot of people confuse between interference and diffraction?
Post by: alancalverd on 05/03/2024 15:12:13
A nondirectional radio beacon (NDB) is what it says: effectively, a medium-frequency point source in the middle of an airfield. You use a rotating aerial to search for a signal minimum, then fly at 90 degrees to that minimum to reach your destination. VHF ground-to-air communication is likewise usually nondirectional.

Radio waves are diffracted by mountains and other large structures. Flying to an NDB in zero wind, you expect to hold a constant compass bearing. If you descend below the height of an intervening structure, the apparent bearing changes towards the edge of that structure.

If you have two transmitters running at the same frequency, you can encounter periodic maxima and minima, and garbled audio, distributed spatially, thanks to interference.

The solution to diffraction is to fly higher. The solution to interference is to request one transmitter to shut down. It helps to know which problem you are dealing with.
Title: Re: why do a lot of people confuse between interference and diffraction?
Post by: varsigma on 05/03/2024 21:08:47
The solution to diffraction is to fly higher. The solution to interference is to request one transmitter to shut down. It helps to know which problem you are dealing with.

Right. I would say the essential difference is that a single source of wavefronts generates waves that will diffract when they encounter surfaces. Otherwise interference is measured or observed when there are two or more sources.
And being a part-time physicist, I note that radio waves are photons with a long wavelength, so a classical result would be expected and I can ignore the fact that photons are quanta.

Actually that's what a lot of physics does. Perhaps because a classical approximation does work quite well. Nasa doesn't design rockets that have to cope with quantum gravity even though we're pretty sure gravity is quantized.
Title: Re: why do a lot of people confuse between interference and diffraction?
Post by: alancalverd on 06/03/2024 10:06:07
a single source of wavefronts generates waves that will diffract when they encounter surfaces.
Pedant hat firmly in place, I would say "edges" , not "surfaces". Surfaces reflect and/or absorb, but em radiation doesn't "bend" round a surface!

I've been pondering on the photon model of radio waves. We generate them by a continuous process - essentially, sinusoidal alternating current in a wire - with no essential discontinuity of frequency or duration. The energy received from a radio transmitter depends only on how long you listen to it  - it doesn't arrive in discrete packets. Why assume that it is quantised?
Title: Re: why do a lot of people confuse between interference and diffraction?
Post by: varsigma on 06/03/2024 12:09:15
a single source of wavefronts generates waves that will diffract when they encounter surfaces.
Pedant hat firmly in place, I would say "edges" , not "surfaces". Surfaces reflect and/or absorb, but em radiation doesn't "bend" round a surface!

I've been pondering on the photon model of radio waves. We generate them by a continuous process - essentially, sinusoidal alternating current in a wire - with no essential discontinuity of frequency or duration. The energy received from a radio transmitter depends only on how long you listen to it  - it doesn't arrive in discrete packets. Why assume that it is quantised?

Ok sorry, I might have still had my topologist's hat on, where a surface can have less than two dimensions "effectively".

You can treat a barrier like a 1-dimensional 'surface' if it acts as a waveguide, more or less, e.g. a breakwater, but along the surface, not below it. That comes with the idea that it's  some kind of aerial if we are talking radio waves. That 'could' be a gotcha that isn't. A radio broadcast antenna is a 1-dimensional waveguide.
Title: Re: why do a lot of people confuse between interference and diffraction?
Post by: hamdani yusuf on 07/03/2024 13:57:31
My answer to the question in the title is because they have learned from erroneous primary sources. And they were not equipped with necessary tools to distinguish between the correct and incorrect explanations.
Title: Re: why do a lot of people confuse between interference and diffraction?
Post by: alancalverd on 09/03/2024 00:40:03
Or maybe because it isn't all that important to most people. To take my example in reply #370 above, the pilot needs to know the difference but 300 passengers, and anyone else who isn't a pilot, doesn't.
Title: Re: why do a lot of people confuse between interference and diffraction?
Post by: hamdani yusuf on 09/03/2024 05:15:40
Or maybe because it isn't all that important to most people. To take my example in reply #370 above, the pilot needs to know the difference but 300 passengers, and anyone else who isn't a pilot, doesn't.
That's right. Most people don't have to know many things because they rely on someone else to know those things to do the necessary activities for them. Not many people can perform brain surgery, or grow staple food reliably, or build livable houses and bridges, or make comfortable shoes. But someone who does have to do those activities need to know the correct information related to those activities. Imagine mechanical engineers trained using impetus theory.
Title: Re: why do a lot of people confuse between interference and diffraction?
Post by: alancalverd on 09/03/2024 10:03:39
Or civil engineers brought up on Aristotelian mechanics. Like the guys who built the Roman sewers, Hadrian's Wall, Angkor Wat, the Pyramids, Stonehenge, and all that other stuff that has hardly lasted 6000 years.....Rubbish, all of it!
Title: Re: why do a lot of people confuse between interference and diffraction?
Post by: hamdani yusuf on 11/03/2024 14:21:02
Or civil engineers brought up on Aristotelian mechanics. Like the guys who built the Roman sewers, Hadrian's Wall, Angkor Wat, the Pyramids, Stonehenge, and all that other stuff that has hardly lasted 6000 years.....Rubbish, all of it!
Pyramids builders lived before Aristotle. They used mathematical model good enough to build stationary buildings. Earth curvature and rotation don't have significant effect on their results.
Title: Re: why do a lot of people confuse between interference and diffraction?
Post by: alancalverd on 11/03/2024 16:20:21
But they are all subject to gravitational and meteorological influences, which are still not fully understood!
Title: Re: why do a lot of people confuse between interference and diffraction?
Post by: hamdani yusuf on 15/03/2024 11:58:33
But they are all subject to gravitational and meteorological influences, which are still not fully understood!
That's a reason why we need to improve our understanding of physical reality, by pointing out limitations of our models.
Title: Re: why do a lot of people confuse between interference and diffraction?
Post by: alancalverd on 15/03/2024 16:12:04
Why bother? They have all outlived their supposed usefulness - apart from the Roman sewers!
Title: Re: why do a lot of people confuse between interference and diffraction?
Post by: hamdani yusuf on 17/03/2024 08:42:08
That's a reason why we need to improve our understanding of physical reality, by pointing out limitations of our models.
Why bother? They have all outlived their supposed usefulness - apart from the Roman sewers!

Because unexpected results come from false assumptions. We don't know what problems are waiting for us to solve in the future. And wrong decisions might bring unacceptable consequences.
Title: Re: why do a lot of people confuse between interference and diffraction?
Post by: alancalverd on 17/03/2024 10:10:37
There's an old adage that "thermodynamics owes more to the steam engine than the steam engine owes to thermodynamics".

A great deal of science begins with an engineering failure, and we question our hypotheses in the light of the unexpected finding, because that shows us where to look!
Title: Re: why do a lot of people confuse between interference and diffraction?
Post by: hamdani yusuf on 17/03/2024 11:03:04
When we found that someone else is using the wrong model of the universe to make decisions, we should at least tell them that there is a better model, and describe the situation where the difference between those models become significant.

Quote
-- Donald Trump supporters are interviewed in Rome, Georgia, revealing that their reasons for supporting Trump are a confusing mishmash of misunderstandings and confusion
---
Title: Re: why do a lot of people confuse between interference and diffraction?
Post by: alancalverd on 18/03/2024 08:37:21
When we found that someone else is using the wrong model of the universe to make decisions, we should at least tell them that there is a better model, and describe the situation where the difference between those models become significant.
.....and join the ranks of Bruno, Galileo, Solzhenitsyn, Navalny......Even if your discovery would benefit those in power, you have to face down all the criticism of your peers as Einstein and Darwin did. 
Title: Re: why do a lot of people confuse between interference and diffraction?
Post by: hamdani yusuf on 18/03/2024 15:39:04
When we found that someone else is using the wrong model of the universe to make decisions, we should at least tell them that there is a better model, and describe the situation where the difference between those models become significant.
.....and join the ranks of Bruno, Galileo, Solzhenitsyn, Navalny......Even if your discovery would benefit those in power, you have to face down all the criticism of your peers as Einstein and Darwin did. 
That's why you need strategies. Direct confrontation doesn't always give the best results.
Title: Re: why do a lot of people confuse between interference and diffraction?
Post by: hamdani yusuf on 23/04/2024 22:39:01
This lecture explicitly show double slit experiment as an example of interference, and single slit experiment for diffraction. If left uncorrected, the confusion will spread among students. To make it worse, they will get positive feedbacks in the exams by giving wrong answers.
Title: Re: why do a lot of people confuse between interference and diffraction?
Post by: alancalverd on 24/04/2024 19:53:34
This lecture explicitly show double slit experiment as an example of interference, and single slit experiment for diffraction.
What would you expect from a guy who can't spell diffraction? 
Title: Re: why do a lot of people confuse between interference and diffraction?
Post by: hamdani yusuf on 28/04/2024 01:38:16
This lecture explicitly show double slit experiment as an example of interference, and single slit experiment for diffraction.
What would you expect from a guy who can't spell diffraction? 
I don't think that language is a major obstacle in understanding scientific explanations. Especially when online translators are freely available.
Title: Re: why do a lot of people confuse between interference and diffraction?
Post by: Eternal Student on 28/04/2024 02:37:08
Hi.

If left uncorrected, the confusion will spread among students. To make it worse, they will get positive feedbacks in the exams by giving wrong answers.

1.   It's not in my power to make all students and teaching staff use the terminlogy only a certain way, sorry.

2.    Some of the important ideas for diffraction and interference are the same.   Blending the terms may not be as bad as you think.   Scientific terminology evolves just like all spoken languages evolve and develop.   If it became essential to point out some differences between one thing and another, there are other ways and other terminology we can use.

Evolving terminology isn't always bad:
       In Britain, the word   "inflammable" meant that something could easily be set on fire or was some kind of fire hazard.   The etymology of the word is from the Latin inflammabilis.    However, English started to evolve in a way where adding "in" as a prefix in front of a word indicated the negation or opposite.   
Examples:   Hospitable  and  Inhospitable;      Tolerable   and   Intolerable;   Edible and Inedible......  plus many others.

As a consequence some people thought that "inflammable" meant that something would NOT burn and indeed, when surveys were done in some workplaces, a few people thought they might be able to use an inflammable material to put a fire out if one started somewhere.   Obviously this is a serious problem.   So the resolution was to change the terminology assocated with something that will burn or ignite easily easily.   HazChem warnings now say  "Flammable"  instead of "Inflammable".     This was probably a senisble decision and may have saved lives.    Technically, it's wrong on many levels, the Latin roots have been lost and the word "Flammable" was just nonsense that wasn't in any dictionary when the decision was made.   However, it was still probably the right decision.

Best Wishes.
Title: Re: why do a lot of people confuse between interference and diffraction?
Post by: alancalverd on 28/04/2024 10:16:35
Some of the important ideas for diffraction and interference are the same. 
I think students should be taught the difference between "one" and "many", or even "two", however unfashionable such pedantry may be.
Title: Re: why do a lot of people confuse between interference and diffraction?
Post by: hamdani yusuf on 01/05/2024 07:33:06
Evolving terminology isn't always bad:
It isn't always good nor acceptable either.
What's the benefit of confusion between diffraction and interference?
Title: Re: why do a lot of people confuse between interference and diffraction?
Post by: hamdani yusuf on 01/05/2024 08:06:30
Some of the important ideas for diffraction and interference are the same.
I think students should be taught the difference between "one" and "many", or even "two", however unfashionable such pedantry may be.
Some people think that single slit experiment is the simplest example of diffraction. They seem to forget that a slit has two edges. Diffraction can be produced by one edge, which is simpler than two.

Interference is better observed by the formation of dark regions caused by destructive interference. It requires at least two sources with similar frequency and polarization.
Title: Re: why do a lot of people confuse between interference and diffraction?
Post by: alancalverd on 01/05/2024 09:09:48
Diffraction is a phenomenon associated with a single source of radiation.

Interference is a different phenomenon which requires (by definition) at least two sources.

What is the problem, for heaven's sake?
Title: Re: why do a lot of people confuse between interference and diffraction?
Post by: hamdani yusuf on 01/05/2024 16:45:17
Fresnel Diffraction at Straight Edge
Quote
Shadow of straight edge at a large distance. Fresnel diffraction pattern.

Experiment like this may contribute to the confusion. Destructive interference seems to occur even when there is only one edge. Don't forget that diffracted light rays can interfere with unobstructed light rays. Just like in Lloyd's mirror, where reflected light rays interfere with direct light rays.
(https://upload.wikimedia.org/wikipedia/commons/1/18/%D0%97%D0%B5%D1%80%D0%BA%D0%B0%D0%BB%D0%BE_%D0%9B%D0%BB%D0%BE%D0%B9%D0%B4%D0%B0.png)
Title: Re: why do a lot of people confuse between interference and diffraction?
Post by: hamdani yusuf on 04/05/2024 00:04:38
My answer to the question in the title is because they have learned from erroneous primary sources. And they were not equipped with necessary tools to distinguish between the correct and incorrect explanations.
By being freely accessible, Wikipedia articles have been the primary sources for many people. When diffraction is defined as interference there, how can they be distinguished from each other?

Quote
Diffraction is the interference or bending of waves around the corners of an obstacle or through an aperture into the region of geometrical shadow of the obstacle/aperture.
https://en.m.wikipedia.org/wiki/Diffraction

Title: Re: why do a lot of people confuse between interference and diffraction?
Post by: alancalverd on 04/05/2024 11:01:28
Time for you to write to Wikipedia. "Aperture" adds nonsense to the confusion: an aperture doesn't have a shadow!
Title: Re: why do a lot of people confuse between interference and diffraction?
Post by: hamdani yusuf on 08/05/2024 14:21:05
Time for you to write to Wikipedia. "Aperture" adds nonsense to the confusion: an aperture doesn't have a shadow!
It says geometrical shadow. Which simply means regions behind the aperture uncovered by geometrical rays from the light source.
Title: Re: why do a lot of people confuse between interference and diffraction?
Post by: alancalverd on 08/05/2024 17:42:14
Then why doesn't it say so?
Title: Re: why do a lot of people confuse between interference and diffraction?
Post by: hamdani yusuf on 09/05/2024 10:39:55
Then why doesn't it say so?

Perhaps it assumes that the statement is simple enough for laypersons to understand what it means.
Title: Re: why do a lot of people confuse between interference and diffraction?
Post by: alancalverd on 09/05/2024 12:20:38
One of the problems with physics is that lay persons and journalists don't understand the difference between  strength, power, force and energy, so they go on to talk nonsense about a zero-carbon economy and other such drivel. The statement that an aperture can cast a shadow is worthy of a politician, not a scientist.